当前位置:首页 -高中英语试卷 - 高中二年级英语试题 - 正文*

高二英语NSEFCIIB单元目标测试卷2005[1].1

2014-5-11 0:24:26下载本试卷

中 学 英 语 能 力          

训练与测试系列    NSEFC- IIB-UNIT-T- 2005            

            高中二年级单元目标测试卷

Unit 11  Scientific achievement

第一节:单元要点 (共15 小题,每小题1 分,满分15 分) 

1.A new cure for the disease is______to be discovered.

  A.like       B.likely     C.possible     D.probable    

2.Do you agree to the plan______at the meeting?

  A.to put forward B.to put away   C.put forward   D.put up     

3.The director on land asked the pilot to______his position.

  A.locate     B.grasp      C.announce    D.arrange     

4.They were filled with joy when they______in this beautiful island.

  A.set free     B.set foot    C.paid visit    D.reached     

5.He was______by new hope.

  A.come to life   B.come true    C.rejuvenated  D.boomed    

6.The Olympics has already______our country's breathtaking economic progress,

  A.showcased    B.achieved    C.boomed      D.grasped

7.Don't forget to take the_______with you.

  A.luggages              B.baggages                       C.two pieces of luggages  D.two luggages  

8.After the president made an official announcement,she expressed her_____opinion.

  A.personal    B.private     C.single      D.individual

9.Listening attentively is_______of the crowd's respect for the speaker.

  A.significant  B.significance   C.sigh       D.signal

10.-Susan,will you please go and empty that drawer?

  -______?(2004 NMET)

  A.What for    B.What is it    C.How is it    D.How come

11.The test we held last time is obviously bad because it______incorrect learning.

  A.relies upon  B.leads to     C.refers to    D.depends on

12.Because of modern______we have a higher standard of living.

  A.technique    B.techniques    C.technology    D.technologies

13.Bethun devoted his whole life to______.

  A.befit mankind           B.befit the mankind

  C.befiting mankind          D.befiting the mankind        

14.The airline______for a plane to take us to Boston yesterday.

  A.called      B.arranged    C.asked      D.sent

15.______teaches______.

   A.Failure;success      B.Success;failure   C.Fail;succeed      D.Succeed;fail

第二节:专题语法练习 (构词法1) (共15 小题,每小题1 分,满分15 分).

    构词法是英语学习的一个重点,也是近年来常考的项目。特别是在词语辨析的综合运用中,对词汇掌握的要求越来越高。检测构词法,在阅读理解中,已不把这种词汇看作生词,而是把这类词的理解看作学生通过对构词法知识的学习,应该具备的一种能力。如有的生词,只要其词根是学生们该掌握的或熟悉的,只是加上了前缀或后缀,再者或是合成词等,命题人就认为不必再注出汉语注释。所以我们在学习英语词汇的时候一定要灵活学习和掌握,随时准备应付所出现的情况。 

16.That man was________enough not to tell the manager that he would not do the job.

  A.care         B.careful       C.careless      D.carelessness

17.The soldier died for saving the child,so his________ is heavier than Mount Tai.

  A.die          B.dead         C.died          D.death

18.He is an expert at chemistry.We all call him a ________.

  A.chemistry      B.chemical      C.chemist       D.physician

19.The three- ________chair isn't suitable for a young child.He may fall off.

  A.legging       B.legged        C.legs          D.leged

20.Stephenson became the________railway engineer in the world.

  A.lead         B.leader        C.leading        D.leadership

21.To everyone's ________,the girl finished the job quite well.

  A.satisfied       B.satisfactory     C.satisfying       D.satisfaction

22. -What are you doing here?

  -Oh,my teacher asked me to write a passage about ________in English.

  -You can write________passage in English?

  A.600 words's 600-words         B.600-word;a 600-words

  C.600 words;a 600-word          D.600 words;a 600-words

23.No one should enter the spot without the________of the police.

  A.permit        B.permission     C.permitting      D.permittence

24.You must come with us to the police ________.Our head is waiting for you.

  A.headquarters    B.headline       C.headmaster     D.headache

25.Letting that animal escape was no accident;you did it ________.

  A.intend        B.intention       C.intentionally     D.intentional

26.The________ordered him to pay a $100 fine.

  A.judger        B.judgment      C.judge         D.judgement

27.My TV is out of order.Can you tell me what is the________news about Iraq War?

  A.lately        B.latest         C.later         D.latter

28.Canada is mainly an________country.

  A.English-speaking B.speak-English   C.spoken-English  D.English-spoken

29.How________ he is! He is always acting________.He is really a ________.

  A.foolish;foolishly;fool                          B.fool;foolish;fool

  C.foolish;fool;fool                D.foolishly;foolish;fool

30.The necklace that she lost is very expensive.It's of great ________.

   A.valuable       B.value         C.valueless       D.unvaluable

第三节:完形填空(共20小题,每小题 1.5 分,满分 30 分)

    阅读下面短文,掌握其大意,然后从各题所给的四个选项中,选出最佳选项,并在答题卡(第10页) 上将该项涂黑。

It happened during the Second World War. A British   31   , called the “Nova Scotia”, was    32    more than a thousand South African    33   and Italian prisoners of war. It was a long way from Europe to South Africa.  But a German submarine (潜艇) was   34    in the sea when the “Nova Scotia”appeared.

The submarine  35  the “Nova Scotia”to the bottom of the sea. Many men were  36  at once. Many others were badly   37   . But hundreds were alive. They were not greatly   38   . The sea was calm and   39   , and they were    40   a ship would find them in the morning. A ship did find them, but it   41   only 183 men. The sharks (鲨鱼) had taken the rest.

Thirty years later, one of the living men said, “I was in a raft (救生筏). There were men all      42    me in the water. They were    43   , or holding onto pieces of wood. Suddenly I heard a   44  . It came from a man who was a few hundred meters   45  . He threw his arms in the air. Then he   46   in the water. He never came   47    again. That was only the beginning. The sharks had smelled the blood of the dead and dying men.   48   seemed to be hundreds of them. Man after man gave a   49    shout and then disappeared. I even saw sharks attack men on the rafts. They   50   right out of the water.”

31. A. train     B. ship       C. hospital    D. school

32. A. carrying    B. gathering    C. organizing   D. training

33. A. students    B. teachers     C. patients    D. soldiers

34. A. passing    B. blocking     C. waiting     D. attacking

35. A. took      B. brought     C. led       D. sent

36. A. killed     B. saved      C. surprised    D. frightened

37. A. beat      B. knocked     C. hurt      D. damaged

38. A. hurried    B. worried     C. suffered    D. lucky

39. A. warm      B. cold       C. silent     D. deep

40. A. planning    B. thinking     C. searching    D. expecting

41. A. held      B. found      C. saved      D. helped

42. A. helping    B. seeing      C. round      D. below

43. A. sinking    B. swimming     C. fighting    D. escaping

44. A. explosion   B. noise      C. laughter    D. shout

45. A. away      B. off       C. far       D. distance

46. A. dived     B. disappeared   C. drowned     D. fell

47. A. back      B. on        C. up       D. over

48. A. Here      B. There      C. Where      D. It

49. A. short     B. loud       C. dull      D. terrible

50. A. jumped     B. swam       C. flew      D. floated

第三部分:阅读理解 (共20 小题,每小题2.5 分,满分50 分)

   A

It costs thousands of dollars to buy or build a new house. A family who wants to own a house usually borrows most of the money needed. The money is paid back in monthly payments. The family must pay the rest of buying price in cash (现金). This is called the down payment.

In most cities, there are many families who need and want new houses. Some of these families can afford to make monthly payments on a house. But they cannot save up enough money to make a down payment. How then can these families become houseowners?

In the city of Indianapolis, an organization under black leadership runs a program of self-help housing called Flanner House Homes. This program has helped hundreds of families with low income become houseowners.

Flanner House Homes supplies the land, materials, tools, and training for a large group of families to build their own houses. These families do not have money for a down payment on a house. Instead, the men used their own labor as a down payment. They work twenty hours a week to help build their houses. Although they have never built houses before, the men are taught certain skills as they work. They work together as teams.

When the houses are finished, the new houseowners have some more than built themselves new houses. With handsome brick and frame (框架) houses taking the place of old, run-down ones, they have turned the streets with small, dirty, crowded houses into pleasant new neighborhoods.

51. Why Flanner House Homes was founded is because        .

  A. the black people needed new houseowners

  B. it helps low-income families become houseowners

  C. many low -income people have difficuty in getting the land, materials and tools

  D. it was easier for an organization to get the supplies

52. From the text we can conclude that         .

  A. many people do not want to own their houses

  B. in most cities everyone owns his own house

  C. Flanner House Homes helps to build houses

  D. a good organization takes great effect (影响) on solving problem

53. Which statement does this text lead you to believe?

  A. A down payment is not always money.

  B. It does not cost much to own a new house.

  C. It is easy to pay for a house.

  D. Flanner House Homes supplies people with the money to buy houses.

B 

   As we approach the 21st century, it is clearer than ever that science and technology are changing the way we live and work. The breakthroughs in bioengineering science are helping to reveal the mysteries of life, holding out new hope for lifesaving cures to some of our most dreaded diseases.

   In 1997, the great news shocked the whole world that the British scientists created a lamb, named Dolly from a single cell. Miraculously, the cell had been taken from the udder of an adult sheep, which marks the first time an adult animal had been successfully cloned.

   But it soon caused worldwide concern over its ethical, social, and scientific implications. Cloning is especially controversial because it raises the prospect of "making" humans genetically identical to an existing man or woman. Some people claim attempting to use these cloning techniques to actually clone a human being is untested and unsafe and morally unacceptable.

   How to look upon this new technique? When it comes to a discovery like cloning, we must move with caution, care and deep concern about the impact of our actions. First, cloning should be greatly esteemed and great thanks should be given to its inventors. Meanwhile, great efforts should be made to place this technique under strict control. Especially we should adopt a document against human cloning since past experience tells us a new invention may cause troubles if it is misused.

54.What are changing the way we live and work?  

A.Cloning.               B.Life saving.

C.Science and technology.        D.The mysteries of life.

55.Some people claim attempting to use these cloning techniques to actually clone a human being is   

  ________.

A.untested               B.unsafe

C.morally unacceptable                  D.above of all

56.How should we look upon cloning?

A.Cloning is terrible.

B.Cloning must be moved with caution, care.

C.Cloning will cause troubles.

D.Cloning is a disease.

57.According to the passage, what should be given to cloning's inventors?

A.Strict control.           B.Great thanks.

C.Care.                D.Caution.

C

The biggest change in the world today is technology. The reason the world is getting smaller is because people can talk to each other more quickly than before with more information.

The computer now has the information of a library of books on all subjects. By sitting at a computer, different software programs allow students the chance to learn about all subjects simply by asking the computer to find the information. The computer becomes the library.Students and parents are now learning what an important tool the computer has become for improving education.

Letters might take a few days even a few weeks to be delivered. Technology has developed a fax machine where a letter can travel anywhere in the world, immediately. This has changed the way people do business. Many computers can receive E-mail. This is like the fax letter. Often, businesses have a fax machine but homes do not. If a computer is in a home, it can receive E-mail messages from anyone in the world. It is like the telephone but it costs much less to receive messages.

The next generation (代) will have computers in most homes and be able to send faxes and E-mail all over the world. In the future, the computer will be as common as the telephone or TV. Technology is changing the way people communicate with each other.

58. The world is getting smaller just because        .

  A. the computer has taken the place of a library

  B. people can communicate with each other much more quickly

  C. people have a shorter talk to each other

  D. people in different places can get together in a short time

59. More and more students find the computer       .

  A. can help them with everything

  B. has become an important tool

  C. can be used for getting more information in a shorter time

  D. can help them go to college

60. Which is most emphasized (强调) in the passage to show how important technology is?

  A. Telephone   B. Fax       C. E-mail     D. Computer

61. We can infer from the passage that       .

  A. the telephone is more common than the computer now

  B. letters won't be found in the future

  C. E-mail is the cheapest way to receive messages

  D. business people had better use fax machine

62. What's the main idea of the passage?

  A. The world is smaller.                  B. Technology changes the world a lot. 

  C. The computer will be widely used.        D. People can communicate with each other easily.

D

Dear Carrie,

Your brother sounds like a very selfish(自私的) person. But I want to know what causes him to act this way. Since he seems to be doing this on purpose, maybe the only way to do with the problem is to try to find the cause.

I think your brother is probably hurting in some way. Perhaps he feels unsafe because he is not a full part of the family. He could be jealous (嫉妒) of your mother's love for the rest of you over him.

It is wrong to fight anger with anger though you think it is a need to help your mother. I suggest you try to help your brother by treating him with more care and love. It must be hard for him to grow up in a family where his sisters had their real mother but he didn't. Try to let him know that you care about him. Over time he may come to know how lucky he is and start to treat you all with the same kindness that you give him.

63. From the passage we may know that        .

  A. there are two mothers in the family

  B. the mother is the brother's stepmother (继母)

  C. the family is not a full family

  D. the brother has three sisters in the family

64. The writer suggests Carrie that        .

  A. she treat her brother with more love and care

  B. she find her mother with more fights

  C. she find the cause for her brother's habit

  D. she fight against her brother with anger

65. The writer thinks Carrie's brother       .

  A. should know Carrie treats him with kindness

  B. should be treated more kindly

  C. is very lucky to live in such a family

  D. is a bad young man

66. In Carrie's letter she must have said that       .

  A. she often punished her brother

  B. she often gave her brother love

  C. she often felt sad and angry in the family

  D. her mother wasn't a good mother

E  Gene technology to benefit people

Among all the fast growing science and technology, the research of human genes, or biological engineering as people call it, is drawing more and more attention now. Sometimes it is a hot topic discussed by people.

The greatest thing that gene technology can do is to cure serious diseases that doctors at present can almost do nothing with, such as cancer and heart disease. Every year, millions of people are murdered by these two killers. And to date, doctors have not found an effective way to cure them. But if the gene technology is applied, not only these two diseases can be cured completely, bringing happiness and more living days to the patients, but also the great amount of money people spend on curing their diseases can be saved, therefore it benefits the economy as well. In addition, human life span(寿命) can be prolonged.

Gene technology can help people to give birth to more healthy and clever children. Some families, with the English imperial family being a good example, have hereditary diseases. This means their children will for sure have the family disease, which is a great trouble for these families. In the past, doctors could do nothing about hereditary diseases. But gene technology can solve this problem perfectly. The scientist just need to find the wrong gene and correct it, and a healthy child will be born.

Some people are worrying that the gene research can be used to manufacture human beings in large quantities. In the past few years, scientists have succeeded in cloning a sheep, therefore these people predict that human babies would soon be cloned. But I believe cloned babies will not come out in large quantities, for most couples in the world can have babies in very normal way. Of course, the

governments must take care to control gene technology.   (http://www.rothamsted.bbsrc.ac.uk)

67. What does "these two killers" in the second paragraph refer to? 

A.gene technology and another treatment of the two diseases.

B.The two murderers who killed the cloned baby

C.The two diseases of cancer and heart disease

D.Hereditary diseases and cancer

68.What's the main idea of the third paragraph?

A.How gene technology can be applied in the field of treating hereditary diseases.

B.Gene technology can be used to clone human babies.

C.Gene technology can help people to give birth of a baby.

D.Gene technology can help the English imperial family out

69.In what way gene technology can help to treat hereditary diseases?

A.Using gene technology, people with hereditary diseases can have more living days.

B.Using gene technology, scientist finds the wrong gene and corrects it.

C.Using gene technology, human babies can be cloned.

D.Doctors can cure cancer and heart disease with the help of gene technology.

70.What is the main purpose of writing this passage?

A.Expressing the writer's idea that gene technology will benefit people

B.Telling people the advantages of gene technology

C.Telling the readers that gene technology will not benefit people

D. Explaining that gene technology will also do harm to the humanity

第四部分:写作 (共两节,满分40分)

第一节:短文改错  (共10 小题,每小题1 分,满分10 分)

 A man works in a factory and get his pay at           71.         

the end of week. One Saturday he got his pay             72.         

then counted the money. Suddenly he found that            73.         

it was wrong:there were five pounds much than            74.         

he should get. He had put the money in his              75.         

pocket and said nothing with others. A week             76.         

went by and it was pay day too. But this time            77.         

there were five pounds less instead of. He was            78.         

so angry and he went to see the boss. The boss            79.         

said to him. “Didn't you notice there was a              80.        

mistake last week? For one mistake, I can shut

my eyes, but for two mistakes, no, I can't.”

第二节:书面表达 (满分30 分)

下面是几幅移栽花草的示意图,请你根据画面所示内容,用英语写一篇100词左右的短文,概括叙述一下移栽的过程。

中 学 英 语 能 力                               

训练与测试系列    NSEFC- IIB-UNIT-T- 2005

             高中二年级单元目标测试卷

Unit 12 Fact and fantasy

第一节:单元要点 (共15 小题,每小题1分,满分15 分 

1.He narrowly escaped______in the fighting.

  A.to kill     B.to be killed   C.killing     D.being killed

2.-Why do people prefer to take a window seat on the plane?

  -Because the window_____the outside world.

  A.takes a view of B.keeps a view of C.has a view of   D.gives a view of

3.The weather report says it will_____cold tomorrow.

  A.turn up     B.turn out     C.turn on     D.turn in

4.She drew the______before switching on the light.

  A.shades      B.masks      C.coverings    D.curtains

5.-It's______that he said:"I am______to help you."

  -Don't take it seriously.He is always playing that trick.

  A.sure;sure    B.certain;certain C.sure;certain   D.certainly;curely

6.-How much is the rent of the house,Mrs Sandra?

  -One hundred dollars,the electricity and telephone fee_______.

  A.containe     B.containing    C.including    D.included

7.Please find some books that______these  matters.

  A.light upon    B.give light upon C.throw light upon D.throws light upon

8.A lot of people here,______,John,would rather have coffee.

  A.for example   B.for instance   C.namely      D.such as

9.My brother worked all summer vacation,saving money to_____his hobby of photography.(03SH)

  A.seek       B.search      C.hunt       D.pursue

10.What is the______between Nangjing and Shanghai?

  A.distance    B.length      C.width      D.height

11.The boy listens to the teacher______,so everything seems easy to him.

  A.attention    B.attentively   C.attentive    D.attend

12.A man who hesitates______everything cannot do anything successfully.

  A.to do      B.doing      C.for doing    D.in doing

13.He observed her______across the street.

  A.to come     B.comes      C.coming      D.came

14.I______what you said but I don't______you.

  A.believe;believe           B.believe in;believe         

   C.believe in;believe in                        D.believe;believe in

15.It is so nice to hear from her.______,we last met more than thirty years ago.(2002 Beijing)

   A.What's more       B.That is to say      C.In other wordsd.    D.Believe it or not

第二节. 专题语法练习 (构词法2) (15分)

16.There were________fish in the river in South America.

   A.in danger      B.danger        C.dangerous      D.dangerless

17.The child looked at me________.

   A.stranger      B.strangely       C.strange        D.strangeless

18.What you said sounded________ but in fact it was untrue.

   A.reasonable     B.reasonful       C.reasonless      D.unreason

19.We have to learn________technology from other countries.

   A.advance      B.advancing      C.advantage      D.advanced

20.Mr Black is an ________in the army,not an________in the government.You can not easily find

   him in his________.

   A.official;officer;office             B.officer;office;official

   C.official;official;official             D.officer;official;office

21.________ speaking,I didn't do it on purpose.

   A.Honestly      B.Honest        C.Honesty       D.Dishonest

22.It sounds like a good plan,but there are some________difficulties in carrying it out.

   A.practiced      B.practical       C.practice       D.practicing

23.His father possesses a________factory,which does most of the pollution to this river.

   A.paper-make    B.paper-making    C.papers-made    D.paper-made

24.Marx left his homeland for some________reasons.

   A.politically     B.politics        C.political       D.politician

25.It's________to persuade him to give up smoking.He's very stubborn.

   A.possible       B.possibly        C.impossible      D.impossibility

26.Dan caught two________birds in the wood last week and they are still________in the cage.

   A.alive;live      B.live;live        C.live;alive       D.alive;alive

27.Ann felt so________that she could hardly open her eyes.

   A.sleepy        B.asleep         C.sleep         D.sleeping

28.We stood there________at the________sight.

   A.frightened;frightful                           B.frightening;frightful

   C.fright;frightening               D.frightful;fright

29.The doctor's advice________him from drinking and smoking.

   A.encouraged    B.couraged       C.encouragement  D.discouraged

30.I'd like to buy a house,modern,comfortable and above all in a quiet ________.

   A.neighbour      B.neighbourhood    C.neighbours     D.neighbour's

第三节. 完形填空 。30分    

One evening Robert was on his way home from the railway station. When he turned round a  31   , he heard footsteps behind him and he thought  32  was coming near. He began to walk  33  . The footsteps became faster, too. He slowed down. The footsteps also  34  down. Now he was   35  that he was being followed. He tried to hide.  36  the steps followed him. He didn't know  37  to save himself, so he jumped over a fence and found   38  in a cemetery, throwing himself down on one of the tombs.   

The man  39  followed. Robert could hear the man jump over   40  . Thoughts of thieves or murderers filled his   41  .Robert stood up and   42  the man who was 43  him. "What do you want?" he said."   44  are you following me?"

 "I say," asked the stranger. "Do you   45  go home like this, 46  are you taking some special exercises   47  ? I want to go to Mr. Brown's and don't know   48   . The station master told me to follow you as you  49  next door.   50  me for asking, but is there much to go before you get home?"

31. A. corner              B. park       C. street       D. way

32. A. no one              B. someone     C. anyone       D. everyone

33. A. slowly              B. more slowly   C. fast        D. faster

34. A. put                B. slowed      C. became       D. turned

35. A. sorry               B. glad       C. afraid       D. sure

36. A. Still               B. Sometimes    C. Often        D. Seldom

37. A. when               B. where       C. why         D. how

38. A. him                B. himself     C. the man       D. the thief

39. A. ahead               B. behind      C. nearby       D. beside

40. A. a bridge             B. a wall      C. the yard      D. the fence

41. A. head               B. idea       C. mind        D. brains

42. A. faced               B. beat       C. hit         D. told

43. A. stealing             B. robbing     C. following      D. meeting

44. A. What               B. Why        C. How         D. When

45. A. never               B. always      C. hardly       D. seldom

46. A. and                B. or         C. so         D. but

47. A. every night           B. at night     C. at the night    D. tonight

48. A. him               B. the house     C. the way           D. the partner

49. A. work               B. play       C. study        D. live

50. A. Answer              B. Show       C. Tell        D. Excuse

第三部分:阅读理解 (共20 小题,每小题2.5 分,满分50 分)

   A

 ZHUHAI-China will soon put a person into space and become the third nation in the world to have manned spaceflight,a top Chinese aerospace(航空航天)official said on November 5.

A manned launch was not far off,Vice President Hu Hongfu of China Aerospace Science Technology Corp(CASTC)told a news conference at the third Zhuhai Air Show. "It will not be long before Chinese astronauts can ride locally-make spaceships into space,"Hu said.The official gave no timerable but said it would happen "at the beginning of the 21st century."

The former Soviet Union and the United States have been putting people into orbit since the early 1960s,but other nations have not considered the challenge worth following.

China has already built and sent up its own satellites for communications(通讯) and weather forecasting.The new communication satellites would help to encourage the country's broadcasting industry,Hu said.

But the company,which includes over 130 aerospace agencies,said the lifting ability and success rate of China's space-launches were almost equal in quality to those of other countries.

China would gradually close the distance between its country's space industry and that of other nations.He added that they were willing to send up commercial satellites for Taiwan.

On China's plan to send up manned spaceflights,Hu said China had made important development after the successful launch(发射)in November last year of the country's first experimental spaceship,Shenzhou. "The whole project is in the research stage and a lot of work needs to be done," Hu said. "We need to have more tests of the unmanned spaceship."

China last year announced a four-step manned spaceflight plan,with the aim of setting up a space-station served by a spaceship traveling between two places.

51.The underlined word "they" here probably refers to____________.

A.the Soviet Union and the USA             B.over 130 aerospace agencies

C.Russia and China                   D.Russia and the USA

52.The main idea of the 7th paragraph is______________.

A.we haven't tested the manned spaceship

B.before we send a manned spaceship,we should carry out many tests

C.though we tried once,we need to do more tests of the unmanned spaceship

D.sending up a manned spaceship can show a country's scientific level

53.Which of the following is closest in meaning to the underlined word "close"?

A.shut                B.drop          C.shorten          D.widen

54.The writer wrote the passage to____________.

A.tell us the present situation of China's aerospace technology

B.remind us what we should do compared to other countries

C.find out the reason why China falls behind

D.tell us that China will send up manned spaceship soon

B

In the year 2000, the world is going to have a population of about 8 billion (十亿). Most scientists agree that the most severe problem is food supply.

Who is going to feed all those people? Where is the food going to come from? Are we going to have enough food? Are we going to produce more artificial(人选的) food?

One way of improving the situation is for people to eat less meat. Why? Because it takes 4 kilos(公斤) of grain protein(蛋白质) to produce half a kilo of meat protein. Clearly, there is not going to be sufficient meat protein for 8 billion people. Therefore, it will also necesssary to change eating habits because meat is the main part of many people's food today.

A possible solution to this latter problem is the soybean (大豆).The soybean plant produces beans which have a very high fat and protein content. Scientists can now make these look and taste like real meat. They can also make many other artificial products such as soybean milk, for example, which has a taste of milk and can be used in cooking in very much the same ways as cow's milk. In fact, one woman in the United States fed her family only on soybeans for years! She gave them soybean beef, soybean chicken, soybean milk, and sometimes just soybeans. Possibly, we are all going to eat soybeans in the future and finally give up meat completely from our tables.

55.What is the main subject of the passage?                    

   A. A solution to man's food problem.

   B. A solution to the population problem.

   C. Advantages of soybean.

   D. How to develop good eating habits.

56.According to the passage, meat will completely disappear in the future because        .

   A. people have to spend too much time energy to produce it

   B. too much grain protein is needed to produce it

   C. it contains too much fat and protein

   D. it is no good to the health

57.In this passage, “artificial food” refers to food made from         .

   A. milk              B. grain              C. protein            D. soybeans

58.Soybean meat is similar to real meat        .

   A. in appearance but not in taste.

   B. neither in appearance nor in taste.

   C. both in appearance and in taste.

   D. in taste, but not in appearance.

C

For years scientists have been worried about the effects of air pollution on the earth's natural conditions. Some believe the air inside many houses may be more dangerous than the air outside.

Most scientists agree that every modern house has some kind of indoor pollution. People began to notice the problem in the early 1970s. When builders began making houses and offices, they did not want to waste energy. To do this they built buildings that limited the flow of air between inside and outside. They also used man-made building materials which are now known to let out harmful gases.

As the problem became more serious, scientists began searching for a way to deal with it. They discovered a natural pollution control system for building green plants.

Scientists believe that a plant's leaves absorb the pollutants (污染物). In exchange the plant lets out oxygen through its leaves. Studies of different plants show that each absorbs different chemicals. So the most effective way to clean the air is to use different kinds of plants.

59. Indoor air pollution may be more dangerous than the air outside mainly because        .

   A. the flow of air indoor is limited        B. the building are too high

   C. people don't want to waste energy        D. the building materials are harmful

60. Some kind of indoor pollution can be found in        .

   A. all kinds of offices              B. all kinds of houses

   C. every modern house      D. every old house

61. In the last paragraph the word“ absorb” may mean        .

   A. give out  B. bring up    C. take in    D. turn into

62. Scientists believe that plants can       .

   A. absorb pollutions and let out oxygen      B. make your houses more prettier

   C. absorb all the pollutants   D. change pollutants into oxygen

63. According to the last paragraph, we should        to clean the air.

   A. plant all kinds of plant   B. have different kinds of plants

   C. have the same kind of plant  D. plant more trees.

D

Beauty has always been regarded as something praiseworthy.Almost everyone thinks attractive people are happier and healthier,have better marriage and have more respectable jobs.Personal advisors give them better advice for finding jobs.Even judges are softer on attractive defendants.But in the executive(主管的)circle,beauty can become a liability.

While attractiveness is a positive factor for a man on his way up the executive ladder,it is harmful to a woman.

Handsome male executives were considered having more honesty than plainer men;effort and ability were thought to lead to their success.

Attractive female executives were considered to have less honesty than unattractive ones;their success was connected not with ability but with factors such as luck.

All unattractive women executives were thought to have more honesty and to be more capable than the attractive female executives.Interestingly,though,the rise of the unattractive overnight successes was connected more to personal relationships and less to ability than that the attractive overnight successes.

Why are attractive women not thought to be able?An attractive woman who is considered to be more feminine(女子气的)has an advantage in traditionally female jobs,but an attractive woman in a traditionally manly position appears to lack the "manly" qualities required.

This is true even in politics. "When the only clue(线索) is how he or she looks,people treat men and women differently",says Anne Bowman,who recently published a study on the effects of attractiveness on political candidates(候选人).She asked 125 undergraduate students to rank two groups of photographs,one of men and one of women,in order of attractiveness.The students were told the photographs were of candidates for political offices.They were asked to rank them again,in order they would vote for them.

The results showed that attractive males completely defeated unattractive men,but the women who had ranked most attractive unchangeably received the fewest votes.

64.The word "liability" most probably means________.

A.disadvantage      B.advantage      C.misfortune      D.trouble

65.In traditionally female jobs,attractiveness________.

A.makes women look more honest and capable

B.strengthens the feminine qualities required

C.is of great importance to women

D.often enables women to succeed quickly

66.Bowman's experiment shows that when it comes to politics,attractiveness_______.

A.turns out to be a disadvantage to men

B.is more of a disadvantage than an advantage to women

C.affects men and women alike

D.has as little effect on men as on women

67.The author writes this passage to________.

A.give advice to job-seekers who are attractive

B.discuss the disadvantages of being attractive

C.demand equal rights for women

D.stats the importance of appearance

E

1970 was World Conservation (自然保护) Year. The United Nations wanted everyone to know that the world is in danger. They hoped that governments would act quickly in order to “conserve”nature. Here is one example of the problem. At one time there were 1, 300 different plants, trees and flowers in Holland, but now only 866 remain. The others have been destroyed by modern man and his technology. We are changing the earth, the air and water, and everything that grows and lives. We can't live without these things. If we continue like this, we shall destroy ourselves.

What will happen in the future? Perhaps it is more important to ask“What must we do now?”The people who will be living in the world of tomorrow are the young of today. A lot of them know that conservation is necessary. Many are helping to save our world. They plant trees, build bridges across rivers in forests, and so on. In a small town in the United States a large group of girls cleaned the banks of 11 kilometers of their river. Young people may hear about conservation through a record called“No one's going to change our world”. It was made by the Beatles, Cliff Richard and other singers. The money from it will help to conserve wild animals.

68.“No one's going to change our world”was        .

A.an important book published in 1970

B.an idea that nobody would accept

C.a record calling on people to conserve nature

D.a rule worked out by the United Nations

69.What is the most important thing for us to do to save our world?

A.We should plant more trees and flowers.

B.We should clean the banks of our rivers.

C.We should know what will happen in the future.

D.We should know what we must do and begin to do it now.                       

70.What's the main idea of the passage?

A.1970 was World Conservation Year.

B.The United Nations wanted everyone to know that the world is in danger.

C.Conservation is necessary.

D.It is the young people who are helping to save our world.

Ⅴ. 短文改错 。 (10分)

Dear Chris,

    I want to tell you little about London before you arrive.                    71.           

Which you know, it's the capital of England and it's on the              72.          

River Thames. It's a very big city of a population of about              73.          

7 million. The people are usually friendful and helpful but              74.          

often in a hurry!

   London is a busy place. People work in offices, banks and            75.           

for companies. The city has been got many parks and gardens.              76.          

When you are here, you must look the London Museum. You can              77.          

visit the National Gallery and the Science Museum. You also              78.          

see the changing of the Guards at Buckingham Palace (白金汉宫).            79.          

    We are all looking forward to see you.                     80.           

Ⅵ. 书面表达 。30分

     根据提示, 用英语介绍爱因斯坦生平,要求写 100-120字。

    ①爱因斯坦出生时间、地点。

   ②他从小喜欢问问题,到十四岁时就自学完了高等数学,那时就决心成为物理学家,将  自己的一生献给科学事业。

   ③爱因斯坦家庭并不富,但他父母亲还是设法送他进了一所技术学校,后来又送 到瑞士  的联邦工学院。

   ④ 1905年获博士学位,就在那时,他开始研究工作,导致了著名的相对论著作的产生。

中 学 英 语 能 力

训练与测试系列   NSEFC- IIB-UNIT-T- 2005         

          高中二年级单元目标测试卷

Unit 13  The water planet

第一节:单元要点 (共15 小题,每小题1 分,满分15 分) 

1.Temperature here______10°C to 30°C.

  A.comes from    B.changes from   C.turns from    D.ranges from

2.The water in oceans also keeps the temperature of the earth steady _____heat.

  A.to absorb and release                 B.by absorbing and reseasing

  C.without absorbing and releasing  D.in absorbing and releasing

3.The old lady is_____to what others say.

  A.stable     B.available   C.sensible    D.sensitive

4.Oil has the_____of floating on water.

  A.property     B.medium      C.disadvantage   D.capacity

5._____I know the money is safe,I shall not worry about it.(2003 Beijing)

   A.Even though  B.Unless      C.As long as    D.While

6.I'm sorry,sir,those shoes are not______in your size.

  A.dense      B.pure      C.available   D.solid

7.The good policy_____a lot to the economic development.

  A.gives      B.contributes   C.cause      D.benifits

8.Because the company was doing more business,it was necessary to______the factory.

  A.extend     B.increase     C.rise       D.make wide

9.The ______river water goes 350 kilometers out to sea.

  A.pure      B.new       C.sweet      D.fresh

10.A left-luggage office is a place where bags________be left for a short time,especially at a railway

   station.(2003 NMET)

  A.should      B.can       C.must       D.will

11.Six players can_____a volleyball team while a football team is______eleven players.

  A.make up;made up           B.make up of;made up

  C.make up;made up of                  D.make up of;made up of

12.His coming______our happiness.

  A.added      B.added up     C.added to     D.added for

13.They say your report doesn't______the fact.But I don't ______them_____this point.

  A.agree to;agree with;at       B.agree on;agree to;on

  C.agree on;agree on;on                 D.agree with;agree with;on

14.-There's coffee and tea;you can have______.

  -Thanks.(2003 NMET)

  A.either      B.each       C.one       D.it

15.What surprised me was not what he said but_____he said it.(2004 Hubei)

  A.the way     B.in the way that C.in the way    D.the way which

第二节. 专题语法练习 (情态动词) 。(15分)

16.When he was there,he ____ go to that coffee shop at the corner after work every day.

  A.would      B.should      C.had better    D.might

17.Sir,you ____ be sitting in this waiting room.It is for women and children only.

  A.oughtn't to   B.can't      C.won't      D.needn't

18.The fire spread through the hotel very quickly but everyone ____ get out.

  A.had to      B.would      C.was able to   D.could

19.there was a lot of fun at yesterday's party.You ____ come,but why didn't you?

  A.must have    B.should      C.need have    D.ought to have

20.Sorry,I'm later.I ____ have turned off the alarm clock and gone back to sleep again.

  A.might      B.should      C.can       D.will

21.My sister met him at the Grand Theatre yesterday afternoon,so he ____ your lecture.

  A.couldn't have attended       B.needn't have attended

  C.mustn't have attended        D.shouldn't have attended

22.I was really anxious about you.You ____ home without a word.

  A.mustn't leave            B.shouldn't have left

  C.couldn't have left                  D.needn't leave

23.Mr Bush is on time for everything.How ____ it be that he was late for the opening ceremony?

  A.can       B.should      C.may       D.must

24.It has been announced that candidates ____ remain in their seats until all the papers have been   

  collected.

  A.can       B.will       C.may       D.shall

25.Oh,I'm not feeling well in the stomach.I ____ so much fried chicken just now.

  A.shouldn't eatt           B.mustn't have eaten

  C.shouldn't have eaten                 D.mustn't eat

26.-When can I come for the photos? I need them tomorrow afternoon.

  -They ____ be ready by 12:00.              

  A. can       B. should     C. might      D. need

27.-Are you going off to Jeff's party?

  -I'm not sure. I ____ go to the concert instead.

  A. must      B. would      C. should     D. might    

28.There was plenty of time. She ____ have hurried.

  A. mustn't     B. couldn't    C. needn't     D. wouldn't      

29.Johnny, you ____ play with the knife, you ____ hurt yourself.

  A. won't; can't            B. mustn't; may   

  C. shouldn't; must          D. can't; shouldn't

30.-It's many years since I last saw you. I didn't recognize you at first.

  -I ____ either, if someone hadn't called you by name.

  A. wouldn't have  B. wouldn't    C. didn't     D. hadn't

第三节. 完型填空 。(30分)

Dorothea Shaw is 71 years old and nearly blind, and she chooses to live alone far away from people in Blize. Her home is in a small village which can be  31  only by sea or air. After a 10-mile walk into the hills, one   32  reaches a piece of land and two small houses so  33   in the thick over-grown forest that only a   34  of people know Dorothea is there.

She lives  35   and totally alone  36   vegetables. Sometimes a local(当地的) man will come and cut   37  for her and a group of soldiers will come   38    her and be greeted with the   39  of a cup of coffee.

At night, she lies in her little room with the dogs and cats on the floors and  40  for hours to any Spanish, English,German or French  41  she can find on her 42  .Sometimes she gets lonely but   43  of the time the animals and the radio are company(陪伴;伙伴) enough.

But recently the very things she had  44   to get free from so well have begun to  45  her.The peace of the forest has been  46  by the noise of earth-moving machines. What she once only heard of  47  on the radio is now on her  48  .Things began to change 3 years ago. The new main north-south road in Blize has  49  the forest only four or five miles away."Now  50  people know I'm here,"she says,"I feel more and more uneasy(troubled or anxious) each day."

31.A.arrived               B.seen        C.reached        D.known

32.A.finally               B.later       C.firstly        D.surely

33.A.hunted                B.built        C.hidden       D.searched

34.A.handful               B.lot        C.deal          D.number

35.A.happily               B.sadly       C.hardly         D.disappointedly

36.A.grew                B.grown       C.to grow        D.growing

37.A.meat      B.vegetable     C.road       D.wood

38.A.towards     B.across       C.upon       D.off

39.A.offer      B.help        C.money      D.admiration(钦佩)

40.A.listens    B.waits        C.looks       D.asks

41.A.TV plays    B.broadcasts    C.films       D.pictures

42.A.radio      B.wall        C.screen      D.table

43.A.all       B.little      C.most       D.least

44.A.failed     B.tried       C.started     D.decided

45.A.catch up with     B.run away from         C.get rid of          D.take care of

46.A.left      B.destroyed     C.made        D.discovered

47.A.carefully    B.hardly       C.uncertainly    D.distantly

48.A.TV set     B.roof        C.doorstep     D.table

49.A.cut into    B.cut up       C.cut through   D.cut off

50.A.less and less      B.more or less         C.fewer and fewer    D.more and more

第三部分:阅读理解 (共20 小题,每小题2.5 分,满分50 分)

A

   More ice melted from the surface of the Greenland Ice Sheet this year than ever before recorded, report scientists from the University of Colorado. The same team found that the extent of Arctic sea ice reached the lowest level in the satellite record in 2002, offering further proof that climate change is already changing the Arctic.

   Researchers from the University of Colorado(UC)say that the speeding up melting appears to be linked to changes in Northern Hemisphere atmospheric circulation patterns. The study also found temperatures during the summer of 2002 were warmer than usual over much of the Arctic Ocean.

  "It is likely that sea ice extent will continue to decline over the 21st century as the climate warms," Researchers added. "With these trends, we may see a 20 percent reduction in the yearly average sea ice by 2050, and by then there might be no ice at all during the summer months."

  The report was released on Saturday at the yearly fall meeting of the American Geophysical Union, just one week after an announcement by the National Aeronautics and Space Administration(NASA) that sea ice is melting about nine percent faster than earlier research had showed, and could disappear entirely by the end of this century.

   "This trend of disappearing arctic sea ice is one example of the environmental damage that can be linked to carbon dioxide emissions," said Morgan, "When we have the means to reduce CO2 emissions and prevent further damage, we should do our best. National leaders must act now to improve energy efficiency and increase the use of renewable energy sources, like wind and solar, before it's too late."

   CU scientists judge that a change in the Greenland climate toward warmer conditions would lead to an increase in the rate of sea level rise, which will bring disasters to the human beings.

51. It can be inferred that ________.

A. scientists have found that Arctic sea ice is lower than before

B. scientists have studied the pictures of the Arctic sent by satellites

C. scientists began to do research on Arctic sea ice in year 2002

D. warm temperature has hardly any effect on the extent of Arctic sea ice now but soon will

52. Scientists fear that __________.

A. there will be no ice left at all in the Arctic Ocean at the end of the 21st century

B. disasters will happen if Arctic sea ice continues to melt faster than before

C. the temperature in Greenland will be too warm for animals and human beings to live on

D. people still think Arctic sea ice is melting slowly

53. It is thought that _________.

A. carbon dioxide emissions are one of the causes of the environmental damage

B. carbon dioxide emissions are an example of the environmental damage

C. most environmental problems are caused by carbon dioxide emissions

D. people can still have the change to solve the above environmental problem

B

Gary Finkle was seriously hurt in a swimming-pool accident seven years ago. A heavy-set, bearded man of 27, he is one of thousands of Americans who have lost actually all feeling and movment from their shoulders down. He lives with his wife, Micky, and a female monkey named Jo outside the village of Andes, New York.

Gary takes part in an unusual organization called Helping Hands:Simian Aides(猴子助手)for the disabled.The organization supplies the disabled with trained monkeys that reduce the disabled person's dependence on family, friends and hired servants.

Using his mouth, Gary controls a small laser(激光)pointer fixed on his wheel-chair. With it, he directs Jo to change books or the tape-recorder. She brings him drinks from a refrigerator(冰箱)and clears away empty containers.

When asked, Jo will fetch the remote control(遥控器)for the TV and place it on Gery's working table where he can operate it with his mouthstick. This mouthstick can be used for practically everything:turning the pages of a book, dialing the telephone, changing channels on TV, working at a typewriter or computer. If Gary's mouthstick drops to the floor, Jo will pick it up and gently put it again into his mouth.

"I can't imagine living without her ,"Gary says. He will always need human assistance for such things as getting in and out of bed, bathing or changing his clothes. But having Jo reduces his dependence on Micky, making it possible for her to do things in town without worrying about her husband's safety.

54.Gary who was disabled can't       .

  A.stand up to shake hands                 B.greet guests

  C.admire the things outside               D.turn his head

55.Before Jo came to the family, Gary          .

  A.depended completely on his servant

  B.depended mainly on an assisting machine

  C.depended on the pity of other people

  D.depended completely on his wife

56.The underlinded word"her"in the last paragraph refers to        .

  A.Gary       B.Micky      C.Jo        D.Finkle

57.This text is mainly about       .

  A.why Gary and Jo are good friends

  B.Gary's accident as a swimmer

  C.Gary's life and experience

  D.trained monkeys that can help the disabled     

C

Waves are beautiful to look at, but they can destroy ships at sea and houses near the shore (海岸). Most waves are caused by winds blowing over the surface of the water. The sun heats the earth, causing the air to rise and the winds to blow. The winds blow across the sea, pushing little waves into bigger and bigger ones.

The size of wave depends on how strong the winds is, how long it blows, and how large the body of water is. In a small bay (海湾) big waves will never build up. But at sea the wind can build up big and strong waves.

A rule says that the height (高度) of a wave (in metres) will usually be no more than one-tenth of the wind's speed (in kilometres). In other words, when the wind is blowing at 120 kilometeres an hour, most waves will be about twelve metres. Of course, some waves may work together to form very big waves that are much higher. In 1933 the United States Navy (海军) reported the largest measured wave in history. It rose in the Pacific Ocean to a height of thirty-four metres.

58. Most waves are caused by       .

   A. the earth's size                B. the sun heating the earth

   C. the surface of the water   D. the winds blowing across the sea

59. When the winds blow across the sea     .

   A. little waves are made bigger and bigger

   B. the air rises above the surface of the water

   C. the heat of the sun is reduced

   D. the body of water becomes larger

60. The size of a wave depends on all of the following EXCEPT       .

   A. how large the body of water is

    B. how hot the body of water is

   C. how long the wind blows

   D. how strong the wind is

61. If the wind is blowing at 30 kilometres an hour, the height of a wave is about      .

   A. twelve metres         B. one-tenth metres C. 120 kilometres   D. three metres

D

    Scientists have uncovered the exact difference between man and mouse.

   Teams of scientists from around the world will publish today the draft genome of the mouse, which enables them for the first time to compare another mammal's entire DNA directly with that of humans.

   The similarities are far greater than the differences—man and mouse share a common ancestor that lived more than 70 million years ago - which has allowed scientists to find treatments for a good many human diseases.

   British members of the project say the draft mouse genome, published in the journal Nature, has provided the "phrasebook" for scientists to translate the language of the human genome.

   "The entire biomedical research community can for the first time fully use this resource to deal with human disease. They will now have powerful tools that will serve them for many years to come," said Dr Jane Rogers, head of sequencing at the Wellcome Trust Sanger Institute in Cambridge, which did 20 percent of the work.

   Comparisons show almost every gene in the mouse has a corresponding gene in humans - a 99 per cent similarity that should help to find how each human gene works. Already the work has been able to show that man and mouse each have about 30,000 genes yet only 300 or so are uniqueto either species, increasing the value of the lab mouse as a "model" for human genetics.

   This is an extraordinary milestone. For the first time we have an opportunity to see ourselves in an evolutionary mirror. The mouse genome stands for a very important chapter in the lap notebook of evolution. By taking away genes to create sick mice, scientists have been able to reproduce a number of human disorders. Knowing the entire genomes of humans and mice will enable them to employ the technique more widely, leading to a rapid development in the understanding of illness.  62. Scientists have found that _______. 

A. men and mice are actually the same kind of animal

B. mice are actually the ancestors of human beings

C. the genome of the mouse is the same as that of humans

D. humans and mice most probably have developed from the same ancestor

63. The importance of the discovery lies in the fact that ________.

A. man has finally found his true ancestor

B. it will be easier for people to find the causes of human diseases and cure them

C. it has enabled the scientists to compare human DNA with that of another mammal

D. it makes it possible for man to understand the complicated language of his genome

64. The word "unique" in the sixth paragraph probably mean ________.

A. having no like or equal     B. the same

C. corresponding           D. excellent

65. It can be inferred that ________.

A. every gene of a mouse works in the same way as that of a human being

B. it has become easier for humans to find their real ancestor

C. more mice will be used in scientific experiments

D. scientists will use less mice in their experiments to protect them

66. Which of the following is the best title of the passage?

A. Mouse 'Book of Life' May Provide Cures for Human Diseases

B. Mice and Men Are the Same

C. Man and Mouse Share a Common Ancestor

D. The Difference Between Man and Mouse

E

A serious threat to farmers in many parts of the world is erosion. Erosion occurs if a large area of land is cleared of trees and is then badly treated by the farmers. The rain and winds may gradually wash away, or blow away, much of the topsoil. When this happens, crops of wheat or corn become weaker and weaker until nothing grows well. If erosion continues, it will turn good land into a desert.

In the past, when erosion appeared, farmers sometimes moved to a new place to farm. Sometimes they could not move, but continued to try to grow food in the unhealthy soil. This caused them to  sink into poverty . Soon some people realized that there was little rich, fertile soil in the world, government began to try to stop erosion.

One of the first experiments to stop erosion began in the United States in 1933. The experiment was carried out in the area of the Tennessee River. Floods and bad farming in this area had ruined (毁坏) land that had once been rich. As a result, most of the people living around the Tennessee River were very poor.

The government built big dams across the river and stored the water for use during the dry season when there was no rain. The government workers also helped farmers to fertilize their soil and to learn new farming methods which could stop erosion.

At the beginning, the farmers were not interested. But soon, good healthy crops began to appear on the land where new methods were used. Ten years after the extremely prosperous, the great success of this experiment had led similar projects in the other parts of the world.

67.The word“erosion”in the first paragraph means      .

A.the clearing of trees        B.excessive (极度) use of the land

C.the destruction of crops          D.gradual loss of topsoil

68.The passage tells us that erosion is a problem which      .

A.first began in the United States in 1933

B.occurs mainly in river areas

C.affects many parts of the world

D.is becoming more and more serious

69.In the second paragraph,“to sink into poverty”means      .

A.to lose topsoil in erosion

B.to become weaker and weaker until nothing grows well

C.to become very poor

D.to turn into a desert

70.The main purpose of this passage is to tell people        .

A.how erosion occurs

B.the importance of using new farming methods

C.something can be done to prevent erosion                     

   D.the serious outcomes (后果) of erosion

Ⅴ. 短文改错 。(10分)

   For a long time scientist have been wondering         71.          

whether there is life on other planets except the                72.          

earth. They have found very little evidence (证据)         73.          

to support this theory. Not long before, some pieces        74.          

of rock from the outer space were discovered in               75.          

Australia. The discovery excited experts throughout         76.          

the world and these pieces of rock contain              77.          

chemicals similar to those finding on the earth           78.          

and in our own body. This shows that life,              79.          

in some form, is not only possible and probable on other planets.     80.          

Ⅵ. 书面表达 。(30分)

假定你是李平。毕业后,从报上得知广州一家经营有方、在中国享有名气的合资企业-东方电脑公司需招聘职员若干名。2005年 3 月 20日,你给该公司经理Mr. Smith 先生写了一封自荐信,基本内容如下表:

注意:1) 正确使用写信的格式。2)词数:90左右。

      

中 学 英 语 能 力

训练与测试系列   NSEFC- IIB-UNIT-T- 2005                         高中二年级单元目标测试卷

Unit 14  Freedom Fighters

Ⅰ. 单元要点练习。(15分)

1.Darwin is the best student in his class,he______a good example_____his classmates.

  A.makes;to     B.does;for     C.sets;to     D.gives;to

2.The  political scandal shook the nation to its very_____.

  A.foundation    B.base       C.basis      D.ground

3.Her parents______her______that unhonest man.

  A.forbid;marry  B.forbid;marrying C.forbade;to marry D.forbade;marrying

4.Roses need special care______they can live through winter.(2004 NMET)

  A.because     B.so that     C.even if     D.as

5.The book is a group of essays whose_____was the will of man to conquer nature.

  A.topic      B.theme      C.subject     D.matter

6.The murderer was arrested and put into______.

  A.the prison    B.a prison    C.prison      D.prisons

7.Missing school to watch the match is______.

  A.out of the question                  B.out of question

  C.beside the question                  D.without question

8.As we joined the big crowd I got______from my friends.(2001 NMET)

  A.separated    B.spared      C.lost       D.missed

9.They voted______whether they should send a factfinding team there.

  A.for       B.against     C.down       D.upon

10.Have you_____the manager about taking a day off next week?

  A.waited      B.demanded    C.suggested    D.approached

11.Don't waste our energies.Let's______in the business.

  A.join together  B.connect     C.join hand    D.join hands

12.Mr Green will finally______success in his work.

  A.gain       B.get       C.achieve     D.win

13.The thief wished to be______again.Ten years later,he_____free and regained his_____.

  A.free;was set;free                   B.freely;was set;freedom

 C.free;was let;freedom                 D.free;was set;freedom

14.He is one of the______at the boxing match.He is going to_____the first match.

  A.judge;judge            B.judges;judge  

  C.judgement;judgement                  D.judgements;judge

15.Although Mary doesn't look_____,she_____an active part in social activities.

  A.active;plays   B.activity;plays  C.active;joins  D.actively;joins

Ⅱ. 专项语法训练 ( 时态和语态) 。(15分)

16.A new cinema ____ here.They hope to finish it next month.

  A.will be built   B.is built     C.has been built  D.is being built    

17.In such dry weather,the flowers will have to be watered if they ____.

  A.have survived  B.are to survive  C.would survive  D.will survive    

18.If the work ______by the end of the month is delayed, the construction company will be fined.   

  A. being completed                   B. have been completed

  C. to be completed                   D. will be completed

19.John and I ____ friends for eight years.We first got to know each other at a Christmas party.

   But we ____ each other a couple of times before that.        

  A.had been;have seen                  B.have been;have seen                

  C.had;been;had seen                   D.have been;had seen

20.This is Ted's photo.We miss him a lot.He ____ trying to save a child in the earthquake.

  A.killed      B.is killed    C.was killed    D.was killing   

21.He will have learned English for eight years by the time he ____ from the university next year.

  A.will graduate            B.will have graduated

  C.graduates              D.is to graduate           

22.-How long ________at this job?

  -Since 1990. (2003北京春季高考题)                          A.were you employed                B.have you been employed         

  C.had you been employed        D.will you be employed          

23.The little girl _____her heart out because she ______her toy bear and believed she wasn't ever 

   going to find it.(2002北京高考题)

  A.had cried; lost           B.cried; had lost     

  C.has cried; has lost                  D.cries; has lost      

24.If city noises ______from increasing,people ______shout to be heard even at the dinner table    

  20 years from now.

  A.are not kept; will have to     B.are not kept; have to              

  C.do not keep; will have to      D.do not keep; have to             

25.-How long _______ each other before they ______ married?

  -For about a year.                           

  A.have they known; get        B.did they know; get

  C.do they know; are going to get   D.had they known; had got

26.The reporter said that the UFO _______east to west when he saw it.

  A.was traveling            B.traveled                          C.had been traveling                  D.was to travel               27.-Is this raincoat yours?

  -No,mine ________ there behind the door.                   

  A.is hanging    B.has hanged    C.hangs      D.hang    

28.-Have you got your test result?

  -Not yet.The papers ______.

  A.are not correcting                  B.have not corrected                        C.are still being corrected      D.have already been corrected

29.Don't take the magazine away.It ______me.

  A.is belonged to           B.belongs to         

  C.was belonged to           D.is belonging to  

30.-Do you like the new pen?

  -Yes,it ________ very well.

   A.is written  B.is writing   C.writes     D.wrote     

Ⅲ. 完形填空 。(30分)

Many people who work in London prefer to live outside it, and to go in to their offices, factories or schools every day by train, car or bus, even though this   31  they have to get up earlier in the morning and reach  32   later in the evening.

One benefit of living outside London is  33  houses are   34  . Even a small flat in London   35  a garden costs quite a lot  36  . With the same money, one can get a little house in the country with a garden of   37   own.

Then, in the country one can be free from the noise and hurry of the town.  38  one has to get up earlier and spend more time in trains or buses, one can sleep  39  at night, and, during weekends and   40  summer evenings, one can enjoy the  41  clean air of the country. If one 42   gardens, one can spend one's spare time digging, planting, watering and doing the hundred and one other jobs which  43  in the garden. Then, when the flowers and vegetables  44  , one has the reward(报偿) of a person who has shared the secrets of  45   .

Some people, however, take  46   in country things; for them,  47  lies in the town, with its cinemas and theatres, beautiful shops and busy streets, dance-halls and restaurants. Such people would  48  that their life was not worth   49  if they had to live it outside London. A walk in one of the parks and a visit   50   the sea every summer is all the country they want.

31.A.shows      B.expresses    C.means        D.requires

32.A.home      B.family     C.flat         D.house

33.A.because     B.that      C.the         D.all

34.A.cleaner     B.nicer      C.bigger        D.cheaper

35.A.with      B.without     C.near         D.opposite

36.A.money      B.to lend     C.to borrow      D.to hire

37.A.it's      B.its       C.one's        D.their

38.A.If       B.Although    C.Because       D.After

39.A.little     B.less      C.longer        D.better

40.A.on       B.for       C.at          D.by

41.A.cold      B.warm      C.fresh       D.pleasant

42.A.pleases     B.likes      C.wants       D.interests

43.A.need      B.needed     C.are needing    D.are needed

44.A.come on     B.come to     C.come up      D.come over

45.A.mankind     B.society     C.science      D.nature

46.A.no interest   B.an interest   C.much interest   D.great interest

47.A.health     B.happiness    C.wealth      D.future

48.A.know      B.feel      C.understand    D.hope

49.A.to live     B.to spend    C.living      D.spending

50.A.for       B.to       C.of        D.at                 

Ⅳ. 阅读理解 。(50分) 

A

The African ancestors(祖先) of today's black Americans were brought to the US as slaves in the seventeenth, eighteenth, and nineteenth centuries. They worked on farms, especially the large farms in the southern states. Slowly they became a necessary part of the economic(经济) system (系统) of the South.

Slaves did not have the right of human beings. According to the law, they were “things”which belonged to the person who bought them. They had to obey the orders of their owners without question. They were not allowed to learn to read; their owners feared the educated slaves would begin to think about the injustice of the system and would learn to struggle for their freedom.Slaves had to work long hours in very unhealthy conditions. Their owners had complete power over them. They could be bought and sold like animals. At the slave markets, black children were separated from their parents and never saw them again. Slave owners had the right to punish the slave who broke the rule or was against the system. Slaves were often beaten by their owners or killed. After the Civil War, one free slave reported that his owner killed an older slave who was teaching a slave badly should be punished. In practice, however, the law meant nothing.

Opposition to slavery began very early in the history of the US-in 1671-but little progress was made until the beginning of the nineteenth century. By 1804 slavery was illegal(非法的) in the northern states. But it continued, it even grew, in the southern states, which depended on cotton for their economic health(繁荣). Slavery ended in the South only after the Civil War. For blacks, however, the end of slavery was only a beginning, the late beginning of a long and difficult struggle for true justice.

51.According to the writer, why couldn't slaves learn to read?

  A. Because reading was not something which they needed for their work.

  B. Because their owners were afraid that books would give them advanced ideas.

  C. Because they did not have enough time to read.

  D. Because they were so poor that they could not go to school.

52. Which of the following can be inferred from the passage?

  A. Many people from other countries became US citizens in the 1700s.

  B. The US government paid much attention to the freedom.

  C. There were some laws that successfully protected the rights of slaves.

  D. Slavery in the United States began in the seventeenth century.

53. What happened to slaves who broke the rules of the system?

  A. They were given hard jobs.    B. They were punished.

  C. They were sent back to Africa.  D. They were sold to other owners.

54. According to the writer,       .

  A. black Americans had true freedom after the Civil War

  B. black people continued to fight for true justice after the Civil War

  C. nobody in the US was against slavery in the seventeenth century

  D. slavery continued, and even grew, in the United States after 1804

B

The baobab tree grows in west Africa and Australia. It is a very strange-looking tree. Its big trunk (树干) sometimes measures as much as ten metres in diameter (直径). It is thicker at the bottom than at the top. The branches nearest to the ground are very long; those near the top are very short. Strange-looking it may be, it is very useful. Its leaves are good to eat. Its white flowers turn into cool fruit which tastes rather like cucumbers (黄瓜). When a baobab tree gets old, its trunk becomes hollow (空心). An old tree has such a huge, hollow trunk that it can hold many people. When it rains, water collects in the hollow. The tree has such a lot of leaves and branches that the water remains cool and fresh. It is such a useful tree that some people can hardly live without it.

55.The trunk of the biggest baobab tree        in girth (周长). 

   A.can be nearly 32 metres     B.is more than 10 metres

   C.is less than 30 metres     D.is as much as 10 metres

56.Which of the following do you think the baobab tree is like?

 

57.Which of the following may be true?

A.We can find the baobab tree everywhere in Africa.

B.The baobab tree is too big for anybody to make use of.

C.People in Africa and Australia like to plant baobab trees around their houses.

D.Monkeys like to eat the fruit of the baobab tree very much.

C

A man shot Martin King in Atlanta on 4th April, 1968. He drove a white Ford car. Who was he? The police found “Harvey Lowmeyer's” gun and “John Willard's”shirt, and then the white car. It was “Eric Starvo Galt's”car. So what was this man's real name? Marks on the shirt and the car sent the F. B. I. (美国联邦调查局) to Los Angeles. 300 detectives questioned people. At last one found a photograph of “Galt's”room in a small hotel. One fingerprint was enough. James Earl Ray's fingerprint were already on the F. B. I. cards. The police watched railway stations, hotels and airports. 3,000 detectives were trying to find Ray- without success. The Canadian police were helping too. They looked at 240,000 photographs in their Passport Office. Then they found one of“Ramon George Sneyd”. “We gave this man a passport last month,”they said.“He went to London on 2nd May.” The man was wearing thick glasses, but he looked like Ray. Then on 8th June, a detective at London Airport saw Sneyd's name on a passenger list.

It was the end of the biggest manhunt (搜捕) in history. The F. B. I. spent 1.4 million dollars,but they got their man.

58. The key to settling the case (案件) was probably the murderer's      .

   A. car     B. fingerprint  C. gun      D. shirt

59. It took the police       to catch the murderer.

   A. 65 days   B. two months   C. eight weeks  D. three months

60. The hunt for Ray cost       dollars.

   A. a million and four hundred

   B. one million four hundred thousand

   C. fourteen million

   D. a million four thousand

61. Martin Luther King's murderer used the name of      .

   A. Harvey Lowmeyer when he bought the gun

   B. John Willard when he paid for the car

   C. Eric Starvo Galt when he lived in Los Angeles

   D. James Earl Ray when he came to Atlanta

62. The detective at London Airport caught Ramon George Sneyd because      .

   A. Sneyd had a Canadian passport

   B. he was sure that he was James Earl Ray

   C. Sneyd's name was not on the passenger list

   D. Sneyd was wearing glasses.

D

No one knows why we sleep, but it's certain that we need to. People who are prevented from sleeping begin to suffer obvious effects after a few days-they think less clearly, and they fall asleep during the working hours ; some may have hallucinations (幻觉).

There are no rules about sleep. Generally speaking , grown-ups sleep about 7 and half hours each night and probably more than 60 per cent get between seven and eight hours. But perhaps eight per cent are quite happy with 5 hours or less, and four per cent or so find that they want ten hours or more. If you feel all right, you're probably getting enough sleep. The important thing is not to worry how much other people get-their needs may be different. Exercise doesn't seem to increase the need sleep-office workers, for example, sleep for about as long as people doing physically active work.

Children sleep more than grown-ups-perhaps 14 to 18 hours  soon after birth, going down to grown-up levels by early teen-age. Sleep patterns also tend to be different in the elderly, who may sleep less at night than they did when younger, find sleep getting more broken, and often make it a rule to sleep during the daytime.

63. According to the passage, some people are unable to think clearly because      .

   A. they have hallucinations

   B. they feel sleepy during the working hours

   C. they don't have enough sleep

   D. they are certain to be kept from going to bed

64. Which of the following is true?      .

   A. All grown-ups must have at least eight-hour sleep

   B. Most of grown-ups sleep for seven or eight hours

   C. Quite a few people need only 5 hours or less for them to sleep

   D. No grown-ups sleep more than ten hours

65. Whether you have got enough sleep is judged by      .

   A. how many hours you have slept

   B. how many hours you need to sleep

   C. whether you do exercise and physical work

   D. whether you feel fresh and energetic

66. According to the passage, a boy of 14 years old sleeps      .

   A. as long hours as a grown-up

   B. much longer hours than a grown-up

   C. for 14 hours each night

   D. for less than 8 hours per night

E

Gina was surprised to find that it was not raining in London. She stepped down from the train that had brought her from Dover to Victoria Station and put her heavy suitcase on the platform. People were hurrying everywhere:through the entrances and exits, up and down the steps, along the other platforms and over to the ticket counters where there were already queues forming. The porters seemed to be busy helping other passengers. In Milan everyone told her that it was always raining in England. Today, however, they were wrong. It was warm and dusty just like at home. She could even hear someone speaking her native language somewhere near the station bookstall. She walked slowly towards the gate at the end of the platform where a railway official was collecting the tickets. She handed him her tickets and walked through. She looked up the station clock. Its hands were pointing to 3:15pm.

A young woman was sitting on a bench reading an English magazine. She looked friendly so Gina went up to her.

“Excuse me, ”She said. The young woman glanced up with a smile.“Can you tell me the way to circus?”

“The circus?”The young woman looked puzzled (困惑). “What circus do you mean?”

Gina shook her head, “I'm sorry. I have just arrived from Italy to attend a school for overseas (海外) students in London. It is near the circus.”

“Do you have a letter with the school's address on it?”

“Oh, yes. Now I remember. ” Gina opened her handbag and found a postcard with a British stamp on it.

“Ah,” said the young woman. “It's Oxford Circus that you want. It's the name of an area in the middle of London. You must go down these steps and take the Tube. Follow the colours. That's the easiest way.”She showed Gine on the coloured map of the underground railway the route from Victoria to Oxford Circus. “Good luck,” she said. “And by the way, I come from Italy, too.”

67. Gina was      .

  A. expecting a friend to meet her

  B. attending to school matters

  C. going to study English

  D. late for her studies

68. “There were already queues forming,” means      .

  A. there were too many people waiting

  B. people were beginning to wait in line

  C. people were already to form queues

  D. people were being told to form up in lines

69. Oxford Circus is      .

  A. a famous circus

  B. a place in London

  C. a railway station to Oxford

  D. another name for the middle of London

70. The young woman was puzzled because she      .

  A. had never been to a circus

  B. didn't know the word “circus”

  C. didn't know what circus Gina was referring to             

    D. didn't know where Oxford Circus was

Ⅴ. 短文改错 。(10分)

   Today I visited the Smiths-my first time visit       71.         

to a American family. They live in a small town.         72.          

It was very kind for them to meet me at the             73.          

railway station and drove me to their home.             74.          

The Smiths did his best to make me feel at home          75.          

They offered me coffee and other drinks.               76.          

   We have a good time talking and laughing            77.          

together. They eager to know everything about            78.          

China and asked me lots of question. In fact             79.          

they are planning to visit China in next year.            80.         

Ⅵ. 书面表达 。(30分)

画中女孩杨梅吃过早饭,准备去看电影,上车后一位老奶奶上车,她赶紧让位,…请你用英语以第一人称的口吻写一篇短文,叙述一下6 幅图的内容。

参考词语

决定做  …to decide to …  

上车  to get on a bus

道别  to say good-bye to …

挥手  to wave to…

中 学 英 语 能 力                     

训练与测试系列    NSEFC- IIB-UNIT-T- 2005

           高中二年级单元目标测试卷

Unit 15  Destinations

第一节 单项填空 (共15 小题,每小题1 分,满分15 分) 

1.After the whole day work,he got very_____,so he got_____the present work.

  A.tiring;tiring of          B.tired;tired for

  C.tired;tired of           D.tired;tiring in

2.After a while,these people_____towards me and didn't answer my question.

  A.became calm   B.get helpful   C.get excited   D.cooled off

3.He gets the  itch______our understanding,and wants to keep in touch_____us.

  A.for;for     B.with;with    C.for;with     D.with;for

4.More and more people choose to shop in a supermarket as it offers a great____of goods.(2003 SH)

  A.variety     B.mixture     C.extension    D.combination

5.Guilin,a dreamland,gives us a______beauty.

  A.fierce      B.greedy      C.breathtaking  D.seasoned

6.Go and see that play.I______you'll enjoy it.

  A.guarantee    B.complaint   C.budget      D.feel

7.Although he was not_____tired,he didn't care______.

  A.a little;a little          B.a bit;a bit   C.a little;a bit  D.a bit;a little

8.Your room is so small.You should avoid_____too much furniture and_____all the empty space.

  A.buying;fill up  B.buying;filling up         C.buy;fill up   D.to buy;to fill up

9.-Are you going to the concert to be held in the hall?

  -No.The ticket is______expensive for me.

  A.far too     B.very much    C.a lot of more  D.too much

10.I believe you.Your words carry more weight than______.

  A.anybody else   B.anybody else's  C.somebody else  D.somebody else's

11.Though she doesn't like living in the country,______she goes there for a picnic.

  A.now and then  B.all the time   C.from then on   D.at any moment

12.Most businessmen are more interested in the_____success of their products than their educational 

  values.(2002 Taiwan)

  A.cultural     B.commercial   C.classical    D.criminal

13.______I thought it was Mary,but when I looked again I saw it was Jane.

  A.At sight     B.In sight     C.At first sight D.On sight

14.Everyone says he is a_____boy.

  A.promised     B.promising    C.to promise    D.promise

15.This _____me_____what we did together during our holidays.

   A.says;about         B.reminds;of         C.tells;with           D.reminds;X

第二节 专项语法训练 (不定式与分词 ) 。(15分)

16.He will show you the right path ____.

  A. talking     B. taken      C. to be taken   D. to take

17.It is an hour for me ____ to speak here.

  A. to ask      B. asking     C. being asked  D. to have been asked

18.He is the patient ____ on tonight.

  A. to have operated B. to be operated C. operated    D. operating

19.I would appreciate ____ back this afternoon.

  A. you to call   B. you call    C. your calling  D. you're calling

20.He hurried to the station only ____ the train had left.

  A. to be told    B. to tell     C. telling    D. told

21.European football is played in 80 countries, ____ it the most popular sport in the world.

  A. making      B. makes      C. made      D. to make

22.When I got back home I saw a message pinned to the door ____ "Sorry to miss you; will call later."

  A. read       B. reads      C. to read    D. reading

23. ____, I'll go over all these lessons before the exam.

  A. if time will permit                  B. Time permits

  C. Time permitted           D. Time permitting

24.-What made Susan so angry?

   -____ in the rain for a long time.

  A. Being kept waiting                  B. Being kept to wait

  C. Because of waiting                  D. She was kept waiting

25.The book, ____ as a surprise for his sister, was lost in the mail.

  A. intended     B. to be intended C. intending   D. being intended

26.He buried the money, without anyone ____ where it was.

  A. knowing     B. known      C. was knowing  D. was to know

27.Did ____ on time make her teacher angry?

  A. not to come   B. her not to come C. her not coming D. not her coming

28.____, the children went to the park.

  A. It was a fine day                   B. Being a fine day

  C. It being a fine day                  D. Because the fine weather

29.I suggested that the murderer ____ to be sentenced ____ death.

  A. refer; into             B. refers; into

  C. referred; to            D. referring; to

30. -Look! How busy the building workers are!

   -Yes, I see. The houses ____ by them will be completed next week.

  A. built                B. being built

  C. to be built             D. having been built

第三节: 完形填空 (共20 小题,每小题1.5 分,满分30 分)

     阅读下面短文,掌握其大意,然后从各题所给的四个选项中,选出最佳选项,并在答题卡(第10页) 上将该项涂黑。

    Mary is ten years old. She is disabled because she has some brain damage. She could not do even the simplest thing until a year ago. She could not comb her hair or  31  herself. Her parents loved her,but they treated her like a baby.

   Mrs. Williams has a daughter and a son. They are both  32  . They lived far from their mother. Mrs. Williams felt  33  . She decided to do something about it. She decided to become a foster grandmother to a disabled child. And that was how she met Mary. Foster grandparents are grandparents who are not related by  34  , but foster grandparents love the children they 35  for. They love them just as natural grandparents love their grandchildren.

   The foster grandparents also help the disabled children in special ways. They help to dress and feed the children. They read to them and tell them stories. They also help with the disabled children's treatment.

   Mrs. Williams became a foster grandparent to Mary when she was nine years old. At first Mary was  36  . She was afraid because Mrs. Williams was a stranger. But she came to see Mary every day. Gradually, she began to  37  Mrs. William.

   At last Mary let Mrs. Williams do things for her. She came at lunchtime and fed her. One day she   38  the spoon and guided it to her  39  . She told her she must learn to feed herself.

   " Most of the lunch ended up on us  40  in Mary's mouth. " Mrs. Williams remembers. " But it was a  41  for Mary . Mary learned to feed herself in a few months."

  Then Mary was ready for more treatment. Thanks to Mrs. Williams, she had learned to do one simple thing, she could learn to do other simple things. Mrs. Williams was  42   to help with Mary's    43  .

   To became foster grandparents, people must be at least sixty - five years old and in good  44  . They must be willing to give their  45  to disabled children. They are volunteers, so they are not paid.

   Mrs. Williams  46   for most foster grandparents when she says, " We all benefit. The disabled children benefit    47   help them live more useful lives.  And we benefit because we know children  48   us and love us. For any  49   , there is no greater happiness   50   that!"

31. A. help                B. enjoy       C. drink       D. feed

32. A. babies               B. students     C. grown - ups    D. children

33. A. lonely               B. sad         C. happy      D. angry

34. A. birth               B. address      C. name       D. age

35. A. look                B. care        C. take        D. search

36. A. sorry               B. glad        C. surprised    D. shy

37. A. know     B. realize      C. recognize     D. trust

38. A. played               B. handed       C. washed      D. hit

39. A. nose                B. eye        C. mouth       D. ear

40. A. but                 B. from        C. instead of    D. in spite of

41. A. job                B. start        C. invention    D. help

42. A. trained              B. forced       C. afraid      D. tired

43. A. treatment             B. medicine     C. food        D. drink

44. A. wealth              B. health       C. position     D. heart

45. A. money               B. life        C. house       D. time

46. A. tells               B. speaks       C. announces    D. informs

47. A. before              B. unless       C. because     D. otherwise

48. A. help                B. need        C. treat      D. touch

49. A. place               B. person       C. time       D. chance

50. A. as                 B. so         C. than       D. that

                   第三部分:阅读理解 (共20 小题,每小题2.5分,满分50分)

                                          A

   When we walk through the city, we all experience a kind of information overload but we pay attention only to those that are important to us. We don't stop, we keep our faces expressionless and eyes straight ahead, and in doing so, we are not just protecting ourselves but are avoiding overloading other people as well.

   We make use of stereotypes(刻板的模式)as convenient ways to make quick judgments about situations and people around us. They may not always be accurate(正确的),and they can often be dangerously wrong, but they are used regularly.

   The problem with the stereotypes is that they restrict experience. By using limited clues to provide us with a rapid opinion of other people or places we may choose to limit our communication. We may decide not to go to certain places because we believe they will not offer something we enjoy.

   In the city, styles of dress are particularly important with regard to self-presentation. Different groups often use clearly identifiable styles of clothes so that they can be easily recognized. It is becoming increasingly common for brand names to be placed on the outside of clothes, and this labeling makes it easy to send out information about fashion and price instantly, and lets others tell at a distance whether an individual has similar tastes and is a suitable person to associate with.

  In England, where social grouping or class continues to make social distinctions(区分),clothes, hairstyles, people's pronunciation and the manner of speaking are all clues to our social group. Class distinctions tend to be relatively fixed, although in the city where greater variety is permitted, they are more likely to be secondary determining factors of friendship and association.

51..People walking in cities ignore the surroundings because         .

A.they do not wish to talk to other people

B.everyone else is expressionless

C.the environment is already familiar to them

D.there is too much information to take in

52.According to the passage, the main disadvantage of using stereotypes is that they       .

A.are likely to lead us into dangerous situations

B.may make us miss some pleasant experience

C.can rarely be relied on

D.make us mentally lazy

53.From the passage we may conclude that         .

A.stereotypes can help to understand people fully

B.people are becoming more interested in fashion

C.dressing can send messages about individuals

D.stereotypes can do more harm than good to people

54.It would appear that in England, a person's class         .

A.might be less important in making friends in a city

B.is mainly determined by his pronunciation

C.plays less of a role than it did in the past

   D.is something that can be changed easily.           

                                   B

  A suicide bomber (人体炸弹)blew up a bus parked outside a Karachi hotel yesterday in a thun-derous explosion that killed 11 French engineers, their Pakistani driver and a passer-by. Twenty-three people were wounded.

  Pakistan's government denounced (公开指责) the blast (爆炸) as an act of terrorism aimed at foreigners, and suspicion (嫌疑)fell on militant Islamic groups (穆斯林军队组织) angered by Pakistani President General Pervez Musharraf's support for the US-led coalition's war in neighboring Afghanistan.

  There was no evidence linking the attack directly to the al-Qaida terrorist network of Osama bin Laden, but many of the militant Islamic groups in Pakistan are sympathetic (同情) to al-Qaida and some have links to the organization, according to The Associated Press.

  Musharraf promised to fight back.

   "We feel this act of international terrorism has to be met with full force. My government has the complete resolve of meeting this threat," he said.

  "I would appeal to the international community to understand our domestic environment resulting from our cooperation against international terrorism," Musharraf said without elaborating.

  French President Jacques Chirac condemned (谴责) the attack and sent his newly appointed defense minister to Pakistan.

  The 11 French killed in the bombing were engineers working at the Karachi seaport for a state-owned French marine construction company, the French Foreign Ministry said in Paris. They were part of a team building a submarine Pakistan bought from France.

  The bus was parked outside the Sheraton Hotel when the bomb went off-apparently in a second vehicle driven by the bomber, tearing a large crater (弹坑) in the road and destroying nearby vehicles.

  "We have recovered a charred (烧焦的) body from a car," said Sindh provincial police chief Sayed Kamal Shah, referring to the suicide bomber and his vehicle. "The sound was so loud I think you could have heard it from six miles away," said Munir Sheikh, a police officer who was actually present at the event and saw the explosion. The death toll stood at 14, including the bomber.

55.The blast in Pakistan yesterday          .

A.killed 11 people and injured 23        B.killed 12 people and injured 23

C.killed 13 people and injured 23        D.killed 14 people and injured 23

56.Which of the following is true according to the passage?

  A.Pervez Musharraf is French President.  

  B.Jacquess Chirac is Pakistani President.

  C.Sayed Kamal Shah is a police officer.

  D.Munir Sheikh is member of a militant Islamic group.

57.Suspicion fell on         .

  A.Osama bin Laden              B.militant Islamic groups

  C.the ai-Qaida terrorists           D.a Pakistan policeman

58.The French Foreign Minister          .

  A.was newly appointed and sent to Pakistan by French President yesterday

  B.condemned the attack in a press conference in Paris yesterday

  C.denounced the blast as an act of terrorism aimed at French people

  D.said the French killed were engineers building a submarine Pakistan bought from France.

59.The suicide bomber         .

  A.died of terrible burns

  B.shot through the window of his vehicle

  C.was found dead in a large crater in the road                    

  D.was arrested by Pakistan police officer Munir Sheikh

                                   C

   In the second half of each year, many powerful storms are born in the Atlantic and Caribbean seas. Of these, only about half a dozen cause the strong, circling winds of 75 miles per hour or more that are named hurricanes, and several usually make their way to the coast. There they cause millions of dollars of damage, and bring death to large numbers of people.

  The great storms that hit the coast start as harmless circling disturbances (大气搅动) hundreds- even thousands--of miles out at sea. As they travel aimlessly over water warmed by the summer sun, they are carried westward by the trade winds. When conditions are just right, warm, wet air flows in at the bottom of such a disturbance, moves upward through it and comes out at the top. In this course, the wetness in this warm air produces rains, and with it the heat is changed into energy in the form of strong winds. As the heat increases, the young hurricane begins to swirl(旋转) in a counter-clockwise(逆时针) motion.

  The average life of a hurricane is only about nine days but it contains almost more power than we can imagine. The hidden energy of the heat given off by a hurricane's rainfall in a single day would satisfy the entire electrical needs of the US for more than six months. Water, not wind, is the main source of death and destruction(破坏) in a hurricane. A typical(典型) hurricane bring 6-to-12 inch   downpour resulting in sudden floods. Worst of all is the powerful movement of the sea - the mountains of water moving toward the low- pressure hurricane center. The water level rises as much as 15 feet above normal as it moves toward shore.

60.What is the worst thing about a hurricane?

   A.The harmful effects of water.           B.The heat it gives off.

   C.It lasts about nine days on average.  D.It's very strong wind.

61.The counter- clockwise swirling of the hurricane is brought about by       .

A.the low-pressure area in the center of the storm

B.the increasing heat

C.the trade winds

D.the force of waves of water

62.The underlined word "downpour" probably means           .

   A.dangerous waves              B.the energy produced by the hurricane

   C.heavy rainfall              D.the progress of water to the hurricane center

63.What's the main idea of the text?

A.The great heat is caused by counter-clockwise swirling.

B.The process of changing storms into hurricanes.

C.Circling disturbances can lead to destructive hurricanes.

D.The cause of a hurricane and its destruction.             

              D

Build the highway and watch the town grow .At first a few shops appear and maybe a restaurant. Then a hotel opens. Eventually new houses are built. A village is born.

This is also how the virtual world has developed. Think of the Internet as the road carrying information between two computers. Think of the World Wide Wed as the village. At first it is just a place on the virtual road where travelers meet .More travelers come bringing new kinds of information .New villager are started.

Every village has a founder. Tim Berners-Lee is the man who wrote the software (软件)programme that led to the foundation of the World Wide Web .How did he get the idea? He tells us on his own web site."One of the things computers were not able to do was store contacts(互通信息) from different sources .The dream behind the Web is of a common space in which we communicate by sharing information ."

Tim Berners-Lee could have followed the Microsoft route by forming a company to sell the programmes he invented .Or he could have joined an existing company .But in his view the Web is a language, not a product. Charging a fee for using his programmes would have slowed the growth of the Web. And other companies would make similar products to compete. Instead of one World Wide Web there would be several smaller Webs .Each would use incompatible (不相容的)software. The Web is valuable because it uses a common computer language to reach people and share information. Competing webs would lose this value. Imagine if somebody sent you a bill every time you spoke a word of English.

In 1994 Tim Berners-Lee formed the newly formed World Wide Web Consortium, or W3C. More than 200 leading companies and laboratories are represented by W3C. Together they make sure that everyone, no matter what their equipment or software, can work equally on the Web .

"The Web can help people to understand the way that others live and love .It helps us understand the humanity of people ,"he says.

64.What does the underlined part "the virtual world "refer to ?

A.The computer system.

B.Scientific research.

C.All the information on the Web and the Internet.

D.People working on the Web and the Internet.

65.What can be inferred from the sentence "Imagine if somebody sent you a bill every time you 

   spoke a word of English" ?

A.The information on the Web should be shared by all.

B.The Web is where all kinds of information are stored.

C.People can get al kinds of information on the Web.

D.People can communicate on the Web .

66.Which is the main idea of the passage?

A.How Tim Berners-Lee founded the World Wide Web            

B.The advantages of working on the World Wide Web

C.Why Tim Berners-Lee wanted to found the World Wide Web

D.How Tim Berners-Lee got the idea of founding the World Wide Web

              E         

     I don't know anyone who doesn't want a real life filled with love. In order for this to happen, the effort must start within us. Rather than waiting for other people to provide the love we desire, we must be a vision and source of love. We must tap into our own loving-kindness in order to set an example for others to follow.

   It has been said that" the shortest distance between two points is an intention. "This is certainly true with regard to a life filled with love. The starting point or foundation of a life filled with love is the desire and commitment to be a source of love. Our attitude, choices, acts of kindness, and willingness to be the first reach-out will take us toward this goal.

   The next time you find yourself frustrated(受挫)at the lack of love in your own life or at the lack of love in the world, try an experiment. Forget about the world and other people for a few minutes. Instead, look into your own heart. Can you become a source of greater love? Can you think loving thoughts for yourself and others? Can you extend these loving thoughts outward the rest of the world-even to people whom you feel don't deserve it?

   By opening your heart to the possibility of greater love, and by making yourself a source of love( rather than getting love) as a top priority (重点), you will be taking an important step in getting the love you desire. You'11 also discover something truly remarkable. The more love you give, the more you will receive. As you put more emphasis on being a loving person, which is something you can control-and less emphasis on receiving love, which is something you can't control-you'll find that you have plenty of love in your life. Soon you' 11 discover one of the greatest secrets in the world: Love is its own reward.

67. According to the passage, the goal of real life is to ______.

A. set an example for others to follow        B. make the shortest distance between two points

C. have a plain attitude to our life            D. fill our life with love

68. If you find yourself frustrated at the lack of love in your life you should _______.

A. forget all about the unhappy things        B. try hard to get more love from others

C. try to make yourself a great source of love  D. care for yourself instead of others

69. According to the idea of the passage, you will find plenty of love in your life as _______.

A. you become a loving person              B. you become a loved person

C. you discovered the secrets in the world     D. you forget the world and other people

70.The best title of the passage is _______ .

   A. More Giving, More Receiving            B. Fill Your Life with Love     

  C. Love is the Reward of Love              D. More Giving than Given  

              第四部分:写作 (共两节,满分40分)

 第一节:短文改错 (共10 小题,每小题1 分,满分10 分)

      In the past my hometown was used to be a beautiful place.       71.          

Thick trees and green grass could be seen in everywhere. In            72.            

order to build house and grow more crops, people cut down             73.            

more and more trees. With time went on, the whole forest was          74.            

almost destroyed. Gradually the green hills have changed into           75.           

wasteland. As a result, sandstorms struck us now and then, from         76.           

which we suffer a lot. However, I do hope all the people should         77.           

realize the terribly result of not caring about our environment.         78.           

What's more, they should take good care of the forests and plant         79.            

trees instead cutting them down to improve our living conditions.        80.           

                                    Li Hua

第二节:书面表达 (满分30 分)

    3月22日(周六)下午,你班与2班举行了一场足球比赛,下面六幅图描述了你在这场足球赛中发生的事,请根据图画用英语写一篇100词左右的日记。

注意:

    1.短文必须包括所有图画内容,可适当增加细节使日记连贯;

    2.在日记的结尾适当发表感想;

    3.日记的格式已经给出。

  

                                                                                             

                                                                                                

                                         Saturday, February 22nd          Fine    

 

中 学 英 语 能 力                     

训练与测试系列    NSEFC- IIB-UNIT-T- 2005                     

           高中二年级单元目标测试卷

Unit 16 The United States of America

第一节:单项填空 (共15 小题,每小题1 分,满分15 分)

1.He put forth two problems at the meeting.The _____is about how to solve the pollution of the river,the_____is on how to deal with the safety of production.

  A.first;late    B.former;second  C.former;latter  D.former;later

2.In course of these years many books had______in the house.

  A.gathered     B.collected    C.accumlated    D.assembled

3.They______me $1,000 for the old house.But I didn't______their offers.

  A.supplied;take  B.provided;receive C.offered;accept  D.offered;refuse

4.His carelessness  led him______failure,which has bad  effect______him.

  A.in;in      B.on;on      C.to;to      D.to;on

5.If you want to  recover______your illness,I remind you_____eat much sugar.

  A.in;not      B.from;not to   C.out of;to not  after;of

6.That shop doesn't have any brown sugar on______at the moment,but it expects to have some tomorrow.

  A.pile       B.sale      C.femand      D.store

7.Let Harry play with your toys as well,Clare-you must learn to______.

  A.support     B.care       C.spare      D.share

8.He wants to get a better_____and earn more money.

  A.job       B.work       C.employ      D.employment

9.Those girls walked out every morning,_____losing weight.

  A.aim at     B.aiming at    C.aimed at     D.to aim at

10.He stayed with his mother till midnight,comforting her and being comforted_____.

  A.in turn     B.in return   C.in vain     D.instead

11.I suggested_____a taxi,but he_______.

  A.to take;insisted walking      B.taking;insisted to walk

  C.taking;insisted on walking     D.to take;insisted on walking

12.The teacher wrote an example on the blackboard to______the point.(2004 Shanghai)

  A.illustrate    B.suggest     C.express     D.recognize

13.Washington,a state in the United States,was named______one of the greatest American presidents.

  A.in honour of   B.instead of    C.in favour    D.by means of

14.The school board listened quietly as John read the demands that his follows______for.

  A.be demonstrating          B.demonstrate  

  C.had been demonstrating       D.have demonstrated         

15.He succeeded in his efforts to_____his fatal weakness.

  A.win       B.beat       C.defeat      D.overcome

第二节:专题语法练习 (非谓语-ing;-ed) (共15 小题,每小题1 分,满分15 分).

16.He walked down the hills,    softly to himself.

 A.sing       B.singing      C.sung       D.to sing

17.I had to shout to make myself    above the noise.

 A.heard       B.hearing      C.heard       D.to hear

18.The graduating students are busy    material for their reports.

 A.collect      B.to collect    C.collected    D.collecting

19.The cars    in Beijing are as good as those    in Shanghai.

 A.produce, produce            B.produced, produced

 C.produced, producing          D.producing, producing

20.When I came in, I saw Dr. Li   a patient.

 A.examine      B.examining    C.to examine     D.examined

21.   a satisfactory operation, the patient recovered from illness very quickly.

 A.Having been given           B.Having given

 C.Giving                 D.Being given

22.He reads newspapers every day to keep himself    about what's going on in the world.  

 A.inform                 B.informing

 C.informed               D.being informed 

23.I delayed   your letter because I had been away for a week.

 A.answer      B.answering    C.writing      D.to post

24.The thief drove as fast as he could to escape    by the police.

 A.to be caught             B.he caught

 C.being caught              D.catching

25.I searched for my wallet and it wasn't there. At first, I thought I    it at home. Then I 

   remembered    it out to pay for the taxi.

 A.must have left;to take       B.may leave;taking

 C.might leave; to take         D.could have left;taking

26.   the news of his father's death, he burst into tears.

 A.After hearing             B.On hearing

 C.While hearing            D.Having heared

27.   his mother, the baby could not help    .

 A.To see;to laugh                     B.Seeing; to laugh

 C.Seeing;laughing                     D.To see;laughing

28.It's no use     so much money on clothes.

 A.spend       B.spent       C.spending     D.being spent

29.The sentence needs   .

 A.improve                B.a improvement

 C.improving                        D.improved

30.If he succeeded   a job, his children wouldn't be suffering from hunger now.

 A.to find                B.to look for

 C.in finding              D.in looking for

第二节: 完形填空 (共20 小题,每小题1.5 分,满分30 分)

    阅读下面短文,掌握其大意,然后从各题所给的四个选项中,选出最佳选项。

    London—life for Cathy Hagner and her three children is set to permanent (不大可能改变的).

Their  31  school day and her job as a lawyer's assistant are busy enough.  32  Hagner also has to take the two boys to soccer or hockey or basketball while dropping off her  33  at piano lessons or Girl Scout Club.

     34  , the exhausted (筋疲力尽的) family doesn't get home until 7 p.m. There is just time for a quick  35  before homework.

   In today's world, middle-class American and British parents treat their children  36  they were competitors  37   for some finishing line.

   Parents take their children from activity to activity to make their future  38  . It seems that raising a genius (英才) has become a more   39   goal than raising a happy and well-balanced child.

"  40  across the country are reporting a growing number of children  41  from stomachaches and headaches  42  exhaustion and stress,"says child expert William Doherty,of the University of Minnesota.

   Teachers are  43  exhausted kids in the classroom. It's a very serious problem. Many children attend  44  clubs by necessity. But competitive pressures also   45  an explosion of activities. They   46  sports, language, music and maths classes for children as  47  as four.

   "There is a new parenting trend (趋势) under way which says you have to tap all your child's potential (潜能) at a young age,  48  you will let them down," says Terry Apter, a Cambridge-based child and adolescent psychiatrist (青少年精神病专家).

   "It isn't entirely  49  : there have always been pushy parents. But what was seen as strange behavior before is now well  50  ."

31.A.half      B.part       C.full       D.whole

32.A.And       B.So        C.But       D.For

33.A.daughter    B.sons       C.girls      D.kids

34.A.Often      B.However     C.Though      D.Seldom

35.A.lunch      B.supper      C.breakfast    D.tea

36.A.even if     B.as if      C.now that     D.in case

37.A.hoping     B.caring      C.calling     D.racing

38.A.equal      B.smooth      C.excited     D.bright

39.A.exact      B.excellent    C.difficult    D.important

40.A.Doctors     B.Lawyers     C.Engineers    D.Businessmen

41.A.dying      B.preventing    C.suffering    D.learning

42.A.due to     B.so as to     C.according to   D.referring to

43.A.dealing with            B.playing with   C.going on with D.getting on with

44.A.grown-up    B.bodybuilding   C.after-school   D.night

45.A.grow      B.reduce      C.stop       D.create

46.A.have      B.enjoy      C.teach      D.include

47.A.old       B.young      C.many       D.much

48.A.except that             B.therefore    C.otherwise   D.unless

49.A.old       B.new       C.wrong      D.right

50.A.respected    B.accepted     C.refused     D.managed

          第三部分:阅读理解 (共20 小题,每小题2.5分,满分50 分)

                                             A

   We believe that there is more to education than preparation for a job. Children must be prepared for all aspects of their adult life work, leisure, personal relationships, creative activities, coping with money matters, independence, parenthood.

   Nevertheless, it would be unrealistic to provide an education which took no account of the needs of employers.

   What type of training does the business world regard as important?

  Many young people applying for jobs were, in the employers' opinions, very weak in the basic skills of handwriting, grammar and spelling. Though additional education at university level improved the students' general ability, a report states, "In basic skills the standards remained stubbornly low."

   It is debatable(有争议的) whether standards have declined(减弱) in recent years. What is certain is that employers do not believe the standard is now high enough. Do technological changes make greater demands upon the students' abilities?

   We should also remember that the job expectations of young people have increased. The girls who would have once become shop assistants or hairdressers now want to be secretaries. Boys who sought an apprenticeship(学徒) 20 years ago now desire to an engineering degree. But it is still the same girls and boys with the same degree of ability. No wonder there are problems in reaching the "necessary standards" of the business world.

   Many employers believed that it was important for teachers to have experience outside the world of college and school. They should work for a while at some other kind of job "to see how the world of business and commerce differs from their own". The teaching profession and society in general needed a greater understanding of manpower needs and therefore of "the desired" direction of the education system.

51.The article mainly talks about            .

   A. no education among young people

   B. meeting the educational requirements of employment

   C. the problem of unemployment

   D. the weaker standards of education

52.In the writer's opinion, education should           .

   A. pay more attention to students' academic(学术的) ability

   B. only meet the needs of employment

   C. be suitable to all aspects of being an adult, including employment

   D. focus on grammar

53."The same girls and boys" in Para. 6 refer to          .

   A. those who are likely to be unemployed

   B. those who have just left middle school

   C. those who are looking for jobs far beyond their abilities

   D. those who might have become shop assistants, hairdressers and apprentices in the past

54.It's suggested that teachers should          .

   A. never change their jobs

   B. spend more time on their school work

   C. improve their teaching methods                      

   D. get some work experience outside school             

B

Dolly is the first cloned sheep ever born artificially in a laboratory. Ever since Scottish scientists announced her birth, her life has been close to that of a rock superstar. A sweater knitted from her wool is a treasured showpiece in a museum. Newspaper reporters dig out the slightest details of her life; even her sexual life makes major news.

Now, in the fourth year of people's fascination with Dolly, it seems a fair time to ask whether she is really worth all that excitement. And the answer, everyone seems to agree, is yes. Initially, the discussion was mostly based on shock. No one has really seen a cloned human being except in science fiction. Faced with the reality, biologists, doctors, lawyers, and religious leaders have all been thinking about how the world might change.

But as time goes by, the cloning technology that brought Dolly into the world has shown that it is not a one-hit wonder. Researchers are producing clones from the cells of an animal with relative ease in laboratories. Others have shown they can combine cloning and genetic engineering to produce cows and sheep that contain proteins for medical uses. There seems to be no doubt that our future will be a future with clones. In fact, nearly every month, there is news of how cloning technology is likely to change our lives. From agriculture to medicine, from psychology to law, some scholars are studying the possible impact of cloning along the way. They are wondering if cloning is the end of humanity, or the start of something quite promising.

55. Dolly is the first sheep        .

  A. reproduced through a special genetic method

  B. appeared on stage as a rock superstar

  C. reported in newspapers for being sexy

  D. treasured in a museum as a showpiece

56. People have been excited about Dolly because they         .

  A. look usually upon cloning technology

  B. hate to live with cloned animals

  C. will be replaced by a better human species

  D. never saw a cloned creature before Dolly

57. The passage closes with a remark which is           of cloning technology.

  A. of a passive attitude to the value        B. uncertain about the impact

  C. in favor of the development    D. sure about the effect 

C

    Square tables create a competitive or defensive relationship between people of equal status .Square

tables are idea for having short conversations or to create a superior-subordinate (上下级) relationship.The most cooperation usually comes from the person seated beside you and the one on the right tends to be more cooperative than the one on the left.The most resistance usually comes from the person seated directly opposite.

   King Arthur used the Round Table as an attempt to give each of his knights an equal amount of authority and status.A round table creates an atmosphere of relaxing and status.A round table creates an atmosphere of relaxing informally and is idea for promoting discussion among people who are of equal status as each person can claim the same amount of table territory (领域) .Removing the table and sitting in a circle also promotes the same result.Unfortunately,King Arthur was unaware that if the status of one person is higher than the others in the group it changes the power and authority of each individual.The King held the most power at the Round Table and this meant that the knights seated on either side of him were non-verbally (无言) granted the next highest amount of power ,the one on his right having a little more than the one on the left,and the amount of power diminished (减弱) relative to the distance that each knight was seated away from the King.Therefore,the knight seated directly across the table from King Arthur was in effect,in the competitive-defensive position and was likely to be the one who gave the most trouble.Many of today's business officials use both square and round tables.The square desk,which is usually the formal desk,is used for business activity,brief conversations,reprimands (训 斥) and the like.The round table,often a coffee table without the around seating,is used to create an informed relaxing atmosphere or to persuade.

58.The underlined word  status  probably has the meaning of         .

   A.chance         B.work             C.position in relation to others D.enviroment

59.The writer wants to tell us         .

   A.square tables have better results in talking

   B.round tables have better results in talking

   C.sitting on the left is better than on the right

   D.different tables have different results in talking

60.When a square desk is used,what does that mean?

   A.It means a formal or relaxing atmosphere.

   B.It means a informal or relaxing atmosphere.

   C.It means a competitive-defensive relationship or a reprimands.

   D.It means a competitive or defensive relationship or to persuade.

61.Look at the drawing,the crown is the King's place,where should the person who is likely to give

   most trouble sit?

 

D

WASHINGTON— Almost 13 percent of Americans still live in poverty in spite of a slight decrease in their numbers last year and a rise in average household (家庭的) income, the Census Bureau reported last week. Last year, the poverty rate dropped to 12.7 per cent of the population from 13.3 per cent the year before, though common household income per year rose for the fourth year running.

According to the US government, poverty is explained as people who earn US$ 13,003 a year for a family of three, and US$ 16,660 a year for a family of four. The number of those living in poverty fell from 35.6 million in 1997 to 34.5 million in 1998 — but that number is still high in light of the growing economy, which has seen the largest peacetime growing in history since US President Bill Clinton took office over six years ago. One reason for the continued high poverty rate is due to immigration(移民), according to Steve Camarota with the Center for Immigration Studies. His figures show the total number of immigrates living in poverty has tripled over the last 19 years from 2.5 million to 7.5 million in 1998. Clinton took the chance to declare the country's strong  financial(财政的) health, saying,“It has helped lift millions of hard-working families out of poverty.”“The best news is that these gains finally are being shared with all groups in America, from the wealthiest to the poorest,”he told reporters at the White House. But the Census report showed that while white households saw their incomes rise 3 per cent from 1997-1998 to a record US$ 424,000, African-American families held steady at an average US$ 254,000 annually. Hispanic (拉丁美洲的) households had a 4.8 per cent increase to US$ 28,300 a year for the period.

62. From the first paragraph, we can get to know that in the last few years        .

  A. American families have become richer

  B. American families have become poorer

  C. most American families have enjoyed a better life

  D. most American families have suffered a lot

63. In America, “poor families”        .

  A. have more people than others

  B. live in the countryside with more children

  C. live in big cities with more children

  D. refer to those whose income goes below a certain amount

64. The underlined word“tripled”means        .

  A. become three times  B. made up of three parts  C. fallen down greatly   D. grown up greatly

65. What Clinton said means that         .

  A. he would support the poor over the world    B. he would be president forever

  C. he had done well as president          D. he would make the poor richer

66. From the whole passage we can conclude that        .

  A. people in America live equally

  B. people in America live happily

  C. people from other countries have made America poorer than before    

  D. people immigrated from other countries may face difficulties in America      

E

Faced with the threat of water shortages, Beijing and Shanghai will take effective measures to save water and protect water resources.

Beijing will stick more strictly to watersaving policies through the readjustment of industrial structures. Beijing is expected to be short of 1.185 billion cubic metres of water by 2020. Beijing will shut down factories with high water consumption (消耗) and pollution including electric power, steel and paper manufacturing equipments. Advanced water-saving technology will be introduced to new industrial projects in the capital city.

Grain-growing areas will be reduced to save ground water and more trees will be planted. Animal breeding and other“highly efficient”agriculture with modern water-saving irrigation methods will be developed.

It is said that water used in agriculture will drop to 35 percent of the city's water consumption in 2010 from 43 percent in 1998, and the figure will continue to drop to 28~30 percent in 2020. Beijing will increase the speed of renovation (修复) of its urban water supply equipments. It's reported that more than 15 percent of water is lost during distribution (分发). Water-saving equipment and efficient management can save Beijing more than 537 million cubic metres by 2010.

Shanghai still faces key problems connected with its water resources and environment. Since 1998, the city has invested nearly US $ 169 million to treat its rivers, especially Suzhou Creek. The city's rivers have become noticeably clearer since putting it into action.

The government will provide a further US $ 24 million for the treatment of rivers and US $ 12 million to treat sewage (污物).

   This year's task is to improve the water quality at the three ports of Longhua, Yangshupu and Hongkou. Another emphasis to raise the water system in Songjiang New Area with a project worth US $ 4.8 million. Efforts will be made to improve public awareness about the need to protect water resources.

67. Grain-growing areas in Beijing will be reduced because         .

  A. grains can't fetch a good price in China

  B. a lot of ground water will be saved a lot by this means

  C. Beijing helps to develop advanced technology

  D. highly effective agriculture needs fewer farmland

68. Beijing can save over 537 million cubic metres by 2010 by means of       .

  a. water-saving equipment   b. effective management

  c. artificial rainfall     d. reducing water pollution

  A.ab       B. bc     C. bd     D. abcd

69. From the passage, we know Shanghai will invest        million dollars on the treatment of river and sewage.

  A. 169   B. 36     C. 40.8       D. 201.7

70. The author wrote the passage to tell us         .

  A. Beijing and Shanghai are short of water                    

  B. to save every drop of water in our daily life

  C. big cities like Beijing and Shanghai are trying their best to protect water resources

  D. water shortages have become one of the most important problems that China has to deal with

第一节:短文改错 (共10 小题,每小题1 分,满分10 分)

    Most of large cities in the world have grown                71.         

without plans, in which London is a such one.                   72.       

London's streets wind its way and there is no reason            73.         

or order to its street number system. Indeed no              74.         

one but a practical taxi driver knows the whole of                 74.         

London. And after he gets his license, he must                          76.       

first tour the city for months, street by the street, then             77         

take a test to prove that he can find his way about.            78.       

New York and Chicago grew in much the same. They            79.         

spread out like pouring water expanding at all directions.      80.      

  第二节:书面表达 (满分30 分)

假设下面六幅图描述的是你在 5 月 3 日(星期五)的一段亲身经历。请你根据画面所示内容,用英语写一篇 100 字左右的日记。

中 学 英 语 能 力                         

训练与测试系列  NSEFC- IIB-UNIT-T- 2005

           高中二年级单元目标测试卷

Unit 17  Disability

Ⅰ. 单元要点练习。(15分) 

1.The album of paintings in this museum is a______from an old professor.

  A.gift      B.present     C.talent      D.genius

2.It was not a serious illness,and she soon______it.

  A.got over     B.got on with   C.got round    D.got out of

3.These things must be put into______.

  A.produce     B.production    C.productive    D.product

4.I have_____the new climate here.

  A.adjusted to  B.agreed with   C.been suited to  D.all the above

5.There are______eggs and______apples in the basket.

  A.two scores;two dozen of       B.two score of';two dozen

  C.two scores of;two dozen       D.two score;two dozens

6.Carl wanted to take_____of a monkey in the zoo,and soon he______it.

  A.the place;used to                   B.place;got used to

  C.place;used to            D.the place;got used to       

7.The manager has got a good business_____so the company is doing well.(2003 Beijing)

  A.idea       B.sense      C.thought     D.thinking

8.We need to consider what_____we will be using for language training.(2004 Shanghai)

  A.abilities    B.appliances    C.facilities   qualities

9.Jim never thought he would_____the first prize in the competition.

  A.end up in    B.be recognized  C.make as effect  D.end up with

10.In order to gain a bigger share in the international market,many state-run companies are striving

  ______their products more competitive.(2002 NMET)

  A.to make     B.making      C.to have made   D.having made

11.Some people like drinking coffee,for it has_____effects.(2004 Shanghai)

  A.promoting    B.stimulating   C.enhancing    D.encouraging

12.Words______me when I wanted to express my thanks to him for having saved my son from the buring house.(2004 Shanghai)

  A.failed      B.left       C.discouraged   D.disappointed

13.-Will we be able to visit the White House?

  -Sure.It's open_____every morning.

  A.for public   B.for the public  C.to the public  D.to public

14.Life is limited,_____there is no limit to_____the people.

  A.and;serve    B.but;serving   C.so;be served   D.besides;having served

15.Those T-shirts are usually $35 each,but today they have a______price of $19 in the Shopping Center.(1999 Shanghai)

A.regular     B.special     C.cheap      D.particular

Ⅱ. 语法专练 (数量及其比较 ) (15分)

16.Miss Brown can jump over the wall which is ____.

  A. six-feet-high           B. six-foot-high

  C. six foot high           D. six feet high

17.I think ____ is enough for those young men.

  A. ten minutes' rest                   B. a ten minute's rest

  C. ten-minute rest          D. a ten minutes rest

18.Shortly after the accident, two ____ police were sent to the spot to keep order.

  A. dozens of   B. dozens      C. dozen      D. dozen of

19.____ of the land in that district ____ covered with trees and grass.

  A. Two fifth; is            B. Two fifth; are

  C. Two fifths; is           D. Two fifths; are

20.____ people in the world are sending information by E-mail every day.

  A. Several million          B. Many millions

  C. Several millions                   D. Many million

21.As a result of destroying the forests, a large ____ of desert ____ covered the land.

  A. number; has            B. quantity; has

  C. number; have            D. quantity; have

22.Three ____ students are needed to plant trees this afternoon.

  A. hundred of the           B. hundreds of the

  C. hundred of             D. hundreds of these

23.-How many pencils would you like, Madam?

  -Well, I want ____ these.

  A. three dozen            B. three dozen of

  C. three dozens            D. three dozens of

24.Karl Marx began to learn ____ Russian language ____.

  A. the; in his fifty                   B. the; in his fifties

  C. /; in his fiftieth                  D. /; in his fifties

25.I carry only enough money to make change for a ____ bill.

  A. ten-dollar             B. ten-dollars

  C. tens-dollar            D. tens-dollars

26.____ of the rivers here have been polluted.

  A. Two-thirds             B. Two-third

  C. Second-third            D. Two-third

27.She seems to be too short for a ____.

  A. twenty-year old girl        B. girl of twenty

  C. twenty-year-old girl        D. girl twenty-year-old

28.Waves of red light are about twice ____ waves of blue light.

  A. so long as             B. as long as

  C. much longer than                   D. no longer than

29.I have invited 50 guests to the party, but actually ____ came.

  A. twice more than          B. twice as much

  C. twice as many           D. as many as twice

30.China is larger than ____ in Africa.

  A. any of the other countries     B. any other country

  C. any country            D. all the other countries

Ⅲ. 完形填空 。(30分)

A teacher was about to give a student a zero for his answer to a physical question,  31  the student said he should  32  a perfect score if the system were not  33  against the student. I was selected as a judge for it. I read the  34 :“Show how it is possible to determine the  35  of a tall building  36  the aid of a barometer (气压计).”The   37  answered:“ 38  the barometer to the top of the building, fix a long rope to it,  39  the   40  to the street, and then bring it up, measuring the length of the rope. The length of the rope is the height of the building.”

I pointed  41  that the student really had a strong reason for a full score, since he  42  the question completely and correctly. On the other hand,  43  a full score were given,  44  could well contribute to a high grade for the student in his  45  course. A high  46  is supposed to certify competence in physics, but the answer did not confirm (证实,确定) this. I suggested that the student have another try at answering the question. I was not surprised that the teacher agreed, but I was surprised that the student did.

I gave the student six  47  to answer the question, with the warning that his answer should show some knowledge of physics.  48  the end of five minutes, he had not written  49 . I asked if he wished to  50 , but he said no. In the next minute, he dashed off his answer, which was proper and made the teacher give up his thought, and I gave the student almost a full score.

31. A. and      B. for      C. so       D. while

32. A. return    B. form      C. receive    D. buy

33. A. built     B. founded    C. set up     D. put up

34. A. question   B. papers     C. exercise    D. sentence

35. A. length    B. height     C. width     D. area

36. A. with     B. under     C. by       D. through

37. A. teacher    B. student    C. paper     D. writer

38. A. Bring     B. Fetch     C. Heat      D. Take

39. A. lowering   B. lower     C. put      D. putting

40. A. rope     B. building    C. top      D. barometer

41. A. out      B. at       C. towards    D. to

42. A. answered   B. replied    C. had answered  D. replied to

43. A. even if    B. once      C. if       D. as if

44. A. it      B. which     C. what      D. as

45. A. maths     B. physics    C. chemistry   D. English

46. A. level     B. grade     C. mark      D. score

47. A. years     B. seconds    C. hours     D. minutes

48. A. In      B. By       C. To       D. At

49. A. anything   B. something   C. nothing    D. everything

50. A. give up    B. give in    C. give off    D. give out

Ⅳ. 阅读理解  (50分)

A

People celebrate birthdays in almost every country on Earth. And they celebrate them in many different ways.

In China and Japan,for example, the arrival of the New Year is more important then a person's birthday. So,everyone adds a year to their ages on New Year' Day, instead of on their birthdays.

In Brazil the special birthday food isn't cake, but special candies. In China, it's long noodles-for long life. In Iceland, pancakes with candies rolled inside them are served on birthdays.

In Great Britain, a birthday is all-day celebration. At school, the birthday kid is lifted up and "bumped" by his classmates. This means the person is lifted by the ankles and arms and bumped on the ground, once for each year and once for growth. In Israel, the same custom is followed, except the birthday person is raised and lowered while sitting in a chair.

In Holland, children not only get gifts, but they give things out, too. It's customary to give cakes, cookies, and candies to your classmates and teacher on your birthday.

In Mexico, it's good luck to give birthday greetings as early in the day as possible. So it's not unusual to be awakened at midnight by friends singing birthday sons.

In India, birthday celebrations also begin before dawn. The birthday is also a religious event, so prayers and blessings must be said before dawn. And the birthday person gets to stay home from school that day.

In Thailand, two tall candles are lighted the night before a birthday. One is as tall as the birthday person. It stands for how long the birthday person will live. If it goes out early, it's bad luck. Another Thai custom involves buying live fish and birds. The birthday person frees the animals, which brings luck.

In Nigeria, people don't celebrate individual birthdays. Instead, everyone in a certain age group celebrates together on a certain day.

   In may countries, birthday customs are changing. Old customs are giving way to Western style birthdays, complete with candle-topped cakes. But one thing never changes-Birthdays are special days for everyone!                     

51. It's a custom to lift and lower the birthday person in a chair in ______.

   A. Britain          B. Israel       C. Britain and Israel       D. Israel and Thailand

52. In Mexico the birthday greetings are usually given at midnight because _____.

A. birthday celebrations must begin before dawn

B. the birthday person must be awakened at midnight

C. prayers must be said before dawn

D. it will cause good things to happen to the birthday person

53. According to the passage, which of the following is NOT true?

A. In Western countries people eat a candle-topped cake at the end of the birthday party.

B. In Western countries a birthday cake is always prepared for a birthday person.

C. Every Japanese adds a year to their ages on New Year's Day.

D. More and more people celebrate their birthdays in Western style.

54. According to the passage, which of the following is true?

A. In Thailand people give fish and birds to the birthday person as gifts.

B. In Holland people give gifts to unbirthday persons.

C. In Nigeria people who were born in the dame period may celebrate their birthdays on the 

   dame day.

D. In Thailand a candle will be lighted on the birthday night.

B

In recent years, in the big cities of the country, there have arisen traffic problems, for cars, trucks, buses and bicycles have increased in number, size and speed. To make matter worse, the population is ever on the increase. As a result the streets are now more crowded than before, Therefore driving in streets today is not so safe as it used to be.

However, all efforts are made to reduce road accidents. Wherever possible, old roads and streets are given new surfaces and narrow roads are made wider. Building or trees that stand in the way of traffic are removed and remarkable road-signs are placed at the proper places.

It is important to see that people on foot are safe on the streets. There is no problem for them to cross the road at the crossing where are traffic lights. Crossing a busy road is difficult and dangerous as cars and trucks move on the streets almost without stopping.

Overhead bridges are built up above these busy roads in increasing numbers. People on foot may now use them to cross the roads while cars and trucks keep going on. The shortcoming is that people who are in poor health may feel it difficult to climb up and down the steps at each end of the overhead bridges.

55. It is difficult and dangerous to cross a busy road because     .

   A. the road is too smooth

   B. the road surface is too wide

   C. there are lots of cars and trucks coming and going continuously

   D. there are a large number of minibus

56. Overhead bridges are built     .

   A. across the crossroads     B. across the narrow roads

   C. across the busy roads     D. across the old streets

57. People on foot may now     .

   A. use overhead bridges to stop traffic flowing

   B. use overhead bridges to cross the roads safely

   C. feel it difficult to climb up and down the overhead bridges

   D. feel it easy to have a good view of the city

58. Persons in poor health feel that it is      to cross the overhead bridges.

   A. easy    B. difficult   C. joyful     D. possible

C

Scientists in India have invented a new way of producing electricity. Their invention does not get its power from oil, coal or other fuels(燃料). It produces electricity with the power of animals. India has about 80 000 000 bullocks(小公牛) which can do all kinds of jobs: working in the fields, pulling carriages through the streets and carrying water for villagers. Indian government has been looking for ways to use less imported(进口的) oil to provide energy. The scientists believe those millions of bullocks can help. One job done by bullocks is to pump water out of wells. The animals do this by walking around and around in a circle. As they walk, they turn a heavy stick that makes the pump move. This simple technology is centuries old, but the scientists thought it could be used to produce electricity. Bullocks walk in a circle only 2 or 3 times a minute. That is much too slow to produce electricity, but it can produce enough power to turn a set of gears(一组齿轮). A large gear sets next to a smaller one. As the large gear turns, it causes the smaller one to turn. That gear turns an even smaller one. Each gear moves faster because it is a little smaller. The smallest gear may turn very fast. Clocks operate with gears, so do cars and so does the device(装置) invented by the scientists to produce electricity.

59.From this passage we know that in the past India produced electricity mainly with    .

   A.local coal B.foreign oil  C.other fuels   D.the power of animals

60.Bullocks produce electricity in the same way as      .  

A.they work in the fields or on the farms

B.they pull carriages through the streets

C.they carry water for Indian villagers

D.they pump water out of wells for farmers

61.Why do the smaller gears move faster than the bigger ones?

A.Because the bigger gears want to synchronize(同步) with the smaller ones, and they 

  have to move more slowly than the smaller ones.

B.Because the smaller gears have to synchronize with the bigger ones.

C.Because the bigger gears are too big in size to move faster than the smaller ones.

D.Because the smaller gears needn't synchronize with the bigger ones.

62.How do bullocks produce electricity according to the passage?

A.Bullocks walk around and around slowly in a circle.

B.Bullocks run around and around rapidly in a circle.

C.Larger gears make smaller ones turn fast enough.

D.The largest gears have the smallest ones move very fast.

D

Hank Viscardi was born without legs. He had no legs, but stumps (残肢) that could be fitted with a kind of special boots. People stared at him with cruel interest. Children laughed at him and called him‘Ape Man’(猿人) because his arms practically dragged on the ground.

Hank went to school like other boys. His grades were good and he needed only eight years to finish his schooling instead of the usual twelve. After graduating from school, he worked his way through college. He swept floors, waited on table, or worked in one of the college offices. During all this busy life, he had been moving around on his stumps. But one day the doctor told him even the stumps were not going to last much longer. He would soon have to use a wheel chair. Hank felt himself go cold all over. However, the doctor said there was a chance that he could be fitted with artificial legs (假腿). Finally a leg maker was found and the day came when Hank stood up before the mirror. For the first time he saw himself as he had always wanted to be-a full five feet eight inches tall. By this time he was already 26 years old. Hank had to learn to use his new legs. Again and again he marched the length of the room and marched back again. There were times when he fell down on the floor, but he pulled himself up and went back to the endless marching. He went out on the street. He climbed stairs and learned to dance. He built a boat and learned to sail it.

When World War Ⅱ came, he talked the Red Cross into giving him a job. He took the regular training. He marched and drilled along with the other soldiers. Few knew that he was legless. This was the true story of Hank Viscardi, a man without legs.

63. It can be inferred from the story that five feet eight inches tall is     .

   A. an average height for a fully grown person

   B. too tall for an average person

   C. too short for an average person        

   D. none of the above

64. The sentence“he talked the Red Cross into giving him a job” implies that the Red Cross     .

   A. was only glad to give him a job

   B. gave him a job because he was a good soldier

   C. gave him a job after he talked to someone whom he knew in the organization

   D. was not willing to give him a job at first

65. When Hank marched and drilled along with the other soldiers, he     .

   A. did everything the other soldiers did     

   B. did most of the things the other soldiers did

   C. did some of the things the other soldiers did

   D. took some special training

66. The writer suggests that Hank Viscardi     .

   A. had no friends                 B. never saw himself as different from others

   C. was very shy                  D. was too proud to accept help from others

E

Michael, a typical (典型的) American, stays home on workdays. He plugs into his personal computer terminal in order to connect with the office. After work, he puts on his headphones, watches a movie on his home video recorder, or plays baseball on the computer. On many days, Michael doesn't talk to any other human beings, and he doesn't see any people except the ones on television.Michael is imaginary, but his lifestyle is very possible. The inventions of modem technology seem to be cutting us off from contacting with our fellow human beings.

The world of business is one area in which technology is isolating (隔绝)us. Experts say, for example, that many people will soon be able to work at home. With access to a large central computer, employees such as office clerks, insurance agents, and accountants could do their jobs at display terminals in their own homes. They would never have to actually see the people they're dealing with. In addition, the way employees are paid will change. Workers' salaries will be automatically paid into their bank accounts (账户), making paper checks unnecessary. No workers will stand in line to receive their pay or cash their checks. Personal banking will change, too. Customers will deal with machines to put in or take out money from their accounts.

Another area that technology is changing is entertainment. Music, for instance, was once a group experience. People listened to music at concert halls or in small social gatherings. For many people now, however, music is an individual experience. Walking along the street or sitting in their living rooms, they wear headphones to build a wall of music around them. Movie entertainment is changing, too. Movies used to be social events. Now, fewer people are going out to see a movie.

Many more are choosing to wait for a film to appear on television or are borrowing videotapes to watch at home. Instead of laughing with others, viewers watch movies in their own living rooms.

67. After work, Michael likes to       .   

   A. listen to music at the concert hall          B. watch a movie in his living room.

   C. run a program on his computer m his office  D. play baseball with his workmates

68. Which of the following is NOT mentioned in the passage?

   A. Clerks will be able to work at home.

   B. One can play baseball on the computer.

   C. One can listen to music without disturbing others.

   D. One can borrow books from libraries at home.

69.What will the author most probably discuss after the last paragraph?

   A. Games and sports.         B. Personal banking.

   C. Music and films.                   D. International business.

60.What is the main idea of the passage?

   A. We may no longer need to communicate with other human beings.

   B. Modem technology seems to be separating human beings.

   C. We may no longer need to work in the office.

   D. Modem technology makes it possible for us to work and entertain ourselves.

Ⅴ. 短文改错 。(10分)

   When Mr. Woolf left university, he got a good job

in a big company, but after he has been there for a        71.          

few years, he decided to have some change. He put an       72.          

ad (广告) in several newspapers, said what experience       73.          

he had , describing the kind of a job he had at           74.          

that time, and the kind he would like have. One of the       75.          

answer he got was from another man who was looking        76.          

for a job, either. The man wrote to him, “Dear Sir,        77.          

when you will get a new job, please be kind enough to       78.          

give my name and address for your present company,        79.          

because I've been trying to find a job as yours.           80.         

Ⅵ. 书面表达 。(30分)

根据提示和要求拟一口头通知稿。

1. 事件:美国一残疾人--史密斯来校演讲。

2. 10月25日下午2-4点,第二教学校201室。

3. 他将介绍他是如何克服困难的。他在9岁时,一次事故中失去左腿和右臂,通过努力他能生活自理,并学有所成。

4. 听完演讲分组讨论:如何鼓励、帮助残疾人,怎样向史密斯教授学习。

5. 词数100 左右

中 学 英 语 能 力                        

训练与测试系列   NSEFC- IIB-UNIT-T- 2005

             高中二年级单元目标测试卷

Unit 18  Inventions

Ⅰ. 单元要点练习。(15分) 

1.Creative thinking is a conscious effort to_____old thought patterns.

  A.get away from            B.break away from

  C.get off               D.break up with  

2.We must star early,to_____finding their house.

  A.allow      B.permit      C.allow for   D.permit for

3.Quite a few children in the country drop school,which is the serious problem that the government should______.

  A.aware of     B.be aware of   C.beware of    D.be beware of

4.It was raining heavily.Little Mary felt cold,so she stood______to her mother.(2002 Beijing)

  A.close      B.closely     C.closed      D.closing

5.I felt______when I found everyone had gone to the cinema without me.

  A.struck      B.stuck      C.rejected     D.refused

6.When the tiger was______the danger,it ran away as quickly as possible.

  A.aware of     B.aware about   C.aware for    D.awake about

7.His weakened condition would not______his being questioned by the police.

  A.allow for    B.allow of     C.allow      D.allow in

8.-I feel tired,dad.

  -Tired?______?

  A.For what     B.For which    C.From which    D.From what

9.Mr.Smith,_____of the_____speech,started to read a novel.(2003 Beijing)

  A.tired;boring   B.tiring;bored   C.tired;bored   D.tiring;boring

10.The blinking of the eyelid of the man is______.

  A.automatic    B.mechanical    C.mechanically   D.automatically

11.This train goes______between the two towns.

  A.backwards and forward       B.forwards and backward

  C.backwards and forwards       D.forward and backwards       

12.He hates the job—but he's determined to_____it_____because he needs the money.

  A.stick;at     B.stick;out    C.stick;to    D.stick;by

13.The collapse of the World Trade Center has put US economy in a difficult______.(2003 Shanghai)

  A.occasion     B.case       C.situation    D.background

14.The old man's two daughters and their______husbands went into the dining room.

  A.respectable   B.respected   C.respective    D.respectful

15.They have an______generator which uses oil and makes all the electricity for the farm.

A.electric     B.electrical    C.electricity   D.electrify

Ⅱ. 专项语法 (定语从句)  (15分)

16.We are living in an age________many things are done on computer.(2003北京春季高考题)

  A.which       B.that        C.whose       D.when

17.______has been announced,we shall have our final exams next month.(2003上海春季高考题)

  A.That      B.As        C.It        D.What

18.We will be shown around the city: schools,museums,and some other places,_____other visitors

  seldom go.(2002北京)

  A.what      B.which      C.where      D.when

19.Alec asked the policeman______he worked to contact him whenever there was an accident.(02上海)

  A.with him    B.who       C.with whom    D.whom

20.He made another wonderful discovery,________ of great importance to science.

  A.which I think is          B.which I think it is          

  C.which I think it          D.I think which is        

21.Then the great day came________he was to march past the palace in the parade.

  A.where      B.since      C.when       D.till

22.-I drove to Zhuhai for the air show last week.

  -Is that the reason________you had a few days off?

  A.why       B.when       C.what       D.where

23.Dorothy was always speaking highly of her role in the play,____,of course,made the others unhappy.

  A.who       B.which      C.this       D.what

24.The result of the experiment was very good,______we hadn't expected.

  A.when      B.that       C.which      D.what     

25.I shall never forget those years_____I lived in the country with the farmer,_____has a great

  effect on my life.

  A.that;which   B.when;which    C.when;who     D.which;that     

26.She is such a lovely girl________is liked by everybody.

  A.what      B.as        C.that       D.which        27.She likes to use words________are clear to him.

  A.of which the meanings        B.meanings of which

  C.whose of meanings                   D.meanings of whose

28.I,________ your friend,will help you out.

  A.who is     B.who am      C.that is     D.I am         29.I was so eager to leave that when I stood in the hall,ready to leave,I didn't even think of saying

  good-bye to the friends______I had eaten and slept for so many weeks.

  A.by whom     B.of whom     C.with whom    D.with which         30.Gone are the days________the working people suffered from cold and hunger.

   A.on that    B.at which    C.when       D.with which     

Ⅲ. 完形填空 。(30分)

Can you  imagine what life would be like if there were no telephone? You could not   31  

your friends on the phone and talk to them. If fire  32  out in your house, you  33  call the fire department. If somebody  34   sick, you could not call a doctor.

In our  35  life we need to communicate with  36  . We do this  37  by speaking to other people and listening to what they have to say to us, and when we are  38  to them we can do this very easily.  39  , our voices will not travel very far even when we  40  , and it is thanks to the   41  of the telephone that we are   42   able to communicate with each other  43  hold talks when we are far apart. We can hear each other   44   clearly as if we   65   in the same room.

The man who made this  46  was Alexander Grahan Bell, a Scotsman, born in Edinburgh in 1847. Bell, a teacher of visible speech (可见语言的), later moved to Canada,  47  all his spare time experimenting.   48   enthusiastic (热情的) was  he in his research for means for   49  by electricity that he left   50  time for his day-to-day work and at one time was almost penniless .

31. A. get on    B. listen     C. see      D. call up

32. A. broke     B. went      C. set      D. put

33. A. should    B. could     C. shouldn't   D. couldn't

34. A. is      B. are      C. becomes    D. were

35. A. daily     B. family     C. school     D. modern

36. A. them     B. another    C. the other   D. one another

37. A. only     B. mostly     C. quickly    D. well

38. A. speaking   B. listening   C. close     D. friendly

39. A. However    B. Of course   C. Therefore   D. For example

40. A. phone     B. shout     C. call      D. laugh

41. A. experiment  B. result     C. discovery   D. invention

42. A. not      B. hardly     C. yet      D. still

43. A. to      B. nor      C. and      D. but

44. A. as      B. so       C. not so     D. so much

45. A. speak     B. were      C. stay      D. worked

46. A. machine    B. possible    C. clear     D. telephone

47. A. having    B. spending    C. talking    D. using

48. A. Very     B. So       C. Much      D. Never

49. A. sending    B. giving     C. taking     D. setting

50. A. some     B. much      C. little     D. enough

Ⅳ. 阅读理解 。(50分)

A

Light skin, high cheekbones(颧骨), small nose-the results of plastic surgery(整形手术): this is the Michael Jackson that fans are familiar with today.

The self-claimed “King of Pop”was standing high on the platform at the American Music Awards in Los Angeles on January 9. He looked like someone who is out of touch with reality; someone whose riches and fame have led to a strange lifestyle for a strange man.

Jackson, 44, was honoured with the title “Artist of the Century”on the Awards night. The title is a witness (见证) to the success and controversy(争议) that have made his long career interesting.

In 1982, his record“Thriller”became one of the most successful albums(专辑) of all time. It sold more than 40 million copies worldwide. His following albums-especially“Bad”-have all enjoyed worldwide success.

As one of the world's most popular singers, Jackson has been a superstar since he was a child. But he has found his fame hard to handle. His face has been appearing in newspapers and magazines for decades. Jackson joined his brothers as a member of the influential R&B group. The Jackson Five, at the age of four, and he soon became the group's lead singer.

When other kids were playing, he was performing. When other kids were learning how to ride a bike, he was performing. When other kids started school, he was performing. His father forced him to stay in and rehearse(排练), rehearse, rehearse, record, record, record. Being a superstar for Michael Jackson is ordinary life-he doesn't know anything else. What is strange to most people might seem completely normal to him. Imagine never being able to completely normal to him. Imagine never being able to walk along the street without a crowd following you. So it's not surprising that he built his own house in the desert hidden by high fences.

He married Lisa Marie Presley, the daughter of Elvis Presley, in 1994. The marriage broke down 19 months later. According to his wife, Jackson never meets people without make-up on his face.

The plastic surgery he has had has brought criticism from across the world. As the young 4-year-old star of the Jackson Five, he had dark black skin, an Afro(非洲式的) face. He now looks totally different.

Despite the criticism and six years out of the spotlight, Jackson's latest album, “Invincible,”sold more than 350,000 copies in its first week of release.

Jackson has two children with his second wife, Debbie Rowe-Jackson. Some people believe that his wife is just a paid employee(雇员)-paid well enough to give him two children.

51.Jackson's face has been appearing in newspapers and magazines for decades        .

  A. because of his light skin, high cheekbones and small nose

  B. because he has been a superstar

  C. because of his riches and fame

  D. because of his good-looking face

52. Michael Jackson was standing on the platform in Los Angeles on January 9 to        .

  A. show that he is out of touch with reality

  B. perform for the audience

  C. attend the ceremony of the American Music Awards

  D. make more people know his latest album-Invincible

53. When people in the world learned the plastic surgery that Jackson has had,        .

  A. they show deep interest in it

  B. they are for what he has done

  C. they don't like him any more

  D. they don't agree with what he has done

54. The passage is mainly about       .

  A. Michael Jackson's way to success

  B. Michael Jackson's lifestyle

  C. Michael Jackson who lost himself in riches and fame

  D. Michael Jackson's album and his life

B

Most of the people who appear most often in the history books are great conquerors(征服者) and generals and soldiers, while the people who really helped civilization(文明) forward are often never mentioned at all.

We do not know who first set a broken leg, or made a seaworthy boat, or calculated(计算) the length of the years; but we know all about the killers and destroyers. People think a great deal of them, so much so that on all the highest pillars(柱子) in the great cities of the world you will find the figure of a conqueror or a general or a soldier. And I think most people believe that the greatest  countries are those that have beaten in the battle the greatest number of other countries and ruled over them as conquerors.

It is just possible they are, but they are not the most civilized. Animals fight; so do savages(野蛮人); so to be good at fighting is to be good in the way in which an animal or a savage is good, but it is not to be civilized. Even being good at getting other people to fight for you and telling them how to do it most efficiently -this, after all, is what conquerors and generals have done--is not being civilized. People fight to settle quarrels. Fighting means killing, and civilized people ought to be able to find some way of settling their quarrels other than by seeing which side can kill off the greater number of the other side, and then saying that the side which has killed most has won, and not only has won, but, because it has won, has been in the fight. For that is what is going to mean in a war; it means saying that might is right.

55.Most people believe that the greatest countries are those that       .  

  A. built the highest pillars

  B. were beaten in battle by the greatest number of other countries

  C. were ruled by the greatest number of conquerors

  D. won the greatest number of battles against other countries

56.In the author's opinion, the countries that ruled over a large number of other countries are      .

  A. possibly the most civilized but not the greatest

  B. possibly the greatest in some degree but not the most civilized

  C. certainly not the greatest in any way

  D. neither the greatest nor the most civilized

57.The author says that civilized people should        .

A.not have any quarrels to settle

B.not fight when there are no quarrels to settle

  C. settle their quarrels without fighting

  D. settle their quarrels by seeing which side can kill off the greater number of the other side

58."Might is fight" in the last sentence means that        .

  A. those who fight believe that the winner is fight and the loser wrong

  B. only those who are powerful should go to war

  C. those who are fight should fight against those who are wrong

  D. in a way only those who are powerful will win

C

New has come from America that we shall soon have no more children. The idea that children are different sorts or people from adults, says Mr Neil Postman, has not always been around. In Medieval Times(中世纪) most children did not go to school, but joined in adult life entertainment(娱乐).

Then printed books were invented and became common. (Before that, the only books were copied by hand, and there were very few of them.) With printing, all families could have books, this meant that adults could get knowledge. Children could not, since the children had to learn to read first. So some knowledge became unsuitable for children. They were no longer just small grown-ups, but had to learn to become grown-ups.

"All this," says Mr Postman, "is now changing." Printing created children, but television is destroying it once again. No one needs to learn to watch television. All knowledge is there for everyone to have. We are going to return to a medieval life-style, and children are once more going to be a part of the adult world.

59.According to the passage, most children in Medieval Times       .

   A. could go to school they could           B. were quite different from their parents

   C. had actually been grown-ups             D. took part in all the activities like their parents

60.We can conclude from the passage that        .             

   A. printing make it easy to tell children from adults

   B. printing makes people's life hard

   C. children can't have become adults since printing was invented

   D. people will go back to the Medieval Times

61.After television is widely used, the idea of children       .

   A. will not be popular                B. will disappear in our life

   C. will be remembered in people's mind       D. will not be fit for modem society

62.This passage suggests that        .

   A. modem children can know what the adults know by modem machines

   B. modem children don't know what happened in the past

   C. modem world is warmly welcomed by children

   D. adults won't be born with children any longer

D

Ever since man began to use the telephone, there have been new problems arising from the carrying of messages. At first, each message was carried by a pair of overhead wires. As a result, telephone exchanges were soon surrounded by thousands of wires. The wires were then replaced by cables (电缆) , each containing many pairs of wires. Each cable, laid underground replaced the overhead wires.

The more extensive (广泛的) telephone services have become, the more demand for these services has increased, particularly the demand for long-distance services. In China, for example, this growth is now over 30% every year. Long-distance telephone exchanges are usually in crowded cities, it is not easy to lay new cables for expanding services.

The use of radio to send telephone messages and to link all telephone exchanges makes it possible to get rid of overhead wires and some underground cables.

Now man has invented the microwave system. In a microwave system messages from various places can be brought together, and then they are sent out and received by radio. After that, the messages are split into their original form. Finally, they are sent to the places where they are going to be sent.

63. A cable is different from an overhead wire, because it       .

   A. is made of wood                 B. needs more exchanges

   C. carries more messages     D. is much longer

64. In the last line of paragraph 2, “expanding”means       .

   A. growing   B. holding    C. using     D. understanding

65. According to the passage, microwave is       .

   A. used in communication     B. to replace the telephone

   C. sent through an underground cable        D. made of radio sets

66. The best title of the passage would be       .

   A. The Telephone and Its Pas            B. How to Use the Telephone

   C. The Needs for Better Telephones       D. From the Wire to the Microwave System

E

FITNESS CENTER TIMETABLE

Body Power:This class uses weights and aerobic movements (a form of very active physical exercise)to increase strength while burning fat. It is suitable for all levels for fitness.

Aerobics:This is a low to middle level of difficulty in aerobic class. It is good for beginner to average levels of fitness.

Beginner Aerobics :This class has a low level of difficulty and simple aerobic movements. It is mainly for people who have not exercised before or who are just returning after a long break of  injury.

Body Stretch  (hold your arms or legs out straight and tighten your muscles):This is a relaxation

     class which stretches all the muscle groups. It is suitable for all levels of fitness. People who

     exercise  regularly or have a stressful (压力) lifestyle would also greatly improve their health  

      in this class.

Body Burn:This is a high level of difficulty in aerobic class that mainly aims at fat burning. It is only for those people with a very high level of fitness.

67.On Wednesday the fitness class at 6 : 30pm is      .

  A.Aerobics      B.Body Power   C.Body Burn     D.Beginner Aerobics

68.The two classes that are suitable for all levels of fitness are      .

  A.Aerobics and Body Power       B.Body Power and Body Stretch     

C.Body Burn and Aerobics       D.Body Stretch and Beginner Aerob ics

69.Lena wants to go to the City Gym on Saturday. She has not exercised for a long time. The best time for her to go would be       .

  A.3 : 00pm      B.5 : 30pm    C.7 : 15pm     D.7 : 30pm

70.The main goal of the Body Burn is to      .

A.reduce stress     B.reduce fat   C.increase strength         D.increase muscle size

Ⅴ. 短文改错 。(10分)

   Television has changed our life. It is not           71.        

only gives the news quickly, but also shows             72.        

them in pictures --more interesting and              73.        

lively as words. No newspaper has ever               74.        

reached so many people and show so                 75.        

clearly what happening right now in                 76.        

their countries and in everywhere else. We              77.        

can also learn difference subjects over TV              78.        

without leave our houses. Teaching can be              79.        

given on TV easily to thousands people.               80.       

Ⅵ. 书面表达 。(30分)

    光明中学依山傍水。校园曾经安静美丽。一年前该校附近建立了一家化工厂。你是该校一名学生,名叫李明,现请你根据下表的内容给本市一家报刊编辑写封信,反映你校受污染问题,呼吁有关部门采取措施,防止污染,词数:80-120。   

中 学 英 语 能 力                        

训练与测试系列  NSEFC- IIB-UNIT-T- 2005

           高中二年级单元目标测试卷  

              Unit 19  The Merchant of Venice  

Ⅰ. 单元要点练习。15分 

1.They will never  get down_____before the enemies.

  A.on their knees           B.on their knee

  C.to their knees           D.to their knee

2.This kind of dress,I think,with many flowers on,_____us women teachers.

  A.doesn't fit             B.isn't suit for

  A.doesn't suit            D.doesn't fit for 

3.He will surely finish the job on time______he's left to do it in his own way.

  A.in that     B.on condition that         C.in case     D.as far as

4.We promised not to quarrel______.

  A.any more     B.any long     C.not any more   D.no longer

5.The match was______of the weather.

  A.in charge    B.in control    C.at the mercy   D.for the mercy

6.China is really developing rapidly______the world.

  A.in the eyes of           B.in view of

  C.in charge of            D.in the place of 

7.Which of the following is not right?

  A.I envy his success.

  B.I envy him his success.

  C.I envy his having read the novel.          

  D.I envy him for having read the book.                     

8.What can we do to keep him_____caught by the  enemy?

  A.to be      B.for being    C.from being    D.from to be

9.The judge ordered that the stranger______all the lose of the damage.

  A.paid for     B.pays for    C.should pay for  D.had paid for

10.His bevavior at the dinner party last night seemed rather______.Many of us were very surprised.

  A.out of place   B.by request    C.out of bad manners         D.out of order

11.I broke Mary's pen by accident,and______she tore up my school work.   

  A.revenge     B.paying back  C.in revenge    D.avenge

12.We asked our father______off smoking.

  A.to swear cutting B.to swear to cut C.swearing to cut D.swaering cutting

13.He sent her off a telegram______his immediate return to London.

  A.announcing    B.to announce   C.declared     D.to declare

14.I'd like to buy a house-moderm,comfortable,and______in a quiet neighbourhood.(2004 Fujian)

  A.in all      B.above all    C.after all   D.at all

15.In dealing with public relations,we should make every effort to prevent the______in personality.

   (2004 Shanghai)

  A.contact     B.contrast     C.connection    D.conflict

Ⅱ. 语法 (从句中的动词变化)。15分

16. I would love ____ to the party last night but I had to work extra hours to finish a report.

  A. to go      B. to have gone  C. going     D. having gone

17.Having passed all the tests, she felt great weight ____ off her mind.

  A. taking     B. taken      C. take      D. to be taken 

18.He was very ____ to find his suggestion ____.

  A. disappointed; to be turned down  B. disappointing; turned down

  C. disappointed; turned down     D. disappointing; turning down

19.I can hardly imagine Peter ____ across the Atlantic Ocean in five days.

  A. sail      B. to sail     C. sailing    D. to have sailed

20.They know her very well. They had seen her ____ up from childhood.

  A. grow      B. grew      C. was growing  D. to grow the mail

21.When I came in, I saw her ____ in the sofa deep in thought.

  A. sat       B. seated     C. seating    D. being sitting

22.I won't have you ____ up and down all day.

  A. ran       B. running     C. run      D. to run

23.It was getting dark, I found a car ____ in a pool by the side of the road.

  A. stick      B. sticking    C. stuck     D. to be stuck

24.The glass of water is too hot. I prefer some cold ____ water.

  A. boiling     B. boiled     C. having boiled D. to boil

25.The form needs ____ in ink.

  A. filling in   B. to fill in   C. being filled in  D. to be filled

26.They are turned around in huge machines to get them ____ the forces they will experience in    

  space flight.

  A. use       B. to use     C. used to    D. be used to

27.She was glad to see her child well ____.

  A. take care of            B. taken care of

  C. taken care             D. taking care of

28.I had rather have trouble and hardship like that than ____ care of by others.

  A. be taken    B. being taken   C. taken     D. taking

29.It was not until she had arrived home ____ remembered ____ the key in her office.

  A. when she; to leave                 B. and she; leaving

  C. that she; to have left       D. she; to leave

30.The present situation there is more serious than ____.

  A. expected              B. expecting

  C. expect               D. has been expected

Ⅲ. 完形填空。30分

   Some personal characteristics play an important role in the development of one's intelligence. But people fail to realize the importance of training these factors in young people.

   The so - called 'non-intelligence factors' (非智力因素)include  31  feelings, will, motivation(动机),interests and habits. After a 30-year follow-up study of 8000 males, American psychologists (心理学)   32   that the main cause of disparities in intelligence is not intelligence  33   , but non-intelligence factors including the desire to learn, will power and self-confidence.

      34  people all know that one should have definite objectives, a strong will and good learning habits, quite a number of teachers and parents don't pay much attention to  35  these factors.

   Some parents are greatly worried   36   their children fail to do well in their studies. They blame either genetic (遗传的)factors, malnutrition (营养不良) or laziness, but  they never take   37  consideration these non-intelligence factors. At the same time, some teachers don't inquire into these, as reasons   38  students do poorly. They simply give them more courses and exercises, or  39  criticize or laugh at them. After all, these students lose self-confidence. Some of them just feel defeated and  40  themselves up as hopeless. Others may go astray (迷途)because they are sick of learning.   41  investigation of more than 1,000 middle school students in Shanghai showed that 46.5 per cent of them were  42  of learning, because of examinations, 36.4 per cent lacked persistence, initiative (主动)and consciousness (正直地、谨慎的) and 10.3 per cent were sick of learning.

   It is clear    43    the lack of cultivation (培养)of non-intelligence factors has been a main    44  to intelligence development in teenagers. It even causes an imbalance between physiological(生理的)and   45  development among a few students.

   If we don't start now to   46   the cultivation of non-intelligence factors, it will not only affect the development of the  47  of teenagers, but also affect the quality of a whole generation. Some experts have put forward  48  about how to cultivate students--non-intelligence factors.

   First, parents and teachers should   49   understand teenage psychology. On this basis, they can help them to pursue (调动) the objectives of learning,   50   their interests and toughening their willpower.

31. A. one's    B. their     C. his      D. her

32. A. came out   B. found out   C. made out       D. worked out

33. A. in itself  B. by itself   C. itself    D. on its own

34. A. Though    B. Nevertheless C. However    D. Moreover

35. A. believing  B. studying   C. cultivating     D. developing

36. A. about    B. when     C. how      D. whether

37. A. for     B. in      C. into     D. over

38. A. why     B. that     C. when     D. how

39. A. ever     B. even     C. still     D. more

40. A. put     B. get      C. handle    D. give

41. A. The     B. An      C. Another    D. A

42. A. afraid    B. ahead     C. aware     D. ashamed

43. A. that     B. how      C. why      D. which

44. A. difficulty  B. question   C. threat    D. obstacle(障碍)

45. A. intelligent B. characteristic C. psychological D. physical

46. A. practise   B. thrust    C. strengthen      D. urge

47. A. intelligence B. diligence   C. maturity(成熟)   D. performance

48. A. projects   B. warnings   C. suggestions  D. decision

49. A. fully    B. greatly    C. very     D. highly

50. A. insuring   B. going     C. encouraging  D. exciting

Ⅳ. 阅读理解50分

A

We may be very pleased with the rapid progress we have made in every field of study, but the way to test a student's knowledge and ability (能力) still remains as poor as it was. We have almost done nothing to improve our examination system .

It is well known that the examination system we are now using may be a good way of testing a student's memory, but it can tell you nothing about a student's ability. It does no good to students and teachers.

As soon as a child begins school, he enters a world of examination that will decide his future or job.

In fact a good examination system should train a student to think for himself, but it now does anything but that. So students are encouraged to remember what is taught. It doesn't enable them to gain more and more knowledge. The students who come out first in the examination often may not be the best in their studies.

Besides, the examinations often drive teachers to cram (填鸭式地教) all the time. They can't teach freely. They are forced to train students what to do with the coming examination from time to time.

There must be some simpler and better way to test a student's true ability besides knowledge.And that is what we should do at once.

51.Now little has been done       .

  A. to decide a student's job     B. to test a student's ability

  C. to test what a student learns   D. to test a student's memory

52.What are examinations very important to according to the article?

  A. A student's fame (声誉).     B. A student's power.

  C. A student's future.                 D. A student's achievements.

53.The examination system we are now using can only make the students       .

  A. remember what is taught      B. gain more and more knowledge  

  C. think for themselves                D. make rapid progress

54.This article tells us about the importance of       .

  A. taking examinations                 B. testing a student's knowledge

  C. working hard           D. getting the examination system improved

B

    Engineering students are supposed to be examples of practicality and rationality (理性), but when it comes to my college education I am an idealist and a fool. In high school I wanted to be an electrical engineer and, of course, any reasonable student with my aims would have chosen a college with a large engineering department, many famous professors and lots of good labs and research equipment. But that's not what I did.

   I chose to study engineering at a small liberal-arts (文科) university that doesn't even offer a major in electrical engineering. Obviously, this was not a practical choice; I came here for more noble reasons. I wanted a broad education that would provide me with flexibility (灵活) and a value system to guide me in my job. I wanted to open my eyes and expand my vision (眼界) by communicating with people who weren't studying science or engineering. My parents, teachers and other adults praised me for such a wise choice. They told me I was wise and grown - up beyond my 18 years, and I believed them.

   I headed off to the college and sure I was going to have an advantage over those students who went to big engineering "factories" where they didn't care if you had values or were flexible. I was going to be a complete engineer: technical expert and excellent humanist all in one.

   Now I'm not so sure. Somewhere along the way my noble ideas crashed into reality, as all noble ideas finally do. After three years of struggling to balance math, physics and engineering courses with liberal-arts courses, I have learned there are reasons why few engineering students try to reconcile (协调) engineering with liberal-arts courses in college.

    The reality that has blocked my path to become the typical successful student is that engineering and the liberal arts simply don't mix as easily as I supposed in high school. Individually they shape a person in very different ways. The struggle to reconcile the two fields of study is difficult.

55.Why did the author choose to study engineering at a small liberal-arts university?

  A. He intended to become an engineer and humanist.

  B. He intended to be a reasonable student with noble ideals.

  C. He wanted to be an example of practicality and rationality.

  D. He wanted to communicate with liberal-arts students.

56.According to the author, by communicating with people who study liberal arts, engineering  

  students can        .

  A. become noble idealists      B. broaden their knowledge

  C. find a better job in the future  D. balance engineering and liberal arts

57.When the author says "Engineering students are supposed to be examples of practicality and

  rationality, but when it comes to my college education I am an idealist and a fool" at the

  beginning of the passage, he actually means          .

  A. he has failed to achieve his ideal aims

  B. he is not a practical and rational student

  C. his choice of attending to a small liberal-arts university is reasonable

  D. his idea of combining engineering with liberal - arts is noble and wise

58.The author suggests in this passage that         .

  A. liberal-arts students are supported to take engineering courses

  B. technical experts with a wide vision are expected in the society

  C. successful engineering students are more welcomed in the society

  D. engineering universities with liberal-arts courses are needed

C

A Chinese scientific expedition left Beijing Tuesday morning for the North Pole to build the nation's first research station in Norway's Svalbad Islands within the Arctic Circle.

The 17-member team of scientists and journalists is expected to conduct a 20-day research project in the North Pole area and build the China Yilite-Morning Arctic Scientific Expedition and Research Station there.

At the conclusion of the three-year project, the longest of its kind made by China, the new, advanced research station will provide Chinese scientists with accommodation(住所) and continuous monitoring and communication capabilities, said Gao Dengyi, the team leader and a scientist on  atmospheric(大气的) physics from the Chinese Academy of Sciences(CAS).

Scientists will conduct through research on the region's climate, environment and resources. They will also gain experience and prepare themselves for the future construction of a lasting scientific research station.

As the Arctic region is one of the world's most sensitive areas to the global climate and environmental change, a long-term scientific study of the area's weather, environment, resources and biology is important, Gao said. After a brief stay in the Norwegian city of Troms, also within the Arctic Circle, the group will arrive in the Svalbad Islands Wednesday and start their construction work there. Being the closest polar region to China, the Arctic has long been viewed by Chinese scientists as important to the country's development.

Ye Du, a scientist on aerology(高空气象学) from the CAS, said that the  vortex(旋涡) flow change of the region may have a strong influence on the winter weather in northern China. Also, the damage of the ozone(臭氧) may seriously have effect on China's climate.

The study of the North Pole, regarded by the world's leading academics to be the “science of the future,”can help mankind gain a better understanding of the earth and outer space. Global organizations like the United Nations, the Intergovernmental Committee on Climate Change and the International Research Committee of the Arctic, have all classified the scientific study on the Arctic as the world's priority(重点).

59.The author wrote the passage to       .

  A. stress the importance of China's setting up research station at the North Pole

  B. show that the Arctic region is one of the world's most sensitive areas to the globe climate

  C. make it known that the North Pole will see the world's first research station there

  D. tell us China is to set up its first research station at the North Pole

60. Chinese scientists have long been viewing the Arctic because        .

  A. people know little about the region

  B. the polar region has great effect on the development of China

  C. the climate in the polar affects China all the year round

  D. the study of the area is regarded as the science of future

61. The study of the North Pole is regarded by the world's leading academics to be the “science of the future,”because       .

  A. it helps better understand the earth and outer space

  B. the Arctic region has effect on the global climate and environment change

  C. people have known little about the North Pole region

  D. global organizations like the United Nations and the International Research Committee of the Arctic have all classified the scientific study on the Arctic as the worldn's priority

62. After reading the passage we know       .

  A. the Chinese scientific expedition is made up of 17 scientists

  B. Troms lies in the Arctic Circle, belonging to Sweden

  C. the winter weather in northern China has something to do with the vortex flow change of the Arctic region

  D. the damage of ozone only affects China's climate

D

JerusalemA U-led commission began investigating(调查) 10 weeks of Middle East blood-shed, yesterday, starting talks with Israeli and Palestinian leaders.

The five-member international team, led by former US Senator George Mitchell, met Israeli Prime Minister Ehud Barak in Jerusalen and was due to meet Palestinian President Yasser Arafat in the Gaza Strip later in the day.

“We don't want to be part of a problem. We want to be part of a solution(解决) to a problem,”European Union foreign policy chief Javier Solana, a member of the commission, told reporters.

The US-led commission is going ahead despite the resignation(辞职) of Barak, who will remain care-taker prime minister until a new government is formed.

Solana said the team aimed to make a study at the situation with the two sides, to meet the leaders of Egypt and Jordan this week, and to make suggestions to the US President by the end of March. Israel and the Palestinians agreed to the creation of the commission at a peace summit in mid-October that failed to end the violence(暴力).

At least 310 people have died since the start of the Palestinian uprising for independence in the Gaza Strip and West Bank-259 Palestinians, 13 Israeli Arabs and 38 Israeli soldiers and other Israelis.

Palestinians, the UN and human rights groups have  accused(控告) Israel of using excessive (过度的) force. Palestinians want the committee to determine who is to blame and to take action.

Israel says its soldiers act only to defend themselves and Israeli civilians. It accuses Arafat of giving a green light to the violence.

It is said that Israeli Foreign Minister Shlomo BenAmi would fly to Paris to try to prevent the UN from deploying(部署) international observers who Israel says will do more to prevent peace than to keep it.

63. From the passage, we know that      .

  A. Barak is going to form a new government

  B. Barak has resigned from his position, and a new government has been formed

  C. Barak remains care-taker prime minister though he has already resigned

  D.Barak's resignation will lead to the stop of clashes

64. Both Palestine and Israel       .

  A. not only agreed to the creation of the commission, but also agreed who was to blame

  B. had already held a peace summit but failed to end the violence

  C. had some civilians killed, the majority of whom are Israeli soldiers

  D. wanted the UN to deploy international observers

65. The underlined phrase giving a green light probably means        in Chinese.

  A. 准许      B. 对抗      C. 利用      D. 阻止

66.The US-led commission is going to do the following things except to       .

  A. meet not only the leaders from Palestine and Israel but also leaders from Jordan and Egypt

  B. advise the UN to deploy international observers

  C. try to find a way to solve the problem

  D. make some suggestions to the US President

                                           E

   An 18th--century statesman Edmund Burke once said, "All that is needed for the success of a misguided cause is that good people do nothing. " One such cause now seeks to end biomedical research because of the theory that animals have rights to decide whether to be used in research. Scientists need to respond forcefully to animal rights advocates (鼓吹者) , whose arguments are puzzling the public and threatening advances in health knowledge and care. Leaders of the animal rights movement attack biomedical research because it depends on public funding, and few people understand the process of health care research. Hearing false reports of cruelty to animals in research settings, many are puzzled that anyone would harm an animal on purpose.

   For example, a grandmotherly woman setting up an animal rights booth at a recent street fair was giving out sheets that encouraged readers not to use anything that comes from or is tested in animals--no meat, no fur, no medicines. Asked what will happen when epidemics (传染病) return, she said, "Don't worry, scientists will find some way of using computers." Such well-meaning people just don't understand.

   Scientists must communicate their message to the public in an understandable way--in human terms, not in the language of biology. We need to make clear the connection between animal research and a grandmother's organ replacement, a father's bypass operation, a baby's vaccinations (接种疫苗), and even a pet's shots. To those who know nothing about the animal research that was needed to produce these treatments, as well as new treatments and vaccines, animal research seems wasteful at best and cruel at worst.

   Much can be done. Scientists could give middle school lessons and present their own research. They should be quick to respond to letters to the editor, in case animal rights misinformation goes unchallenged and gains a false appearance of truth. Research institutions could be opened to tours, to show that laboratory animals receive humane care. Finally, because there are a lot of patients, the health research community should actively admit to its cause not only well-known personalities such as Stephen Cooper, who has made courageous statements about the value of animal research, but all who receive medical treatment. If good people do nothing, there is a real possibility that uninformed citizens will wipe out the precious of medical progress. 

67.The purpose of the author beginning his article with Edmund Burke's words is to         .

   A. call on scientists to take some actions

   B. argue against the cause of animal rights

   C. warn of the failure of biomedical research

   D. show the success of the animal rights movement

68.In the eyes of the author, misguided people are likely to think that using an animal in research

   is          .

   A. cruel but necessary       B. justice but unnecessary        

   C. meaningful and wasteful     D. inhuman and unacceptable        

69. What can be inferred from the passage?

   A. People care very little about returns of epidemics.

   B. The public lacks knowledge of biomedical science.

   C. Stephen Cooper is very seriously concerned about animal treatment.

   D. Scientists should make efforts to develop new cures by means of hi-tech.

70.The underlined word "threatening" in paragraph 1 means         .

   A. doing a favour to           B. drawing attention to 

   C. causing a danger to.          D. making their way to

Ⅴ. 短文改错。10分

Two Germany tourists in Spain were having                           71.        

great language trouble in a cafe (咖啡馆) with            72.        

waiters. A glass of milk was that they wanted,            73.        

but they were unable to make the waiter understood.         74.        

    At last one of them took out of his pen and drew       75.        

a cow in a piece of paper. The waiter smiled,            76.        

nodded and dashed off. He returned back               77.        

with two tickets to the bullfight (斗牛比赛). At           78.        

the sight of the two tickets they were dumbfound,                      79.        

knowing what use there was in having the two tickets.        80.       

Ⅵ. 书面表达。30分

    在日常生活中,因特网起越来越重要的作用。请根据下表所给提示为某英文报写一篇题为 On the Internet 的征文稿。

英特网的主要用途

注意:1. 征文稿必须包括表内所有信息,行文连贯通顺;

       2. 词数 100 左右;

       3. 标题已给出,不计入总词数。

   

中 学 英 语 能 力                        

训练与测试系列  NSEFC- IIB-UNIT-T- 2005   

           高中二年级单元目标测试卷

                   Unit  20  Archaeology

Ⅰ. 单元要点练习。15分 

1.______money we're quite rich,but not______happiness.

  A.In terms of;in terms of       B.In turn;in turn

  C.In the eyes of;in the eyes of    D.In terms of;in the eyes of

2.My companion sat silent now for some time,_____in thought.

  A.gone       B.missing     C.lost       D.losing

3.He depends on his parents_____himself.

  A.rather than on  B.instead of    C.in place of   D.take the place of

4.I know he still has some money______.So I bet he couldn't______the robbery.

  A.in hand;have hands in        B.in hand;have a hand in       

  C.at hand;have the hand in      D.at hand;have a hand in

5.The engine of the ship was out of  order and the bad weather______the helplessness of the crew at sea.(2003 Shanghai)

  A.added to     B.resulted from  C.turned out    D.made up

6.-Do you like______here?

  -Oh,yes.The air,the weather,the way of life,everything is so nice.(2004 NMET)

  A.this       B.these      C.that       D.it

7. I like _______ in the autumn when the weather is clear and bright. (NMET Hunan)

  A. this                B. that       C. it                D. one

8.When you are sending email to me,please_____your

  A.take;to     B.put;to      C.send;from    D.address;to

9.During his studies,he_____several new facts about the history of the place.

  A.uncovered    B.unearthed   C.found of     D.invented

10.The old temple______the 19th century is being rebuilt.

  A.being dated back to                 B.to being dated from

  A.dating back to           D.going back in time to       

11.-Will you please help me to repair the computer now?

  -Sorry,I'm busy and haven't a minute to______.(2000 Shanghai)

  A.save       B.spare      C.spend      D.share

12.The price of the decorations to be sold in the supermarket is too______for these customers.

  A.expensive    B.high       C.dear       D.highly

13.Before the______of gunpowder,men fought with bows ans arrows.

  A.discovery    B.invention    C.creation     D.finding

14.Mother sent the food on the table and told the children to______.

  A.dig in      B.dig for     C.dig out     D.dig up

15.These trees cannot be grown in such a cold______as ours.

A.weather     B.climate     C.season      C.space

Ⅱ. 语法专练 ( it的用法) (15分)   

16.Was it during the Second World War_____ he died?        

  A.that       B.while       C.in which     D.then       

17.Is ______ necessary to complete the design before National Day?

  A.this       B.that       C.it        D.he    

18.I don't think ______ possible to master a foreign language without much memory work.      

           A.this       B.that       C.its      D.it   

19.It was not _____ she took off her glasses _____ I realized she was a famous film star.  

  A.when , that   B.until , that   C.until , that   D.when , then 

20.I was disappointed with the film .  I had expected ______ to be much better.

  A.that       B.this       C.one        D.it  

21.It was only when I reread this poems recently _____ I began to appreciate their beauty.

  A.until       B.that       C.then       D.so 

22.I hate_____when people talk with their mouths full.

  A.it        B.that                 C.these     D.them 

23.It is the ability to do the job _____ matters not where you come from or what you are.

  A.one        B.that       C.what       D.it 

24.It's demanded that we    there on foot.

  A.not to go    B.don't go     C.not go      D.won't go

25."It" is often used to       a baby.

  A.mean to      B.stick to     C.point to     D.refer to

26.    you met the Englishman?

  A.Where it was that          B.Who it was that

  C.Where was it that          D.Where was that

27.    in 1914     the First World War broke out?

  A.Was that, that            B.Was that, when

  C.Was it. that             D.Was it, when

28.It is important that she     with Mr Williams immediately.

  A.speak       B.spoke       C.will speak     D.to speak

29.    that there's another good harvest this year.

  A.It says      B.It is said    C.It was said    D.He was said

30.It's no use     over spilt milk.

  A.cry        B.crying      C.that you cry    D.for you to cry

III.  完形填空(共20小题;每小题1.5分,满分30分)

   The Chinese Antarctica (南极洲) expedition returned to Shanghai on March 20  31   a month living on the ice. It was the first time Chinese scientists  32   the Amery Ice Shelf (埃默里冰架)  33   in the world.

   The team collected ice samples (标本) and data on temperatures,  34  necessary for research on global warming.

   So far, about 27 countries   35  Antarctica for scientific research.   36  the living conditions are very hard, many scientists have to stay there for long periods of time.   37   how do they manage their lives on the ice?

   Well, during the summer, the temperature in Antarctica   38   about -20℃. It falls to about -60℃ in winter. So, even if there are 24 hours of sunshine, scientists must   39  wearing all the correct clothes to protect them from the cold.

   Although  40  technical clothing provides much protection against the extreme cold, scientists often need to go outside  41   there is an icy wind blowing and the temperature is -30℃ .

    "That's   42  many researchers wear a beard. It really   43  keep you warm," says German explorer Arved Fuchs

   Usually, the polar researchers have good meals. Some even have   44   on holidays like Christmas and New Year's Day.

   Life on the ice   45  living with other researchers in tight living spaces.

Most people sleep in dorm (宿舍) style rooms. More remote (偏远的) field camps use  46  tents. So, at least some researchers don't have to  47  annoying snorers (讨厌的打鼾者).

   It is important for a polar researcher   48  good physical condition. They can be  49  to climb to heights of more than 3,000 metres. And the Antarctic atmosphere has  50   .

31. A. followed    B. following    C. to follow         D. had followed

32. A. has visited  B. have visited           C. had visited        D. visited

33. A. the longest ice shelf                B. what is the longest ice shelf

   C. that is the longest ice shelf            D. which the longest ice shelf

34. A. which is    B. what are    C. those are         D. which are

35. A. had gone    B. have been    C. have visited       D. visited

36. A. Although    B. As        C. However          D. So

37. A. And      B. So        C. Because          D. But

38. A. arrives     B. falls      C. reaches           D. takes

39. A. keep      B. avoid      C. refuse            D. feel like

40. A. high      B. so       C. such             D. highly

41. A. sometime    B. when       C. where            D. sometimes

42. A.because     B. how       C. why              D. that

43. A.did       B. doesn't     C. do               D. does

44. A.feasts     B. teas      C. coffees           D. drinks

45. A. refers     B. remains     C. means            D. explains

46. A. common     B. shared      C. sharing           D. separate

47. A. put up with  B. put a stop to  C. pay attention to    D. take care of

48. A. being for   B. to be in    C. to have in         D. having for

49. A. advised     B. agreed      C. required          D. suggested

50. A. less oxygen than that of any other continent

   B. less oxygen than one of any other continent

   C. less oxygen than that of any continent

   D. as much oxygen as that of any other continent

第三部分:阅读理解(共20小题:每小题2.5 分,满分50分)

A

   13 years ago, William Stiles, an expert in American Indian history, discovered the treasure as you see here :a 1996 Volvo.

   He bought it because ads of the Times said Volvos were so tough that they fasted an average of 11 years in Sweden.

   As Mr. Stiles remembers: One ad said that a Volvo was so tough that you could "Drive it like you hate it." I did exactly that. In my field work, I've driven this car 295,000 hard miles. Much of it through former Indian countryside. It is held up even better than promised. Driving it like I hated it made me love it.

   Expressions of love are not uncommon among Volvo owners. In fact, 9 out of 10 people, who have bought new Volvos are happy.

   So if you are unhappy with your present car, do what Mr. Stiles once did after reading one of our ads. Buy one of our cars.

51.William Stiles bought a 1996 Volvo because ______.

   A. he was promised that it wouldn't easily be worn out

   B. he learnt that it was as modern as any of its kind.

   C. he knew that it was the best seller in the Swedish markets.

   D. he was fooled into believing that it worked well

52.After buying the car, William Stiles _____.

   A. drove it whether he felt unhappy with it

   B. had it tested in different kinds of conditions

   C. drove it madly across the country as if he hated it

   D. tested it in his field work to see how long it could last

53.The underlined part in the third paragraph "I've driven this car 295, 000 hard miles. " implies that

   ______.

   A. he drove a long way completely tired out

   B. the road conditions were unusually bad

   C. life was hard during his long ride

   D. he had much trouble all the way

54.William Stiles came to love the car because ______.

   A. he had found some other good points of the car

   B. At proved very useful in his field work

   C. it had lasted longer than 11 years

   D. more Volvo owners showed love for it

B

    All the useful energy at the surface of the earth comes from the activity of the sun. The sun heats and feeds mankind. Each year it provides men with two hundred million tons of grain and nearly ten million tons of wood.

   Coal, oil, natural gas, and all other fuels are stored-up energy from the sun. Some was collected by this season's plants as carbon compounds (碳化物).Some was stored by plants and trees ages ago.

   Even waterpower can be got from the sun. Water turned into vapor by the sun falls as rain. It courses (流淌) down the mountains and is turned into electric power.

   Light carries only the energy that comes from the sun's outer layers (层) , and much of this energy that is directed toward the earth never arrives. About ninetenths of it is absorbed by the atmosphere of the earth. In fact, the earth itself gets only one half-billionth of the sun's entire output of radiant (辐射) energy.

55.What's the subject discussed in the text?

   A. The activity of the sun.         B. The energy from the sun's activity.

   C. The resources of the earth.       D. The useful energy of the sun.

56.Radiant energy is stored as carborn compounds by  ____.

   A. plants                        B. rain

   C. coal, oil, and natural gas        D. things that can't be burned

57.The largest part of the radiant energy directed toward the earth is______.

   A. stored by the current season's plants      B. turned into fuel

   C. absorbed by the earth's atmosphere   D. used for electric power

58.The sun's energy indirectly provides the earth with ______.

   A. fuel and waterpower       B. light and sound

   C. rain and snow           D. earthquakes and hurricanes

C

   *Monday, September 25, 2000

   SYDNEY. Rich experienced Jan-Ove Waldner of Sweden stands in the way of a Chinese clean sweep of four table tennis gold medals after his defeating champion Liu Guoliang of China in straight sets (连胜) in the semi-finals of the Sydney 2000 Olympic Games.

   King Karl Gustay and Queen Silvia of Sweden were among the spectators as the fifth seed Waldner, the 1992 Barcelona gold medallists, surprisingly won over third seeded Liu 21 to 19; 21 to 16; 21 to 19.

   Waldner now faces China's Kong Linghui, a 1996 men's doubles gold medallist; in today's final Kong formed in the final after sweeping aside seventh seed Swede Joergen Persson 21 to 12; 13 to 21; 21 to 16; and 21 to 13.

   "If he (Kong) plays me like he did with Persson, he will be very hard to beat," said Waldner.

   *Wednesday, September 27, 2000

   Nanjing Four excuted

   The High people's Court of East China's Jiangsu Province yesterday rejected the appeals (上诉) of four Chinese murderers of a German family and retained (保留) the death punishment handed down (宣判) by a local court in the provincial capital, Nanjing. The four accused were excuted immediately after the issuance (公布) of the final ruling of the High-Court.

   * Wednesday, September 27,2000

   Beijing HK official

   Vice-premier Qian Qichen met here yesterday with Anson Chan Fang Onsang, chief secretary of administration for the Hong Kong Special Administrative Region (HKSAR) . Qian heard a report by Chan on the current situation in the  HKSAR.

59.Which is the best title for the first item of news?

   A. Waldner's Mind State before Men'Final.

   B. Waldner Won over Liu Guoliang.

   C.Waldner, only Bar to Chinese Sweep.

   D. Kong Linghui Will Defeat Waldner.

60.The four Chinese murderers of a German family ______.

   A. were sentenced to death

   B. were sentenced to life in prison

   C. were handed over to the High People's Court

   D. escaped death yesterday

61.Vice-premier Qian Qichen met the chief secretary of HKSAR

   A. in Hong Kong Tuesday September 26

   B. in Beijing Tuesday September 26

   C. in Hong Kong Wednesday September 27

   D. in Beijing Wednesday September 27

D

   Many people who are rich are also well known. Ted  Sweeney was an exception to this rule. His family moved to San Francisco from Los Angeles when he was one month old. That's where he grew up. At the age of seventeen he was hit by a train. Although he was not seriously hurt, the railway paid him $ 25,000 . Instead of going to college, he bought a small store. Six months later, the government bought his land to build a new highway. He sold it for $ 95,000 .

   With this money he moved to Detroit. He started a small company that made parts for car manufacturers. It was very successful and by the time he was 23, he was a millionaire. When he was 24, he got married. He and his wife had three daughters in the next five years. By the time he was 30, he had over ten million dollars.

   Then tragedy struck. He was involved in a traffic accident. He did not die but his wife and daughters did.Six months later, he sold everything he owned and put his money in stocks. Ted then moved to New York. He lived for the next forty years in a oneroom apartment.

   He spent most of his days wandering through the city, looking in garbage cans for food. He never worked; He rarely talked to anyone except himself. Most people were afraid of him. His clothes were always old and dirty.Shortly before he died, he moved back to Los Angeles. After spending two weeks there, he was put in jail because he had no money and no job. City workers tried to help him. They offered him work, but he would not work. Towards the end, he would not talk to anyone at all. When he died, he was a lonely man. But someone remembered his name. They knew he had lived in Detroit. After his stocks were sold and all the taxes paid, there was still over a hundred million dollars left.

62.Sweeney was born in ____.

   A. San Francisco                  B. Chicago        

   C. Los Angels            D. the place this passage didn't mention 

63.In the traffic accident ______ people died.

   A. two        B. three   C.four    D. five

64.Ted returned to Los Angeles ______.

   A. when he sold his land              B. after the traffic accident

   C. after he was free from prison         D. shortly before he died

65.Ted led a miserable life since _______.

   A. he sold his small store             B. he sold his land

   C. started a small company         D. he moved to New York

66.Ted Sweeney was a person ________.

   A. who was very famous

   B. who was very rich and famous

   C. who was very rich but not famous

   D. who was neither rich nor famous

E

   THREE women, praised as heroes for reporting bad practices at their places of work---the FBI (Federal Bureau of Investigation (联邦调查局)、 World Corn (世通公司) and Enron Corporation (安然公司) ---have been named "persons of the year" by Time Magazine at the end of 2002.

   They are Coleen Rowley from the FBI, Sherron Watkins, who worked at the now closed Enron Corporation, and Cynlhia Cooper, who exposed bad accounting (会计) methods at World Corn.

   The magazine chose the three women as they value truth and have shown courage?"for believing that truth is one thing that must be moved off the books and for stepping in to make sure that it wasn't. "

   Time managing editor Jim Kelly said the women stand for a critical struggle facing the US---how to restore trust in disgraced (丢脸的) places.

   "All three are sticking to what is right. All three of them are made of very strong character," Jim said.

   Rowley, 48, wrote a letter to FBI Director Robert Mueller in May. In it, she criticized the FBI for taking no notice of evidence before September 11 ,2001, which hinted of (暗示) an attack. She later told the Senate (参议院) that the FBI was buried in bureaucracy (官僚作风) and "careerism" (追求名利).

   Coopr, 38, was a World Corn internal auditor (内部审计员) . She warned the company's board (董事会) in June of US $ 3.8 billion in accounting mistakes. A month later, the telecommunications giant (通讯巨头) declared

the largest bankruptcy (破产) in US history.

   Watkins, 43, sent memos (备忘录) in August 2001 warning Enron chairman Kenneth Lay that improper accounting could cause the company to collapse. The company later declared bankruptcy, and Watkins resigned as a vice president in November.

Time's cover story on the three women compares them with Sept. 11 firefighters as heroes chosen by Circumstance (环境) .

   "They were people who did right just by doing their jobs right---which means completely, with eyes open and with bravery the rest of us always hope we have and may never know if we do. " the magazine writes.

   Last year. Time editors selected then-New York Mayor Rudolph Giuliani for leading the city's response to the Sept. 11 terrorist attacks.

   The 2002 picks are unusual because the vast majority of the magazine's persons of the Year have been well-known public figures---world leaders, war heroes, corporate chiefs.

   Rowley, Cooper and Watkins are nationally unknown before 2002. They said some colleagues now hate them for uncovering the mistakes of their leaders.

   "There is a price to be paid," Cooper said. "There, have been times that I could not stop crying." AGENCIES

67.What do the three women have in comnmon?

   A. Struggling to restore trust in their work places.

   B. Connected with accounting scandals (会计丑闻).

   C. Connected with the September 1 1 attacks.

   D. Connected with the firefighters.

68.Who declared the largest bankruptcy in US history?

   A. the Federal Bureau of Investigation.

   B. Enron Corporation.

   C. World Corn.

   D. Enron and World Corn.

69.When did Shcrron Watkins give up her work for Enron?

   A. In August last year when she sent out warnings.

   B. In October last year.

   C.Three months after she uncovered the improper accounting in her company.

   D. When her company declared bankruptcy.

70.Why is the choice of "Persons of the Year 2002" unusual?

   A. They are well-known public figures.

   B. They were not famous before.

   C. They were women.

   D. They are hated by their colleagues.

Ⅴ. 短文改错。10分

From Monday until Friday most people busy               71. ________

working or studying, and in the evenings and             72. ________

on weekends they are free to relax and enjoy himself.         73. _______

Some watch television or go to the movies.

others participate (参加) in the sports.                  74. ________

It depends on individual interests. There are many

different ways to spending our spare time.                  75. ________

Almost everyone has some kind of a hobby. It             76. ________

may be anything from collecting stamps to

make model airplanes. Some hobbies are very expensive,    77. ________

but others don't cost anything at all.

Some collections are worth a lot of money; others are         78. ________

valuable only to their own.                                79. ________

My youngest brother collects match boxes.

He has almost 600 of them but I doubt

that they are worth any money.                           80. _______

第二节 书面表达(满分30分)

    以Radio and Television为题目,写一篇短文。

注意:1.字数:100---130左右。 2. 参考词汇:

essential (必需的) part; inform sb. of sth; entertain (使快乐) sb. with sth; instruct (通知;向……提供事实情况); event; sailor; pilot; astronaut; instrument of communication; performer; ball games; close; by means of

中 学 英 语 能 力          

训练与测试系列    NSEFC- IIB-UNIT-T- 2005            

              高 中 二 年 级 下 学 期 期 中 考 试

               英  语 模 拟 试 题

第一节  单项填空(共15小题;每小题1分,满分15分)

     从A、B、C、D四个选项中,选出可以填入空白出的最佳答案。

1.-Could I use your towel?

           -_______.

   A. Yes, you could use it.                B. Why, of course.

   C. No, you couldn't.                  D. My towel?

2. He got to the station hurriedly only ______ the train had gone.

           A. to tell               B. telling           C. told          D. to be told

3. It's known to us that it takes years of ______ practice to gain ______ skills of an expert.

   A. the, the             B. X ; the      C. a; the       D. X ; X

4.-Miss King ______ music at a high school for five years and now is an actress.

   -No wonder I often hear her sing in the garden.

   A. had taught           B. taught       C. is teaching    D. has been teaching

5.It was an exciting moment for these football fans this year, ______ for the first time in

   years their team won the world cup.

   A. that               B. while       C. which       D. when

6.The man was severely injured in last weekend's terrorist attack taking place in Baghdad

   and died ______ afterwards.

   A. mostly              B. shortly          C. easily          D. hardly

7.It is in Steven Spielberg's first film, "Jaws, ____ a big white shark attacks swimmers ____

   are spending their holidays in a small village by the sea.

   A. where; who            B. which; that       C. that; that    D. where; that

8.As we all know, Huanghe is ______ river.

   A. China's second longest               B. the China's second longest

   C. the second China's longest             D. China's the second longest

9.No bread eaten by man is so sweet as ______ earned by his own labour.

   A. one                B. what       C. such        D. that

10.She won't leave the TV set, ______ her husband and children are waiting for her supper.

   A. as if               B. even if          C. once        D. so long as

11.-Is that 110? A thief is in my house…

   - Ok, help is ______.

   A. in the way            B. in its way        C. on the way    D. by the way

12. The President, together with his bodyguards, ______ to the nuclear station ______ there

   was an accident 20 minutes ago.

   A. have come, which  B. came; in which       C. has come; where   D. come; in which

13.-Can children swim in this pool?

   -Yes. However, at no time ___they do so alone.

   A. may               B. need       C. could         D. should

14.-Excuse me? 

           - ______

           -How can I get to the nearest post office?

   A. That's OK!            B. Pardon?      C. What's wrong       D. Yes?

15.Hard work and lack of sleep has ______ her beauty and youth in recent years.

   A. worn out            B. tried out         C. made out      D. sent out

第二节  完形填空(共20小题;每小题1.5分,满分30分)

   阅读下面短文,掌握其大意,然后从36---55各题所给的四个选项(A、B、C和D)中,选出最佳选项。

   Robert performs his 24-foot boat past willow (柳树) young trees that stick out of the waters of the Mississippi River.   16  dances off maples (枫树), their branches heavy with    17   spring leaves. But a   18   inspection discloses (显露) trouble behind the beautiful sight. Rubbish is  19   in a logjam (浮木阻塞) and hangs from the tree's branches.

   For the next three hours Robert and his team pull plastic bags, tanks, bottles and  20  bowling pins out of the water. Then they   21  for a picnic table caught in the trees. Welcome to the Mississippi River Beautification Project begun in 1997 as Robert's one man   22  to pick up rubbish   23  a 400-mile stretch (伸展) of the 2,340--ile river. That year, often working   24  , the 22-year-old cleaned 150 miles of shoreline.

   At first the project appeared   25   and useless. But Robert tried his best to find   26  , and with their backing, he soon had a new boat and five-man   27   --- the Boom Crane Crew. Last year alone the crew    28   from the water 44,055 gallon drums,  1,104 tires and enough plastic bags to   29   a football field. Much of the waste will be   30   .

   "Robert's operation is the only one actually in   31   on the river," says Mark of the Mississippi River Basin Alliance, a union of environmental groups. "It's the   32   I have ever seen in 20 years, and he is inspiring (激励) others to do the   33    ."

   "The river has given me a livelihood and brought me so much  34  ,"Robert says, "I want to do something in   35  ."

16. A. Sunlight             B. Air               C. Water          D. Moonlight

17. A. ripe                B. Brown           C. falling          D. fresh 

18. A. wider               B. quicker         C. closer          D. stricter

19. A. placed               B. piled              C. trapped          D. thrown

20. A. ever               B. even               C. nearly          D. merely

21. A. come               B. save               C. head           D. fight

22. A. work               B. discovery        C. place          D. effort

23. A. on                B. in                C. off           D. along

24. A. alone               B. strongly             C. quietly         D. fiercely

25. A. large               B. happy              C. impossible    D. possible

26. A. supporters             B. workers          C. engineers        D. pioneers

27. A. family               B. team               C. grade           D. class

28. A. came                B. pulled              C. pushed          D. appeared

29. A. clean               B. cover               C. build          D. spread

30. A. burned              B. buried            C. used          D.recycled

31. A. ending              B. progress             C. discussion       D. common

32. A. biggest              B. earliest            C. latest            D. luckiest

33. A. deed                B. favor             C. same             D. good

34. A. time                B. joy               C. pride             D. waste

35. A. return              B. turn               C. pay              D. trouble

第三部分:阅读理解(共20小题:每小题2分,满分40分)

    阅读下列短文,从每题所给的四个选项(A、B、C和D)中,选出最佳选项。

A

   ENGLAND coach Sven---Goran Eriksson faces an anxious wait to see if David Beckham will be available for England's Euro 2004 qualifier against Turkey on Octoberll. Beckham injured his right foot in Real Madrid's 0-2 loss to Valencia on Saturday in Madrid. He landed heavily on his foot, bruising the instep , after the studs of his left boot caught in the turf as he took a free-kick late in the game.

   CHINA'S hope of snatching its first-ever APT(Association of Tennis Professionals) title was dashed on Sunday as Zhu Benqiang and Zeng Shaoxuan were beaten 2-6, 4-6 by Australia's Wayne Arthurs and Paul Hanley in the doubles final of the Shanghai Heineken Open. But the event was a historic achievement for China anyway, as prior to it, no Chinese player had made it to the final of an APT tour event. "We need more time to get used to the way foreign players play," said 22-year-old Zeng. "We are still young, which means we have many opportunities to improve ourselves," he said.

36. Valencia is the name of ______.

   A. a man              B. a woman      C. a football team         D. a place

37. Who are tennis players?

   A. Zhu Benqiang and Arthurs               B. Eriksson and Zeng Shaoxuan

   C. Zhu Benqiang and Beckham              D. Beckham and Zeng Shaoxuan

38. The underlined word "instep" in paragraph one means ______.

   A. part of a shoe          B. part of a foot       C. leg        D. back

39. Which of the following statement is wrong?

   A. Australian players won the game.

   B. China's players joined in the APT games for the first time.

   C. It is likely that David Beckham will not attend England's Euro in 2004.

   D. The two Chinese tennis players haven't lost heart.

B

   One of the biggest problems in developing countries is hunger. An organization called Heifer International is working to improve this situation. The organization sends needed farm animals to families and communities round the world.

   An American farmer, Dan West, developed the idea for Heifer International in the 1930s. Mister West was working in Spain where he discovered a need for cows. Many families were starving because of a civil war in that country. So Mister West asked his friends in the United States to send some cows. The first Heifer animals were sent in 1994.

   Since that time, more than four million people in one hundred and fifteen countries have had better lives because of Heifer animals. The organization provides families a chance to feed themselves and become self -supporting. It provides more than twenty kinds of animals, such as sheep, goats, pigs and cows. Last year. Heifer International helped more than thirty thousand families in forty-six countries.

   To receive a Heifer animal, groups must first explain their needs and goals. They must also make a plan which will allow them to become self-supporting. Local experts usually provide training. The organization says that animals must have food, water, shelter, health care, and the ability to reproduce. Without them, the animals will not remain healthy and productive.

   Heifer International also believes that groups must pass on some of their success to others in need. This belief guarantees that each person who takes part in the program also becomes a giver. Every family that receives a Heifer animal must agree to give that animal's first female baby to other people in need. Families must also agree to pass on the skills and training they received from Heifer International. This concept of "passing on the gift" helps communities become self-supporting.

30. This passage mainly ______.

   A. explains what Heifer International is

   B. states huger is one of the biggest problems in developing countries

   C. criticizes Heifer International for its requirements

   D. tells a story about Dan West, the founder of Heifer International

31. What are those families expected to do with the Heifer animals they receive?

   A. To eat them when they are short of food.

    B. To raise them for the organization.

   C. To give them to the people in need.

   D. To become self-supporting.

32. What should a family or an organization do to get help from Heifer International?

   A. Make Heifer International believe that they're going hungry.

   B. Agree to pass on some of the animals to other families.

   C. Prove to Heifer International their need for help and that they'll do as required.

   D. Promise that they'll help other families in need.

C

   Nathan's second-hand bike was good enough to get him anywhere he wanted to go. Of course, it couldn't match Tim's in a normal speed race, but all the fancy gears in the world wouldn't get you up Black Mountain. The slope was so steep (陡峭的) in places that you had to get off your bike and push.

    On this particular afternoon, the odds (possibilities) were all on Nathan's side. For a start, he was stronger. Tim had always been thin, but recently he'd started to grow so fast that his arms and legs were like pieces of spaghetti. As well as being considerably fitter than his friend, Nathan knew the shortcuts (the quickest way) up Black Mountain.

   So when Nathan accepted the challenge (挑战), he was sure he'd win. He nearly always did, in competitions with Tim. That was what was strange. Tim was so competitive, and yet he wasn't any good at physical things.

   By the time the two boys were three quarters of the way to the top. Nathan was clearly ahead. While the road zig-zagged back and forth diagonally up the mountainside, in certain places there were also vertical tracks, where water rushing down the slope in winter had bitten erosion courses into the soil. These had in turn been so overgrown by bushes and long grass in the spring that they resembled tunnels through the bush. The trick was to know where you could cut off a whole bend in the road by leaving the road and pushing your bike up one of these semi梒oncealed pathways. If you chose the wrong one you'd struggle up for ten minutes and then suddenly reach a dead end. Then there was no choice but to go back.

33. What did Tim challenge Nathan to do?

   A. test who had the faster bike.           B. decide who was the most competitive.

   C. find the secret tracks on the mountain.   D. race to the top of Black Mountain.

34. Which of the following advantages did Nathan have over Tim?

   A. Nathan had a better bike and knew the shortcuts up the mountain.

   B. Nathan was fitter and knew the quickest ways up the mountain.

   C. Nathan was stronger and more competitive than Tim.

   D. Nathan was stronger and had a better bike.

35. The underlined part "the odds were all on Nathan's side" means that ______.

   A. Tim would not try very hard in the race.

   B. Tim had not yet grown as big as Nathan.

   C. Nathan had a greater chance of winning the challenge.

   D. Nathan had more friends who thought he would win.

36. Which of the following conclusions can be drawn from the third paragraph?

   A. This would be the most significant challenge that the boys have had.

   B. Nathan would be most likely to win this challenge in spite of Tim's competitiveness.

   C. This challenge would be too difficult for both boys.

   D. Tim thought this challenge was more important than his friendship with Nathan.

D

   Tricia Mack is a 27-year-old aerospace engineer for the National Aeronautics and Space Administration and is travelling to Russia to work on an international space station. It was a quick journey from Rochester Hills to NASA, but instead of being an astronaut, Ms Mack is training them. She knew she wanted to be involved with outer-space, even as far back as her days at the West Middle School. And although Ms Mack knew where she wanted to go, she also knew what it would take. She offers simple advice for kids who might see themselves as future NASA employees: Study your math and science.

   Ms Mack teaches everything from putting on the suits to explaining how to read the warming features. Ms Mack shows astronauts how to understand information on the suit itself, as well as warning they might hear in their helmets (头盔). Later, she'll teach astronauts how to do certain things in their suits.

   "We have a huge pool that the astronauts are placed in. They wear weights so it feels like neutral gravity(无引力) where they're not sinking and they're not floating to the surface," she said. "Underwater, astronauts have a stand-in for the conditions in outer-space. "

   Ms Mack will continue this role as she begins work on an international space station project. More than 40 flights through 2004 will be taken to the space station. Those include trips by the American spaceships and the Russian Soyuz rocket, which hold three persons.

37. When Ms Mack studied at West Middle School, she ______.

   A. was eager to do something for the astronauts

   B. knew she would be a very good teacher

   C. wasn't good at math or science

   D. was determined to work on the study of outer space

38. As an engineer, Ms Mack ______.

   A. makes suits for astronauts

   B. designs equipment for Space Centre

   C. prepares the astronauts for the conditions in outer space

   D. trains astronauts how to operate the shuttles or rockets

39. In the near future, ______.

   A. 2004 flights will be taken to the space station

   B. more than 40 flights to the space station will hold people

   C. American astronauts can stay in space longer than Russian astronauts

   D. Russians and Americans will continue their work on the space station project

50. In this text, the underlined word "stand-in" means ______.

   A. a person who stands by and looks on while astronauts are working

   B. something that takes the place of conditions in outer space

   C. someone who waits for astronauts in a huge pool

   D. research in outer space instead of an astronaut

51. Which of the following shows the right order?

   a. she went to Russia to work on an international space station.

   b. She wanted very much to become an astronaut.

   c. Ms Mack taught astronauts how to use their space suits.

    d. Mack studied in a college and graduated form it yeas later.

   e. Ms Mack continued her teaching and did a good job.

   f. Tricia Mack studied at Rochester Adams High School.

   g. She found a job at NASA's Lyndon B. Johnson Space Centre

    A.d,g,b,a,f,e,c                          B.b,f,d,a,g,c,e

   C. f, d, b, g, a, c, e                      D. f, b, d, g, c, a, e

E

   Education is not only the teaching and learning of knowledge, but also transmitting of moral standards (道德标准) and the developing of a person's character.

   Liberal education is now spreading all over the world. Much has been done to try to find better teaching methods. Students are encouraged to take part in lessons which are often held in the form of group discussions. They are encouraged to have their own thoughts and their own ideas. They are properly guided in the application of the knowledge learnt. Severe punishment no longer exists even if students misbehave (行为不端) themselves. Teachers and students are just like friends, even though basic manners are sometimes paid little attention to. They are guided to become independent? responsible persons. The ideas are of course a perfect one if it can really be carried out to perfection. However, Utopia (乌托邦) is a place which only exists in the imagination and will never come into reality.

   Considering the ever-increasing rate of juvenile delinquency (青少年犯罪) and the untamed (粗野的) appearance of the youth on all occasions, we must admit that immature teenagers should at least be guided properly. Just close your eyes and think how many students there are in every hundred that will meet the duties of a student if they are not pushed. What is the use of mere advice to one who might, because of his nature, tend to be lazy and evil? Advice will become useful and be accepted only if it is supported by a reasonable degree of control and punishment.

   It is hard for one to judge the success and failure of an education system. But from the obvious bad behavior of the juveniles who are actually the victim of liberal education, we can see that something must be done to save the spoilt (宠坏的) children.

52. The new relationship established between teachers and students ______.

   A. helps students to be independent and responsible

   B. makes students behave better

   C. cause students to pay less attention to their manners

   D. encouraged students to be closer and more polite to their teachers

53. One of the disadvantages of liberal education is that ______.

   A. more and more teenagers are ill-mannered

   B. the youth spend too much on their clothing

   C. the youth are more dependent

   D. the youth don't want to be responsible for the society

54. A reasonable degree of punishment is needed in school because ___

   A. too much punishment will hurt the youth

   B. teachers are allowed to punish students

   C. it will help the youth accept useful suggestion

   D. the teenagers are to be punished

55. In this passage, the write thinks ______.

   A. liberal education is popular and reasonable

   B. much should be done to perfect liberal education

   C. liberal education is not practical and should be done away with

   D. schools and teachers should be responsible for the spoilt children

第四部分:写(共两节,满分35分)

第一节  短文改错(共10小题,每小题1分;满分10分)

此题要求改正所给短文中的错误。对标有题号的每一行作出判断;如无错误,在该行右边横线上划一个勾(√);如有错误(每行只有一个错误),则按下列情况改正:

该行多一个词:把多余的词用斜线(\)划掉,在该行右边的横线上写出该词,并也用斜线划掉。

该行缺一个词:在缺词处加一个漏字符号(∧),在该行右边横线上写出该加的词。

该行错一个词:在错的词下划一横线,在该行右边横线上写出改正后的词。

注意:原行没有错的不要改。

   In the past few years, great changes have been taken place rapidly      56. _______

in Beijing, which make people's lives more convenient and colorful.       57. _______

Beijing is a nice city of a history of over 3,000 years; it has a population   58. _______

of 13 million , with many places of interests . These years, many buildings  59. _______

have set up, such as museums, parks, business markets, highways         60. _______

and overpasses; and also a large amount of buses have been put into       61. _______

use. Recently, the environment in Beijing has been improving.            62. ________

Plenty of flowers, grass and trees have been planting. Lakes and rivers     63.________

are also being turned clean. However, a fine new type of fuel has          64. ________

come into use, which can stop the air of the city polluted.                65. ________

第二节 书面表达(满分25分)

    下面是你班班会上以“中学生是否能佩带手机”为题进行讨论的情况。请你写一封信给21st Century报的编辑说明该情况,并附带表明你自己的观点。

注意:1、要包含所有的要点,但不要逐句翻译;  

       2、字数约120字;

       3、信的开头和结尾已写好

       4、生词:辐射radiation    奢侈的luxurious   亲戚relative

Dear Sir,

     I'm writing to tell you ...

       Best wishes.

                      Yours, Kate

中 学 英 语 能 力          

训练与测试系列    NSEFC- IIB-UNIT-T- 2005            

              高 中 二 年 级 下 学 期 期 末 考 试

英  语  模  拟  试  题

第一节  单项填空(共15小题;每小题1分,满分15分)

   从A、B、C、D四个选项中,选出可以填入空白出的最佳答案。

1.In the market, vegetables are sold by ________ kilogram, I mean, by ________ weight.

   A.the, /     B./, /       C.the, the     D./, the

2.The clouds had come down ________ suddenly we had no warning.

   A.quite     B.rather      C.so        D.too

3.I ________ while reading the English textbook.Luckily, my roommate woke me up in time.

   A.had fallen asleep        B.have fallen asleep C.fell asleep D.fall asleep

4.Look! The boy  is holding ________ jewelry in his hands.

   A.a piece of   B.a        C.a bar of     D.a little bit of

5.If I had not been ill, I ________ in bed for weeks.

   A.couldn't lie B.shouldn't have lain         C.would lay   D.shouldn't have laid

6.The work is ________ smoothly. I'm sure it will be finished in a month or so.

   A.progressing  B.improving    C.moving      D.keeping

7.—Did you look up the time of trains to Shanghai?

   —Yes, the early train is ________ to leave at 5:00 am.

   A.likely    B.about      C.possible     D.due

8.The girl ________ forward to buying a new gold watch.

   A.referred to looks        B.referred to look C.referred to looking D.referring to looking

9.—Why did he say so?

   —Sorry, I don't understand ________ he has said means.

   A.all what   B.what all     C.all that     D.that all

10.There are many beautiful islands ________ the west coast of the country.

   A.along     B.off       C.on        D.from

11.—How are you getting on with your business?

   —I'm glad to say it is ________.

   A.breaking up  B.taking up    C.turning up    D.picking up

12.Just like a voyage at sea, our life journey, ________ days are limited, is full of difficulties.

   A.whose     B.which      C.as        D.what

13.—I promise that his daughter ________ get a nice gift on her birthday.

   —Will it be a big surprise to her?

   A.should    B.must       C.would      D.shall

14.Mr. Smith had been to Beijing for several times, he should have been in China now, ________ he?

   A.hadn't     B.haven't     C.didn't      D.shouldn't

15.—It's five years since I worked here.

   —________?

   A.Have you worked here happily   B.How long will you work here

   C.Where do you work now      D.Do you want to work here longer

 第二节 完形填空(共20小题,每小题1.5分,满分30分)

    阅读下面短文,掌握其大意,然后从36~55各题所给的A、B、C、D四个选项中,选出最佳选项,并在答题卡上将该项涂黑。

   Anna lived on the side of a valley. One winter, there was a very big flood, and  a lot of houses   16  Anna's were washed away. Anna's house was high enough to escape the flood, so when the  water had disappeared and the other houses were   17   there with no roof and no walls and all covered with   18  , her house was   19   quite all right. Her house was quite small, her husband was dead,  and she had four children,   20   Anna took in one of the families that had lost    21   in the flood and she   22   her home with them until it was    23   for them to rebuild their house. Ann's friends were   24   when they saw Anna do this. They could not understand why Anna wanted to give   25   so much more work and trouble when she already had quite a few children to   26  ."Well," Anna   27   her friends, "at the end of the First World War, a woman in the town where I   28   lived found herself very poor, because her husband   29   in the war and she had a lot of children,   30   I have now. 

   The day before Christmas, this woman said to her children, ' We won't be able to have much for Christmas this year, so I'm going to   31   only one present for all of us. Now I'll go and get it. She came back   32   a girl who was even poorer than they, and who had no parents. '   33  ' she said to her children. The children were    34   to get such a present. They welcomed the little girl, and she grew up as their sister,    35   was that Christmas present."

16.A.down below             B.just around   C.next to        D.above

17.A.rising               B.appearing     C.falling        D.standing

18.A.water                B.trees       C.dust         D.mud

19.A.just                B.already      C.yet          D.still

20.A.so                 B.but        C.for          D.since

21.A.nothing               B.everything    C.anything       D.something

22.A.made                B.found       C.shared        D.built

23.A.possible              B.necessary     C.important      D.valuable

24.A.worried               B.disappointed    C.puzzled       D.impressed

25.A.them       B.herself     C.them all     D.her

26.A.support     B.supply      C.grow       D.feed

27.A.explained to   B.asked for    C.talked with   D.spoke as

28.A.actually     B.then       C.before      D.later

29.A.had killed    B.killed      C.had been killed D.might be killed

30.A.for       B.as        C.like       D.that

31.A.get       B.send       C.buy        D.make

32.A.for       B.from       C.like       D.with

33.A.Show the present.                   B.Let's go and see the present.

   C.Can it be the present?      D.Here's our present.

34.A.worried and sad B.excited and happy    C.anxious       D.sorry

35.A.it        B.she       C.such       D.I

第三部分:阅读理解(共20小题;每小题2分,满分40分)

A

The Man Who Never Puts a Foot Wrong Some people do not like anything to be out of place (不循规蹈矩); they are never late for work; they return their books to the library on time; they remember people's birthdays; and they pay their bills as soon as they arrive. Mr. Dodds is such a person.  Mr. Dodds works in a bank, and lives of his own. The only family he has is in the next town: his sister lives there with her husband and her son, Mark. Mr. Dodds does not see his sister or her family from one year to the next, but he sends them Christmascards, and he has not forgotten one of Mark's seventeenth birthdays.Last week Mr. Dodds had quite a surprise. He drove home from the bank at the usual time, driving neither too slowly nor too fast; he parked his car where he always parked it, out of the way of other cars, and he went inside to make his evening meal. Straight away,there was a knock at the door. Mr. Dodds opened the door,to find a policeman standing on the door-step.

"What have I done wrong?" Mr. Dodds asked himself. "Have I driven on the wrong side of the road? Has there been some trouble at the bank? Have I forgotten to pay an important  bill?"

 "Hello, uncle," said the policeman. "My name's Mark."

36. "The Man Who Never Puts a Foot Wrong" means a person who ________.  

   A. never puts a wrong foot  

   B. always walks in a certain way

   C. likes doing things regularly and correctly

   D. does everything carefully

37. His sister ________.

   A. is the only member of the family that he knows

   B. lives with Mr. Dodds, with her husband and son

   C. lives in the next town with her husband and son

   D. has only one family, a son and Mark, her husband

38. "He has not forgotten one of Mark's seventeen birthdays" means ________.

   A. he has not forgotten Mark's seventeenth birthday

   B. he sent Mark something on his seventeenth birthday

   C. he always sends a Christmas card on Mark's birthday

   D. he always sends Mark something on his birthday

39 There was a knock at the door when Mr. Dodds was ________.

   A. making his meal

   B. parking his car

   C. ready to make his evening meal

   D. just about to shut the front door

40. The policeman was there ________.

   A. to meet Mr. Dodds, his uncle.  

   B. to ask Mr. Dodds to go and see his uncle.

   C. to ask Mr. Dodds to mark his name on his door.

   D. to see Mr. Dodds about some trouble at the bank.

B

HOLIDAY HOMES IN MALLORCA

   Holiday apartments in Mallorcan sailing and fishing port quiet even in summer season. Beautifully situated apartments with views of sea and mountains, yet near to shops and restaurants. Cars and bicycles for hire. Sailing and sports clubs nearby.         

----------ITALY IN COMFORT----------

    Luxury coach (=carriage) tours of Italy, out of normal holiday season. 21 leisurely (休闲) days to visit five Italian cities starting from London 1st May, 1st September. The tours are guided by Professor Martin Davis, formerly Head of Italian Studies, London University. See the arts and culture of historic Italy. 

----------KIBBUTZ HOLIDAYS IN ISRAEL----------

   Working holidays on kibbutz(co-operative farm) in Israel. All nationalities welcome for one to three months, if prepared to work morning with kibbutz members. Accommodation(住), food and trips to historic sights all provided free--you pay only for the special low-cost return flight.

----------TWO WEEKS ON A CARIBBEAN ISLAND----------

   Two-week holidays in the luxurious Hotel Splendid, on a lovely  beach with golden sands and deep-blue sea. Tennis, golf, sailing  and all water sports. Trips and tours around the islands arranged. Near to town of Castries with lively evening entertainment(娱乐) --dancing to local bands.                      

1st November—31st March=£720 per person              

1st April—30th October=£850 per person

   ·Jack and his wife Mary, who have recently retired, want to see places of cultural and historic interest abroad, but Mary hates flying.  

   ·Peter and Maria, university students, want to travel as far as possible on very little money, and would like to get to know a country by working there for three months with other young people.  

   ·Michael, a young computer programmer, has been working hard and needs a holiday to relax--in winter. He would like to go some- where warm and sunny, where he can swim in the sea, and he enjoys sports and dancing.  

   ·Harry and Kate, both teachers, and their two teenage sons, have to take their holiday during the school summer holidays. There must be plenty for the boys to do, although Harry and Kate just want beautiful scenery, good food and wine--and peace.  

41. Michael would mostly enjoy________, where he can go in winter.

   A. spending two weeks in the splendid hotel on the Caribbean island

   B. visiting five Italian cities starting from London   

   C. seeing the splendid arts and culture of historic city   

   D. driving cars and bicycles along the seaside  

42. The most suitable holiday for Peter and Maria would be________, as it is cheap and sounds very

   interesting.

   A. the summer holiday in Mallorca

   B. the tours guided by a professor 

   C. the kibbutz holiday in Israel 

   D. the tours arranged near to town of Castries  

43. The best holiday for Jack and Mary would be _________.

   A. the leisurely 21-day coach tour of Italy   

   B. the working holidays for 1 - 3 months on a farm   

   C. the splendid 14-day trip around the islands   

   D. the interesting 2-week stay in a luxurious hotel  

44. Harry and Kate and their sons would like ______, which is  quiet even in the busy summer season.

   A. a holiday working on a kibbutz in Israel

   B. a holiday visiting ancient cities by coach in Italy

   C. a holiday hotel on a lonely beach on the Caribbean island 

   D. a holiday apartment in the fishing port in Mallorca

C

   Hong Kong, major commercial center for Asia, and  with a population which has grown at an

alarming rate to over 5 million, is a city highly dependent on mass (大量的, 大规模的) transit of all sorts, both local and long distance. An ordinary  Hong Kong worker or businessman, going about his daily activities, simply must use transportation at one time or another.  

   Because Hong Kong is in two parts, Kowloon, on the mainland side, and Hong Kong, the island, with Hong Kong's harbor in between Hong Kong's mass transit systems, in addition to going over land they must also cross water.  

   Going from home to work, or going shopping from one side of the harbor to the other, the Hong Kong resident has three choices. One way is to take a bus which will cross the harbor through an underwatertraffic tunnel moving slowly through bumper- to-bumper (一辆接一辆) traffic. Another way is by ferry boat , a pleasant ride which crosses the harbor in from seven to fifteen minutes.  

   But by far the fastest way of crossing the harbor is the newly built underground electric railway, the Hong Kong Metro(地铁). If one gets on the train in the Central District,the commercial area of Hong Kong on the island side, he can speed across the harbor in an astonishing three minutes. On the other side of the harbor the railway continues, snaking back and forth through the outlying districts of Kowloon, allowing one to get off a short distance from his destination.  

   The story of the Metro is an encouraging one for supporters  of mass transit. Although building the system was certainly a challenging task, the Japanese firm hired to construct it did so in record time. Construction got underway in 1979 and it was completed in 1980.  

   For the average commuter (一般持月票往返两地的乘客) the system has only onedisadvantage; it is more expensive than by bus or ferry. One can ride the bus across the harbor for half as muchor he can ride the ferry across for less than one-fifth as much.

45.Hong Kong public transportation extends ________.

   A.over hills and valleys          B.across land and water  

   C.through mountains            D.throughout the Kowloon area

46.Crossing the harbor by train is ________. 

   A.by far the most economical methods        B.the most pleasant method  

   C.the least pleasant method            D.the fastest method

47.The business area on the island side of Hong Kong is referred to as ________. 

   A.Kowloon              B.the Central District  

   C.the Hong Kong Metro           D.downtown Hong Kong

48.The underground railway ________.  

   A.winds through Kowloon           B.ends when it reaches Kowloon  

   C.snakes across the harbor         D.circles Kowloon

D

   On May 21,1999, some American scientists were working at the computers to look for information they needed. Suddenly they saw a lot of very bright red spots crossing the computers screens. At the same time the computers were working much slower. To find out what was happening, they stopped their work to check some parts of the computers. To their horror, they found out that most of their stored information was got rid of by computer viruses! Obviously all these computers had been infected by computer viruses.

   It is said that the computer viruses were made by two or three Philippine young men fond of playing tricks. They all had excellent education. They created the viruses just to show their intelligence. The kind of computer virus is named"I Love You"Virus. This virus can hide in computers for a long time .When the time comes they will attack the computers by lowering the important functions, damaging their normal programs or even getting rid of a great deal of information which operators of the computers often use or store. What's worse, it still can reproduce itself in great quantities within a short time.

   We have come to know that "I Love You" Virus often attacks computers on Mondays and that it is spreading to many computers in the world. Among the countries that suffered computer viruses recent years are Britain, Australia, Switzerland and the U.S. Those who made the computer viruses have been found out slowly and carefully, But till now ,how to get rid of the terrible viruses remains a problem.

49.Two or three Philippine young men created the computer viruses to ________.

   A.damage the computers

   B.test their ability quickly

   C.tell the world that they were intelligent

   D.play a trick on operators of the computers

50.The most serious damage caused by the viruses is that ________.

   A.the computer's functions are lowered

   B.the normal programs are damaged

   C.most the information stored in the computers is gone

   D.the computers infected by the viruses can no longer be used

51.According to the passage, which of the following is true?

   A.Scientists are trying to find a way to get rid of the viruses.

   B.the viruses will come to the new computer after staying in the old one for some time.

   C.Last year four countries found their computers were infected by viruses.

   D.The "I Love You"Virus is a great ham to human health.

E

   To Whom It May Concern:

   My husband and I got married in 1965 and for the first ten years of our marriage I was very happy to stay home and raise our three children. Then four years ago, our youngest child went to school and I thought I might go back to work.

   My husband was very supportive and helped me to make my decision. He emphasized all of the things I can do around the house, and said he thought I could be a great success in business.

   After several weeks of job-hunting I found my present job, which is working for a small public relations firm. At first, my husband was very proud of me and would tell his friends , "My clever little wife can run that company she's working for."

   But as his joking remark approached reality, my husband stopped talking to me about my job. I have received several promotions and pay increases ,and I am now making more money than he is. I can buy my own clothes and a new car. Because of our combined incomes, my husband and I can do many things that we had always dreamed of doing ,but we don't do these things because he is very unhappy.

   We fight about little things and my husband is very critical of me in front of our friends. For the first time in our marriage, I think there is a possibility that our marriage may come to an end.

   I love my husband very much, and I don't want him to feel inferior, but I also love my job. I think I can be a good wife and a working woman, but I don't know how .Can you give me some advice? Will I have to choose one or the other or can I keep both my husband and my new career?

    Please help."Distressed"

42.The letter was most probable written ________.

   A.in 1975              B.around 1980

   C.four years ago              D.in 1965

43.Her husband ________ when she first found her present job.

   A.was very critical of her         B.felt disappointed

   C.was proud of her         D.was happy but critical

44.What does the underlined word "promotion" mean?

   A.scolding      B.criticism             C.prize         D.advancement

45.As her income increased, ________.

   A.she found a gap emerged between her and her husband

   B.she bought more clothes and a house

   C.she did the many things she and her husband dreamed of

   D.she felt very proud of herself

第四部分:书面表达

第一节 短文改错(共10小题;每小题1分,满分10分)

When I walked into the classroom, the teacher was handing

in the tests. I was feeling very nervous. I had not studied     46._______

at all at the weekend as I had thought it would be easy test.    47._______

I went through the test for many times but I could only answer    48._______

three out from the twenty questions. I did not want to fail         49._______

the exam. Then, I put my book under my desk, opening it        50._______

and started looking for the answer. The teacher wasn't         51._______

looking at me, but I copied something. Suddenly, I felt a hand    52._______

on my shoulder! The teacher caught me cheating. I don't        53._______

know what to say. Luckily, the teacher did not punish for       54._______

cheating but instead gave me a second chance.                55._______

第二节书面表达(满分25分)

    天空中的人造地球卫星越来越多,请根据下列表格,简要介绍一下人造地球卫星。

注意:1.词数120左右。

       2.参考词汇:军事的 military

 

中 学 英 语 能 力          

训练与测试系列    NSEFC- IIB-UNIT-T- 2005           

            高中二年级英语试题参考答案

Unit 11  Scientific achievement

1---15  BCABC ACAAA BCCBA

16--30 BDCBC  DCBAC  CBAAB

31--50  BADCD  ACBAD  CCBDA  BCBDA

51--70  BDA  CDBB  BCDAC  BACC  CABA

71.get 改为 gets            72. week 前加 the / every / each   73. then 改为 and

74. much 改为 more 75. 去掉 had     76. with 改为 to 77. too 改为 again

78. 去掉 of    79. and 改为 that   80.√

书面表达参考答案:

     Have you planted a flower? If you haven't, now let me show you how to do it.

    Generally speaking, planting flowers needs five steps. Firstly, you should cover the bottom of the pot with some small stones. Secondly, you put about two inches of soil on top of the stones. Thirdly, you first make a hole in the centre of the soil, and then put the flower you want to plant into the soil. Fourthly, you add more soil until it almost reaches top of the pot, ad press the soil lightly with your hands. The last step is very simple-water the flower. But you don't forget to move it under the shade.

           

Unit 12 Fact and fantasy

1---15  DDBDB  DCADA  BACDD

16--30 CBADD  ABBCC  CAADB

31--50  ABDBD  ADBBD  CACBB  BDCDD

51--70  BCCD  ABDC ACCAB  ADBB  CDC

71. a little       72. Which-As      73. of a -with a      74. friendly

75. and-or      76. been 去掉       77. look-see        78. can also

79. √          80. see-seeing

书面表达:

  Einstein was born in Ulm, Germany in 1879. When he was a boy, he liked to ask questions. By the time he was 14, he had taught himself advanced mathematics. Just at that time he made up his mind to be a physicist and devote himself to science.

  His family, however, were poor, but his parents did manage first to send him to a technical school, then to the Federal Institute of Technology in Switzerland. He received his doctor's degree at the University of Zurich in 1905. It was then that he first began to do the research work which led to his famous Theory of Relativity.

    Einstein was one of the greatest scientists in the world.

Unit 13   The water planet

1---15 BBDCC CBADB  CCDAA

16--30  AACDA ABADC BACBA 

31--50  CACAA  DDBAA  BACBA  BDCCD

51--70  BBA   ADCD  DABD  DBACA   DCCC

71. scientist→scientists      72.except→besides       73. √

74. before→ago           75. 去掉 the             76. excited前加has

77. and→because          78. finding→found        79. body→bodies

80. and-but

书面表达

VI. One possible version:

                      I Middle School

                     Jingzhou,Hubei, China

                     March 20, 2005

Dear Mr. Smith,

    I learn from the newspaper that your company is a well-managed computer company and that several clerks are wanted in your company.

    My name is Li Ping. I'm male and healthy. I've graduated from Computer Department of Nanjing University. I was born in Jingzhou, Hubei Province, in July 5,1984.

    I'm good at computer and English. I'm able to read science books in English. I do well in English writing. I have some knowledge of Japanese. In my spare time, I often play football.

    If possible, please ring me.  My telephone number is 0515-. If I am accepted, I'm sure I'll be fit for the position.

    Looking forward to your early reply.

                                                Yours,                                               Li Ping

Unit 14  Freedom Fighters

1---15 CACBB  CAADD  DCDBA

16--30  DBCDC CBDAA AACBC

31--50  CABDB DCBDA  CBDCD ABBCB

51--70 BDBB  ADD  BABDA  CDBA  BBBC

71. 去掉time    72. a→an         73. for→of          74. drove→drive    75. his→their

76. √          77. have→had    78. eager前加were    79. questions       80. 去掉in

书面表达:

                          On the way to the cinema.

   Today is Sunday.After having breakfast I decided to see a film.The cinema was a bit far away,so I had to take a bus there.

   When I was sitting on the bus,an old woman got on it.There were no seats free,so I stood up at once and said to her,"Come over and take my seat,please."The old woman walked over and sat down.She was excited,saying "Thank you very much,dear little girl."

   Soon the bus stopped in front of the cinema.I got off the bus and said goodbye to the old woman.The old woman waved to me with a smile

 Unit 15  Destinations

1---15  CDCAC  ACBAB  ABCBB

16--30  DDBCA  AD DAA  A CCCB

31--50 D C A A B  D D B C C  B A A B D  B C B B C  

51--70 DBCA   DCBDA  ABCD  CAC  DCAB    

71 去掉 was  72. 去掉 in 73. house→houses  74. With→As 或 went→going 75.√    76. struck→strike  77. However→So  78. terribly→terrible  79.they→we  80. instead ∧ of 

书面表达

Saturday, February 22nd

   This afternoon we had a football match with Class 2. When I was running toward the bell, I fell over. I was hurt so badly that I could hardly stand up. Some of my classmates came up and helped me onto a bike. They took me to the nearest hospital. The doctor in the hospital examined me carefully and said that there was nothing serious. After I was given some medicine, I was allowed to go home. I was so sorry that I couldn't play the match. But I was happy that I had so many friends to help me when I was in trouble.

内容要点:

1.与2班进行足球比赛2.摔倒3.受伤4.送医院5.检查与治疗6.出院7.感想

Unit 16 The United States of America

1---15  CCCDB  BDABA  CAACD  

16--30 BCDBB ACBCD BCCCC

31--50  CCAAB  BDDDA  CAACD  DBCBB

 1--70  BCDDA  DBCDC  CCDAC  DBABA         

71.large 前加 the  72.in → among    73.its → their      74.number → numbering

75.practical →practiced 76.after → before   77.去第二个 the     78.√ 

79.grew → grow  80.at → in 

书面表达

May 3 rd, Friday              Fine

This afternoon, on my way to the cinema, I saw a case fall off a man's bike. I shouted out to the man to stop, but he didn't hear me and rode away.

I was wondering what to do when I had an idea. I stopped a taxi and got into it with the case. Soon we caught up with the man. Getting the case back, he was so thankful that he offered me some money, but I refused it politely.

Then the driver took me to the cinema. When I paid him, the driver pushed my hand away and said with a smile,“Neither will I accept your money.”

Unit 17   Disability

1---15  AABBB  DBCDA  BACBB

16--30  DACCA  BABBA  ABBCC

31--50  DCCAB  ABDBD  ACCAB  BDDAA

51--70  BDAC   CCBB  BDBA  ADAB  BDAB

71. has-had    72. √    73. said-saying     74. 去掉 a    75. like to

76. answers     77. either-too     78. 去掉 will      79. for-to

80. as-like

书面表达

  One possible version:

   Attention please! I've something to tell you.

   Professor Smith-an American disabled person will come to our school to give us a lecture from 2 to 4 o'clock on the afternoon of October 25 in Room 201, the No. 2 teaching building. His lecture is about how he smoothed away all sorts of difficulties. When he was nine years old, he lost his left leg and right arm in an accident. He struggled in his life and managed to look after himself. He achieved great success in his work, too.

  After the lecture we will have a discussion in groups on how to encourage disabled people, how to help them and how to learn from Professor Smith.

   That's all. Thank you.

Unit 18  Inventions

1---15  BCBAC  ABAAA  CBCCA

16--30  DBCCA  CABCB   BABCC

31--50  DADDA  DBCAB  DDCAB  BABAC

51--70  BCDC  BBDB  DABA  CAAD  CBAB

71. 去掉 is    72.√    73. them-it     74. as-than    75. show-shown

76. what was      77. 去掉in      78. different     79. leaving     80. of people

书面表达:

                   Guangming High School

                   July 6th ,2005

Dear editor,

   Our school lies at the foot of a mountain with a small river passing by. It used to look like a beautiful garden. There used to be green trees and all kinds of flowers in our school all the year round. It used to be a quiet place and it was good place for a school.

   Things have changed since a chemical works was built near our school a year ago. Every day they produce much waste water and some waste and harmful gases. The terrible pollution has done great harm to us. And the great noise from the works has greatly affected our teaching and studying activities. It makes our school no longer a quiet and good place for study. We suffer a lot from the pollution.

   We do hope you can help us to take steps to protect our school and stop it from being polluted.

                       Yours truly

                  Unit 19  The Merchant of Venice  

1---15  ACBAC  ACCCA  CBABD

16--30  BBCCA  BBCBA  CBACA

31--50  ABCAC BCABD  BAADC CACAD

51--70 BCAD   75--78  ABAB   DBAC   CBAB   87--90 ADBC

71.German      72.√       73.that-what    74.understand        75. 去掉 of

76.in-on     77. 去掉 back     78.to-for      79.dumbfounded      80.not knowing

书面表达:

One possible version:

On the Internet

    The Internet is playing a more and more important role in our daily life. On the net, we can learn news both at home and abroad and all kinds of other information as well. We can also send messages by e-mail, make phone calls, go to net school, read various kinds of books and learn foreign languages by ourselves. Besides, we can enjoy music, watch sports matches and play chess or cards. On the net, we can even do shopping, have a chat with others and make friends with them. In a word, the Internet has made our life more colorful.

Unit  20  Archaeology

1---15  ACABA  DCDBC  BBBAB

16--30  ACDBD  BA BCD  CCABB

31--50  BCADC  ABCAD   BCDAC  : DABCA

51--70  ADBCB  ACACA   BCCDD   CACCB

71. busy 前加are  72. 把第一个and改为but  73. himself---hemselves

74.去掉the  75. spending 改为spend或改 to为of  76. 去掉 a

77. make---'making  78.正确  79. own---owners  80. that---if 或whether

One possible version:

   Radio and television are of great importance to us. Both of them have become an essential part of our daily life, keeping us informed of the news of the day, instructing us in many fields of interest, and entertaining us with singing, dancing and acting.

   Radio has a great effect on the world. Events of universal interest can be reported to the entire globe a few seconds after they happen. Sailors on ships at sea, pilots in planes in the air, even astronauts circling the earth are able to keep in touch with each other by means of radio.

   Television is another major instrument of communication, permitting us to see as well as to hear the performer. Since its appearance, it has had a tremendous effect on the daily life of people everywhere. A family can watch ball games on TV. It makes it possible for the entire world to be closer than ever before.

    

高 中 二 年 级 下 学 期 期 中 考 试英 语 模 拟 试 题 参考答案

1---15  BDBBD   BCADB   CCDDA

16--35  ADCCB   CDDAC   ABBBD   BACBA

36--55  CABBA   DCDBC   BDCDB   DCACB

56. have been- have 57. make-makes    58. city of- city with   59. interests-interest

60. have-have been  61. amount-number62. 正确  63. planting-planted  64. However-Besides

65. polluted-being polluted

Dear Sir ,

   I'm writing to tell you something about our discussion on the topic "Whether Middle School Students Can Carry Mobile phones (Cell phones)" at the class meeting.

   Forty percent students agreed, for they thought it was convenient for them to communicate with their relatives and friends. They also thought they could surf the Internet to search for some information and store some useful information in them. Besides, they could play games for rest if they feel tired after a long time's study.

   However, sixty percent of us were against it. They said the radiation of cell phones was harmful to health, and the ringing sound did somewhat affect the cell phone holders and others. Also, cell phones were luxurious things for middle school students and it was easy to lead them to compare with each other.

   As far as I am concerned. I agree with the majority. It is not suitable for middle school students to have cell phones. We can get in touch with others by other means and we can read more books to enrich ourselves.

Best wishes.

Yours, Kate

高中二年级下学期期末考试英语模拟试题

1---15   ACCAB   ADABB   DADDC  

16--35   ADDDB  BCACB   AABCB   ADDBD

36--55   DCACA  ACADB   DBACC   ABCDA

56.in--out 57.an easy 58.去掉for 59.from--of 60.opening---opened 61.answer--answers

62.but--so 63.don't--didn't 64.punish me 65.正确

One possible version

   More and more man-made satellites are traveling in space now. We can use communication satellites to send and receive radio and telephone signals, TV and radio programs, which has greatly helped the people in different areas to understand each other better and get in touch more easily. Weather satellites now can send information and photos of weather conditions to the weather station on the earth so that people get to know the change of weather as soon as possible. With the help of research satellites, more and more experiments have been carried out to discover the secrets of nature and space. Military satellites are playing an important part in world peace because they can fulfill different kinds of special tasks.

姓名  李 平

性别

出生年月

1984年7月

出生地

湖北省荆州市

毕业学校

南京大学计算机系

业余爱好

足球

身体状况

良好

特    长

精通微机、英语。可阅读英文科技书籍和用英语写作,稍懂日语

通信地址

湖北省荆州市第一中学

联系电话

0716-

建   厂   前

建  厂  后

校园内树木常青

各种鲜花争艳校园安静,

美似花园

工厂每天排出大量污水、

废气严重影响师生的健康

工厂的噪音也严重干扰了学校的教学活动

师生苦不堪言

信  息

看国内外新闻、获取其它信息

通  讯

发 e-mail、打电话

学  习

上网上学校、阅读各种书籍、自学外语

娱  乐

欣赏音乐、观看体育比赛、玩棋牌游戏

生  活

购  物

比例

40%学生赞同

60%学生不赞同

理由

1、便于同亲友联系;

2、可以上网查资料;

3、空余时间玩玩游戏调节大脑;

4、存储有用信息。

1、手机辐射对人体健康有害;

2、响铃影响自己和他人学习;

3、属奢侈品;

4、容易引起相互攀比。

种类

运用情况

作用与意义

通讯卫星

传递无线电信号;播放、接收广播、电视节目

有利于不同地区的人们取得联系,相互了解

气象卫星

把天气情况的资料与图片发送到地球上的气象站

使人们及时了解天气的变化情况

科研卫星

用于实验与研究

揭开大自然与太空之谜

军事卫星

完成多种任务

维护世界和平显身手